高校生のための数学の質問スレPART330

このエントリーをはてなブックマークに追加
1132人目の素数さん
前スレ
高校生のための数学の質問スレPART329
http://uni.2ch.net/test/read.cgi/math/1332861239/

【質問者必読!】
まず>>1-3をよく読んでね

数学@2ch掲示板用 掲示板での数学記号の書き方例と一般的な記号の使用例
http://mathmathmath.dotera.net/

・まずは教科書、参考書、web検索などで調べるようにしましょう。(特に基本的な公式など)
・問題の写し間違いには気をつけましょう。
・長い分母分子を含む分数はきちんと括弧でくくりましょう。
  (× x+1/x+2 ;  ○((x+1)/(x+2)) )
・丸文字、顔文字、その他は環境やブラウザによりうまく表示できない場合があります。
 どうしても画像を貼る場合はPCから直接見られるところに見やすい画像を貼ってください。
 ピクトはPCから見られないことがあるので避けてください。
・質問者は名前を騙られたくない場合、トリップを付けましょう。 (トリップの付け方は 名前(N)に 俺!#oretrip ←適当なトリ)
・質問者は回答者がわかるように問題を書くようにしましょう。でないと放置されることがあります。
  (変に省略するより全文書いた方がいい、また説明なく習慣的でない記号を使わないように)
・質問者は何が分からないのか、どこまで考えたのかを明記しましょう。それがない場合、放置されることがあります。
  (特に、自分でやってみたのにあわないので教えてほしい、みたいなときは必ず書くように)
・970くらいになったら次スレを立ててください。
2132人目の素数さん:2012/04/19(木) 23:22:31.12
基本的な記号の使い方は以下を参照してください。その他については>>1のサイトで。
■ 足し算/引き算/掛け算/割り算(加減乗除)
 a+b → a 足す b   (足し算)     a-b → a 引く b    (引き算)
 a*b → a 掛ける b  (掛け算)     a/b → a 割る b    (割り算)
■ 累乗 ^
 a^b     a の b乗
 a^(b+1)  a の b+1乗
 a^b + 1  (a の b乗) 足す 1
■ 括弧の使用
 a/(b + c) と a/b + c
 a/(b*c)  と a/b*c
 はそれぞれ、違う意味です。括弧を多用して、キチンと区別をつけてください。
■ 数列
 a[n] or a_(n)     → 数列aの第n項目
 a[n+1] = a[n] + 3  → 等差数列の一例
 Σ[k=1,n]a_(k)     → 数列の和
■ 積分 ( "∫"は「せきぶん」「いんてぐらる」「きごう」などで変換せよ(環境によって異なる).)
 ∫[0,1] x^2 dx = (x^3)/3|_[x=0,1]
 ∫[0,x] sin(t) dt
■ 三角関数
 (sin(x))^2 + (cos(x))^2 = 1
 cos(2x) = (cos(x))^2 - (sin(x))^2
■ ベクトル
 AB↑ a↑
 ベクトル:V=[V[1],V[2],...], |V>, V↑, vector(V)
 (混同しない場合はスカラーと同じ記号でいい.通常は縦ベクトルとして扱う.)
■行列
 (全成分表示):M=[[M[1,1],M[2,1],...],[M[1,2],M[2,2],...],...], I=[[1,0,0,...],[0,1,0,...],...]
 (行(または列ごと)に表示する. 例)M=[[1,-1],[3,2]])
■順列・組合せ
 P[n,k]=nPk, C[n.k]=nCk, H[n,k]=nHk
3132人目の素数さん:2012/04/19(木) 23:23:35.37
主な公式と記載例

(a±b)^2=a^2±2ab+b^2
(a±b)^3=a^3±3a^2b+3ab^2±b^3
a^3±b^3=(a±b)(a^2干ab+b^2)

√a*√b=√(ab)、√a/√b=√(a/b)、 √(a^2b)=a√b [a > 0、b > 0]
√((a+b)±2√(ab))=√a±√b [a > b > 0]

ax^2+bx+c=a(x-α)(x-β)=0 [a≠0、α+β=-b/a、αβ=c/a]
(α,β)=(-b±√(b^2-4ac))/2a  [2次方程式の解の公式]

a/sin(A)=b/sin(B)=c/sin(C)=2R [正弦定理]
a^2=b^2+c^2-2bccos(A)      [余弦定理]

sin(a±b)=sin(a)cos(b)±cos(a)sin(b)  [加法定理]
cos(a±b)=cos(a)cos(b)干sin(a)sin(b)

log_{a}(xy)=log_{a}(x)+log_{a}(y)
log_{a}(x/y)=log_{a}(x)-log_{a}(y)
log_{a}(x^n)=n(log_{a}(x))
log_{a}(x)=(log_{b}(x))/(log_{b}(a))  [底の変換定理]

f'(x)=lim_[h→0] (f(x+h)-f(x))/h  [微分の定義]
(f±g)'=f'±g'、(fg)'=f'g+fg'、(f/g)'=(f'g-fg')/(g^2) [和差積商の微分]
4132人目の素数さん:2012/04/19(木) 23:34:09.37
いちおつ
5132人目の素数さん:2012/04/20(金) 01:20:13.16
          __ノ)-'´ ̄ ̄`ー- 、_
        , '´  _. -‐'''"二ニニ=-`ヽ、
      /   /:::::; -‐''"        `ーノ
     /   /:::::/           \
     /    /::::::/          | | |  |
     |   |:::::/ /     |  | | | |  |
      |   |::/ / / |  | ||  | | ,ハ .| ,ハ|
      |   |/ / / /| ,ハノ| /|ノレ,ニ|ル' 
     |   |  | / / レ',二、レ′ ,ィイ|゙/   私は只の数ヲタなんかとは付き合わないわ。
.     |   \ ∠イ  ,イイ|    ,`-' |      頭が良くて数学が出来てかっこいい人。それが必要条件よ。
     |     l^,人|  ` `-'     ゝ  |        さらに Ann.of Math に論文書けば十分条件にもなるわよ。
      |      ` -'\       ー'  人          一番嫌いなのは論文数を増やすためにくだらない論文を書いて
    |        /(l     __/  ヽ、           良い論文の出版を遅らせるお馬鹿な人。
     |       (:::::`‐-、__  |::::`、     ヒニニヽ、         あなたの論文が Ann of Math に accept される確率は?
    |      / `‐-、::::::::::`‐-、::::\   /,ニニ、\            それとも最近は Inv. Math. の方が上かしら?
   |      |::::::::::::::::::|` -、:::::::,ヘ ̄|'、  ヒニ二、 \
.   |      /::::::::::::::::::|::::::::\/:::O`、::\   | '、   \
   |      /:::::::::::::::::::/:::::::::::::::::::::::::::::'、::::\ノ  ヽ、  |
  |      |:::::/:::::::::/:::::::::::::::::::::::::::::::::::'、',::::'、  /:\__/‐、
  |      |/:::::::::::/::::::::::::::::::::::::::::::::::O::| '、::| く::::::::::::: ̄|
   |     /_..-'´ ̄`ー-、:::::::::::::::::::::::::::::::::::|/:/`‐'::\;;;;;;;_|
   |    |/::::::::::::::::::::::\:::::::::::::::::::::::::::::|::/::::|::::/:::::::::::/
    |   /:::::::::::::::::::::::::::::::::|:::::::::::::::::::::O::|::|::::::|:::::::::::::::/
6132人目の素数さん:2012/04/20(金) 07:05:47.78
          __ノ)-'´ ̄ ̄`ー- 、_
        , '´  _. -‐'''"二ニニ=-`ヽ、
      /   /:::::; -‐''"        `ーノ
     /   /:::::/           \
     /    /::::::/          | | |  |
     |   |:::::/ /     |  | | | |  |
      |   |::/ / / |  | ||  | | ,ハ .| ,ハ|
      |   |/ / / /| ,ハノ| /|ノレ,ニ|ル' 
     |   |  | / / レ',二、レ′ ,ィイ|゙/   私は只の数ヲタなんかとは付き合わないわ。
.     |   \ ∠イ  ,イイ|    ,`-' |      頭が良くて数学が出来てかっこいい人。それが必要条件よ。
     |     l^,人|  ` `-'     ゝ  |        さらに Ann.of Math に論文書けば十分条件にもなるわよ。
      |      ` -'\       ー'  人          一番嫌いなのは論文数を増やすためにくだらない論文を書いて
    |        /(l     __/  ヽ、           良い論文の出版を遅らせるお馬鹿な人。
     |       (:::::`‐-、__  |::::`、     ヒニニヽ、         あなたの論文が Ann of Math に accept される確率は?
    |      / `‐-、::::::::::`‐-、::::\   /,ニニ、\            それとも最近は Inv. Math. の方が上かしら?
   |      |::::::::::::::::::|` -、:::::::,ヘ ̄|'、  ヒニ二、 \
.   |      /::::::::::::::::::|::::::::\/:::O`、::\   | '、   \
   |      /:::::::::::::::::::/:::::::::::::::::::::::::::::'、::::\ノ  ヽ、  |
  |      |:::::/:::::::::/:::::::::::::::::::::::::::::::::::'、',::::'、  /:\__/‐、
  |      |/:::::::::::/::::::::::::::::::::::::::::::::::O::| '、::| く::::::::::::: ̄|
   |     /_..-'´ ̄`ー-、:::::::::::::::::::::::::::::::::::|/:/`‐'::\;;;;;;;_|
   |    |/::::::::::::::::::::::\:::::::::::::::::::::::::::::|::/::::|::::/:::::::::::/
    |   /:::::::::::::::::::::::::::::::::|:::::::::::::::::::::O::|::|::::::|:::::::::::::::/
7132人目の素数さん:2012/04/20(金) 07:07:25.88
          __ノ)-'´ ̄ ̄`ー- 、_
        , '´  _. -‐'''"二ニニ=-`ヽ、
      /   /:::::; -‐''"        `ーノ
     /   /:::::/           \
     /    /::::::/          | | |  |
     |   |:::::/ /     |  | | | |  |
      |   |::/ / / |  | ||  | | ,ハ .| ,ハ|
      |   |/ / / /| ,ハノ| /|ノレ,ニ|ル' 
     |   |  | / / レ',二、レ′ ,ィイ|゙/   私は只の数ヲタなんかとは付き合わないわ。
.     |   \ ∠イ  ,イイ|    ,`-' |      頭が良くて数学が出来てかっこいい人。それが必要条件よ。
     |     l^,人|  ` `-'     ゝ  |        さらに Ann.of Math に論文書けば十分条件にもなるわよ。
      |      ` -'\       ー'  人          一番嫌いなのは論文数を増やすためにくだらない論文を書いて
    |        /(l     __/  ヽ、           良い論文の出版を遅らせるお馬鹿な人。
     |       (:::::`‐-、__  |::::`、     ヒニニヽ、         あなたの論文が Ann of Math に accept される確率は?
    |      / `‐-、::::::::::`‐-、::::\   /,ニニ、\            それとも最近は Inv. Math. の方が上かしら?
   |      |::::::::::::::::::|` -、:::::::,ヘ ̄|'、  ヒニ二、 \
.   |      /::::::::::::::::::|::::::::\/:::O`、::\   | '、   \
   |      /:::::::::::::::::::/:::::::::::::::::::::::::::::'、::::\ノ  ヽ、  |
  |      |:::::/:::::::::/:::::::::::::::::::::::::::::::::::'、',::::'、  /:\__/‐、
  |      |/:::::::::::/::::::::::::::::::::::::::::::::::O::| '、::| く::::::::::::: ̄|
   |     /_..-'´ ̄`ー-、:::::::::::::::::::::::::::::::::::|/:/`‐'::\;;;;;;;_|
   |    |/::::::::::::::::::::::\:::::::::::::::::::::::::::::|::/::::|::::/:::::::::::/
    |   /:::::::::::::::::::::::::::::::::|:::::::::::::::::::::O::|::|::::::|:::::::::::::::/
8132人目の素数さん:2012/04/20(金) 07:11:27.89
          __ノ)-'´ ̄ ̄`ー- 、_
        , '´  _. -‐'''"二ニニ=-`ヽ、
      /   /:::::; -‐''"        `ーノ
     /   /:::::/           \
     /    /::::::/          | | |  |
     |   |:::::/ /     |  | | | |  |
      |   |::/ / / |  | ||  | | ,ハ .| ,ハ|
      |   |/ / / /| ,ハノ| /|ノレ,ニ|ル' 
     |   |  | / / レ',二、レ′ ,ィイ|゙/   私は只の数ヲタなんかとは付き合わないわ。
.     |   \ ∠イ  ,イイ|    ,`-' |      頭が良くて数学が出来てかっこいい人。それが必要条件よ。
     |     l^,人|  ` `-'     ゝ  |        さらに Ann.of Math に論文書けば十分条件にもなるわよ。
      |      ` -'\       ー'  人          一番嫌いなのは論文数を増やすためにくだらない論文を書いて
    |        /(l     __/  ヽ、           良い論文の出版を遅らせるお馬鹿な人。
     |       (:::::`‐-、__  |::::`、     ヒニニヽ、         あなたの論文が Ann of Math に accept される確率は?
    |      / `‐-、::::::::::`‐-、::::\   /,ニニ、\            それとも最近は Inv. Math. の方が上かしら?
   |      |::::::::::::::::::|` -、:::::::,ヘ ̄|'、  ヒニ二、 \
.   |      /::::::::::::::::::|::::::::\/:::O`、::\   | '、   \
   |      /:::::::::::::::::::/:::::::::::::::::::::::::::::'、::::\ノ  ヽ、  |
  |      |:::::/:::::::::/:::::::::::::::::::::::::::::::::::'、',::::'、  /:\__/‐、
  |      |/:::::::::::/::::::::::::::::::::::::::::::::::O::| '、::| く::::::::::::: ̄|
   |     /_..-'´ ̄`ー-、:::::::::::::::::::::::::::::::::::|/:/`‐'::\;;;;;;;_|
   |    |/::::::::::::::::::::::\:::::::::::::::::::::::::::::|::/::::|::::/:::::::::::/
    |   /:::::::::::::::::::::::::::::::::|:::::::::::::::::::::O::|::|::::::|:::::::::::::::/
9132人目の素数さん:2012/04/20(金) 07:14:19.08
          __ノ)-'´ ̄ ̄`ー- 、_
        , '´  _. -‐'''"二ニニ=-`ヽ、
      /   /:::::; -‐''"        `ーノ
     /   /:::::/           \
     /    /::::::/          | | |  |
     |   |:::::/ /     |  | | | |  |
      |   |::/ / / |  | ||  | | ,ハ .| ,ハ|
      |   |/ / / /| ,ハノ| /|ノレ,ニ|ル' 
     |   |  | / / レ',二、レ′ ,ィイ|゙/   私は只の数ヲタなんかとは付き合わないわ。
.     |   \ ∠イ  ,イイ|    ,`-' |      頭が良くて数学が出来てかっこいい人。それが必要条件よ。
     |     l^,人|  ` `-'     ゝ  |        さらに Ann.of Math に論文書けば十分条件にもなるわよ。
      |      ` -'\       ー'  人          一番嫌いなのは論文数を増やすためにくだらない論文を書いて
    |        /(l     __/  ヽ、           良い論文の出版を遅らせるお馬鹿な人。
     |       (:::::`‐-、__  |::::`、     ヒニニヽ、         あなたの論文が Ann of Math に accept される確率は?
    |      / `‐-、::::::::::`‐-、::::\   /,ニニ、\            それとも最近は Inv. Math. の方が上かしら?
   |      |::::::::::::::::::|` -、:::::::,ヘ ̄|'、  ヒニ二、 \
.   |      /::::::::::::::::::|::::::::\/:::O`、::\   | '、   \
   |      /:::::::::::::::::::/:::::::::::::::::::::::::::::'、::::\ノ  ヽ、  |
  |      |:::::/:::::::::/:::::::::::::::::::::::::::::::::::'、',::::'、  /:\__/‐、
  |      |/:::::::::::/::::::::::::::::::::::::::::::::::O::| '、::| く::::::::::::: ̄|
   |     /_..-'´ ̄`ー-、:::::::::::::::::::::::::::::::::::|/:/`‐'::\;;;;;;;_|
   |    |/::::::::::::::::::::::\:::::::::::::::::::::::::::::|::/::::|::::/:::::::::::/
    |   /:::::::::::::::::::::::::::::::::|:::::::::::::::::::::O::|::|::::::|:::::::::::::::/
10132人目の素数さん:2012/04/20(金) 07:14:44.71
          __ノ)-'´ ̄ ̄`ー- 、_
        , '´  _. -‐'''"二ニニ=-`ヽ、
      /   /:::::; -‐''"        `ーノ
     /   /:::::/           \
     /    /::::::/          | | |  |
     |   |:::::/ /     |  | | | |  |
      |   |::/ / / |  | ||  | | ,ハ .| ,ハ|
      |   |/ / / /| ,ハノ| /|ノレ,ニ|ル' 
     |   |  | / / レ',二、レ′ ,ィイ|゙/   私は只の数ヲタなんかとは付き合わないわ。
.     |   \ ∠イ  ,イイ|    ,`-' |      頭が良くて数学が出来てかっこいい人。それが必要条件よ。
     |     l^,人|  ` `-'     ゝ  |        さらに Ann.of Math に論文書けば十分条件にもなるわよ。
      |      ` -'\       ー'  人          一番嫌いなのは論文数を増やすためにくだらない論文を書いて
    |        /(l     __/  ヽ、           良い論文の出版を遅らせるお馬鹿な人。
     |       (:::::`‐-、__  |::::`、     ヒニニヽ、         あなたの論文が Ann of Math に accept される確率は?
    |      / `‐-、::::::::::`‐-、::::\   /,ニニ、\            それとも最近は Inv. Math. の方が上かしら?
   |      |::::::::::::::::::|` -、:::::::,ヘ ̄|'、  ヒニ二、 \
.   |      /::::::::::::::::::|::::::::\/:::O`、::\   | '、   \
   |      /:::::::::::::::::::/:::::::::::::::::::::::::::::'、::::\ノ  ヽ、  |
  |      |:::::/:::::::::/:::::::::::::::::::::::::::::::::::'、',::::'、  /:\__/‐、
  |      |/:::::::::::/::::::::::::::::::::::::::::::::::O::| '、::| く::::::::::::: ̄|
   |     /_..-'´ ̄`ー-、:::::::::::::::::::::::::::::::::::|/:/`‐'::\;;;;;;;_|
   |    |/::::::::::::::::::::::\:::::::::::::::::::::::::::::|::/::::|::::/:::::::::::/
    |   /:::::::::::::::::::::::::::::::::|:::::::::::::::::::::O::|::|::::::|:::::::::::::::/
11132人目の素数さん:2012/04/20(金) 07:15:48.14
          __ノ)-'´ ̄ ̄`ー- 、_
        , '´  _. -‐'''"二ニニ=-`ヽ、
      /   /:::::; -‐''"        `ーノ
     /   /:::::/           \
     /    /::::::/          | | |  |
     |   |:::::/ /     |  | | | |  |
      |   |::/ / / |  | ||  | | ,ハ .| ,ハ|
      |   |/ / / /| ,ハノ| /|ノレ,ニ|ル' 
     |   |  | / / レ',二、レ′ ,ィイ|゙/   私は只の数ヲタなんかとは付き合わないわ。
.     |   \ ∠イ  ,イイ|    ,`-' |      頭が良くて数学が出来てかっこいい人。それが必要条件よ。
     |     l^,人|  ` `-'     ゝ  |        さらに Ann.of Math に論文書けば十分条件にもなるわよ。
      |      ` -'\       ー'  人          一番嫌いなのは論文数を増やすためにくだらない論文を書いて
    |        /(l     __/  ヽ、           良い論文の出版を遅らせるお馬鹿な人。
     |       (:::::`‐-、__  |::::`、     ヒニニヽ、         あなたの論文が Ann of Math に accept される確率は?
    |      / `‐-、::::::::::`‐-、::::\   /,ニニ、\            それとも最近は Inv. Math. の方が上かしら?
   |      |::::::::::::::::::|` -、:::::::,ヘ ̄|'、  ヒニ二、 \
.   |      /::::::::::::::::::|::::::::\/:::O`、::\   | '、   \
   |      /:::::::::::::::::::/:::::::::::::::::::::::::::::'、::::\ノ  ヽ、  |
  |      |:::::/:::::::::/:::::::::::::::::::::::::::::::::::'、',::::'、  /:\__/‐、
  |      |/:::::::::::/::::::::::::::::::::::::::::::::::O::| '、::| く::::::::::::: ̄|
   |     /_..-'´ ̄`ー-、:::::::::::::::::::::::::::::::::::|/:/`‐'::\;;;;;;;_|
   |    |/::::::::::::::::::::::\:::::::::::::::::::::::::::::|::/::::|::::/:::::::::::/
    |   /:::::::::::::::::::::::::::::::::|:::::::::::::::::::::O::|::|::::::|:::::::::::::::/
12132人目の素数さん:2012/04/20(金) 07:21:26.35
          __ノ)-'´ ̄ ̄`ー- 、_
        , '´  _. -‐'''"二ニニ=-`ヽ、
      /   /:::::; -‐''"        `ーノ
     /   /:::::/           \
     /    /::::::/          | | |  |
     |   |:::::/ /     |  | | | |  |
      |   |::/ / / |  | ||  | | ,ハ .| ,ハ|
      |   |/ / / /| ,ハノ| /|ノレ,ニ|ル' 
     |   |  | / / レ',二、レ′ ,ィイ|゙/   私は只の数ヲタなんかとは付き合わないわ。
.     |   \ ∠イ  ,イイ|    ,`-' |      頭が良くて数学が出来てかっこいい人。それが必要条件よ。
     |     l^,人|  ` `-'     ゝ  |        さらに Ann.of Math に論文書けば十分条件にもなるわよ。
      |      ` -'\       ー'  人          一番嫌いなのは論文数を増やすためにくだらない論文を書いて
    |        /(l     __/  ヽ、           良い論文の出版を遅らせるお馬鹿な人。
     |       (:::::`‐-、__  |::::`、     ヒニニヽ、         あなたの論文が Ann of Math に accept される確率は?
    |      / `‐-、::::::::::`‐-、::::\   /,ニニ、\            それとも最近は Inv. Math. の方が上かしら?
   |      |::::::::::::::::::|` -、:::::::,ヘ ̄|'、  ヒニ二、 \
.   |      /::::::::::::::::::|::::::::\/:::O`、::\   | '、   \
   |      /:::::::::::::::::::/:::::::::::::::::::::::::::::'、::::\ノ  ヽ、  |
  |      |:::::/:::::::::/:::::::::::::::::::::::::::::::::::'、',::::'、  /:\__/‐、
  |      |/:::::::::::/::::::::::::::::::::::::::::::::::O::| '、::| く::::::::::::: ̄|
   |     /_..-'´ ̄`ー-、:::::::::::::::::::::::::::::::::::|/:/`‐'::\;;;;;;;_|
   |    |/::::::::::::::::::::::\:::::::::::::::::::::::::::::|::/::::|::::/:::::::::::/
    |   /:::::::::::::::::::::::::::::::::|:::::::::::::::::::::O::|::|::::::|:::::::::::::::/
13132人目の素数さん:2012/04/20(金) 07:48:55.53
いつから名前がバカオツなんだか
キチガイアホ晒し
14132人目の素数さん:2012/04/20(金) 08:06:56.58
          __ノ)-'´ ̄ ̄`ー- 、_
        , '´  _. -‐'''"二ニニ=-`ヽ、
      /   /:::::; -‐''"        `ーノ
     /   /:::::/           \
     /    /::::::/          | | |  |
     |   |:::::/ /     |  | | | |  |
      |   |::/ / / |  | ||  | | ,ハ .| ,ハ|
      |   |/ / / /| ,ハノ| /|ノレ,ニ|ル' 
     |   |  | / / レ',二、レ′ ,ィイ|゙/   私は只の数ヲタなんかとは付き合わないわ。
.     |   \ ∠イ  ,イイ|    ,`-' |      頭が良くて数学が出来てかっこいい人。それが必要条件よ。
     |     l^,人|  ` `-'     ゝ  |        さらに Ann.of Math に論文書けば十分条件にもなるわよ。
      |      ` -'\       ー'  人          一番嫌いなのは論文数を増やすためにくだらない論文を書いて
    |        /(l     __/  ヽ、           良い論文の出版を遅らせるお馬鹿な人。
     |       (:::::`‐-、__  |::::`、     ヒニニヽ、         あなたの論文が Ann of Math に accept される確率は?
    |      / `‐-、::::::::::`‐-、::::\   /,ニニ、\            それとも最近は Inv. Math. の方が上かしら?
   |      |::::::::::::::::::|` -、:::::::,ヘ ̄|'、  ヒニ二、 \
.   |      /::::::::::::::::::|::::::::\/:::O`、::\   | '、   \
   |      /:::::::::::::::::::/:::::::::::::::::::::::::::::'、::::\ノ  ヽ、  |
  |      |:::::/:::::::::/:::::::::::::::::::::::::::::::::::'、',::::'、  /:\__/‐、
  |      |/:::::::::::/::::::::::::::::::::::::::::::::::O::| '、::| く::::::::::::: ̄|
   |     /_..-'´ ̄`ー-、:::::::::::::::::::::::::::::::::::|/:/`‐'::\;;;;;;;_|
   |    |/::::::::::::::::::::::\:::::::::::::::::::::::::::::|::/::::|::::/:::::::::::/
    |   /:::::::::::::::::::::::::::::::::|:::::::::::::::::::::O::|::|::::::|:::::::::::::::/
15132人目の素数さん:2012/04/20(金) 08:17:02.30
          __ノ)-'´ ̄ ̄`ー- 、_
        , '´  _. -‐'''"二ニニ=-`ヽ、
      /   /:::::; -‐''"        `ーノ
     /   /:::::/           \
     /    /::::::/          | | |  |
     |   |:::::/ /     |  | | | |  |
      |   |::/ / / |  | ||  | | ,ハ .| ,ハ|
      |   |/ / / /| ,ハノ| /|ノレ,ニ|ル' 
     |   |  | / / レ',二、レ′ ,ィイ|゙/   私は只の数ヲタなんかとは付き合わないわ。
.     |   \ ∠イ  ,イイ|    ,`-' |      頭が良くて数学が出来てかっこいい人。それが必要条件よ。
     |     l^,人|  ` `-'     ゝ  |        さらに Ann.of Math に論文書けば十分条件にもなるわよ。
      |      ` -'\       ー'  人          一番嫌いなのは論文数を増やすためにくだらない論文を書いて
    |        /(l     __/  ヽ、           良い論文の出版を遅らせるお馬鹿な人。
     |       (:::::`‐-、__  |::::`、     ヒニニヽ、         あなたの論文が Ann of Math に accept される確率は?
    |      / `‐-、::::::::::`‐-、::::\   /,ニニ、\            それとも最近は Inv. Math. の方が上かしら?
   |      |::::::::::::::::::|` -、:::::::,ヘ ̄|'、  ヒニ二、 \
.   |      /::::::::::::::::::|::::::::\/:::O`、::\   | '、   \
   |      /:::::::::::::::::::/:::::::::::::::::::::::::::::'、::::\ノ  ヽ、  |
  |      |:::::/:::::::::/:::::::::::::::::::::::::::::::::::'、',::::'、  /:\__/‐、
  |      |/:::::::::::/::::::::::::::::::::::::::::::::::O::| '、::| く::::::::::::: ̄|
   |     /_..-'´ ̄`ー-、:::::::::::::::::::::::::::::::::::|/:/`‐'::\;;;;;;;_|
   |    |/::::::::::::::::::::::\:::::::::::::::::::::::::::::|::/::::|::::/:::::::::::/
    |   /:::::::::::::::::::::::::::::::::|:::::::::::::::::::::O::|::|::::::|:::::::::::::::/
16132人目の素数さん:2012/04/20(金) 08:19:35.89
          __ノ)-'´ ̄ ̄`ー- 、_
        , '´  _. -‐'''"二ニニ=-`ヽ、
      /   /:::::; -‐''"        `ーノ
     /   /:::::/           \
     /    /::::::/          | | |  |
     |   |:::::/ /     |  | | | |  |
      |   |::/ / / |  | ||  | | ,ハ .| ,ハ|
      |   |/ / / /| ,ハノ| /|ノレ,ニ|ル' 
     |   |  | / / レ',二、レ′ ,ィイ|゙/   私は只の数ヲタなんかとは付き合わないわ。
.     |   \ ∠イ  ,イイ|    ,`-' |      頭が良くて数学が出来てかっこいい人。それが必要条件よ。
     |     l^,人|  ` `-'     ゝ  |        さらに Ann.of Math に論文書けば十分条件にもなるわよ。
      |      ` -'\       ー'  人          一番嫌いなのは論文数を増やすためにくだらない論文を書いて
    |        /(l     __/  ヽ、           良い論文の出版を遅らせるお馬鹿な人。
     |       (:::::`‐-、__  |::::`、     ヒニニヽ、         あなたの論文が Ann of Math に accept される確率は?
    |      / `‐-、::::::::::`‐-、::::\   /,ニニ、\            それとも最近は Inv. Math. の方が上かしら?
   |      |::::::::::::::::::|` -、:::::::,ヘ ̄|'、  ヒニ二、 \
.   |      /::::::::::::::::::|::::::::\/:::O`、::\   | '、   \
   |      /:::::::::::::::::::/:::::::::::::::::::::::::::::'、::::\ノ  ヽ、  |
  |      |:::::/:::::::::/:::::::::::::::::::::::::::::::::::'、',::::'、  /:\__/‐、
  |      |/:::::::::::/::::::::::::::::::::::::::::::::::O::| '、::| く::::::::::::: ̄|
   |     /_..-'´ ̄`ー-、:::::::::::::::::::::::::::::::::::|/:/`‐'::\;;;;;;;_|
   |    |/::::::::::::::::::::::\:::::::::::::::::::::::::::::|::/::::|::::/:::::::::::/
    |   /:::::::::::::::::::::::::::::::::|:::::::::::::::::::::O::|::|::::::|:::::::::::::::/
17132人目の素数さん:2012/04/20(金) 08:32:39.02
          __ノ)-'´ ̄ ̄`ー- 、_
        , '´  _. -‐'''"二ニニ=-`ヽ、
      /   /:::::; -‐''"        `ーノ
     /   /:::::/           \
     /    /::::::/          | | |  |
     |   |:::::/ /     |  | | | |  |
      |   |::/ / / |  | ||  | | ,ハ .| ,ハ|
      |   |/ / / /| ,ハノ| /|ノレ,ニ|ル' 
     |   |  | / / レ',二、レ′ ,ィイ|゙/   私は只の数ヲタなんかとは付き合わないわ。
.     |   \ ∠イ  ,イイ|    ,`-' |      頭が良くて数学が出来てかっこいい人。それが必要条件よ。
     |     l^,人|  ` `-'     ゝ  |        さらに Ann.of Math に論文書けば十分条件にもなるわよ。
      |      ` -'\       ー'  人          一番嫌いなのは論文数を増やすためにくだらない論文を書いて
    |        /(l     __/  ヽ、           良い論文の出版を遅らせるお馬鹿な人。
     |       (:::::`‐-、__  |::::`、     ヒニニヽ、         あなたの論文が Ann of Math に accept される確率は?
    |      / `‐-、::::::::::`‐-、::::\   /,ニニ、\            それとも最近は Inv. Math. の方が上かしら?
   |      |::::::::::::::::::|` -、:::::::,ヘ ̄|'、  ヒニ二、 \
.   |      /::::::::::::::::::|::::::::\/:::O`、::\   | '、   \
   |      /:::::::::::::::::::/:::::::::::::::::::::::::::::'、::::\ノ  ヽ、  |
  |      |:::::/:::::::::/:::::::::::::::::::::::::::::::::::'、',::::'、  /:\__/‐、
  |      |/:::::::::::/::::::::::::::::::::::::::::::::::O::| '、::| く::::::::::::: ̄|
   |     /_..-'´ ̄`ー-、:::::::::::::::::::::::::::::::::::|/:/`‐'::\;;;;;;;_|
   |    |/::::::::::::::::::::::\:::::::::::::::::::::::::::::|::/::::|::::/:::::::::::/
    |   /:::::::::::::::::::::::::::::::::|:::::::::::::::::::::O::|::|::::::|:::::::::::::::/
18132人目の素数さん:2012/04/20(金) 08:46:21.88
          __ノ)-'´ ̄ ̄`ー- 、_
        , '´  _. -‐'''"二ニニ=-`ヽ、
      /   /:::::; -‐''"        `ーノ
     /   /:::::/           \
     /    /::::::/          | | |  |
     |   |:::::/ /     |  | | | |  |
      |   |::/ / / |  | ||  | | ,ハ .| ,ハ|
      |   |/ / / /| ,ハノ| /|ノレ,ニ|ル' 
     |   |  | / / レ',二、レ′ ,ィイ|゙/   私は只の数ヲタなんかとは付き合わないわ。
.     |   \ ∠イ  ,イイ|    ,`-' |      頭が良くて数学が出来てかっこいい人。それが必要条件よ。
     |     l^,人|  ` `-'     ゝ  |        さらに Ann.of Math に論文書けば十分条件にもなるわよ。
      |      ` -'\       ー'  人          一番嫌いなのは論文数を増やすためにくだらない論文を書いて
    |        /(l     __/  ヽ、           良い論文の出版を遅らせるお馬鹿な人。
     |       (:::::`‐-、__  |::::`、     ヒニニヽ、         あなたの論文が Ann of Math に accept される確率は?
    |      / `‐-、::::::::::`‐-、::::\   /,ニニ、\            それとも最近は Inv. Math. の方が上かしら?
   |      |::::::::::::::::::|` -、:::::::,ヘ ̄|'、  ヒニ二、 \
.   |      /::::::::::::::::::|::::::::\/:::O`、::\   | '、   \
   |      /:::::::::::::::::::/:::::::::::::::::::::::::::::'、::::\ノ  ヽ、  |
  |      |:::::/:::::::::/:::::::::::::::::::::::::::::::::::'、',::::'、  /:\__/‐、
  |      |/:::::::::::/::::::::::::::::::::::::::::::::::O::| '、::| く::::::::::::: ̄|
   |     /_..-'´ ̄`ー-、:::::::::::::::::::::::::::::::::::|/:/`‐'::\;;;;;;;_|
   |    |/::::::::::::::::::::::\:::::::::::::::::::::::::::::|::/::::|::::/:::::::::::/
    |   /:::::::::::::::::::::::::::::::::|:::::::::::::::::::::O::|::|::::::|:::::::::::::::/
19132人目の素数さん:2012/04/20(金) 09:57:00.55
>>5-18があぼーんなんだがwww
20132人目の素数さん:2012/04/20(金) 11:55:27.17
リアルで嫌なことでもあって荒れてるんだろう
寄生先の親と喧嘩でもしたか?
21132人目の素数さん:2012/04/20(金) 12:54:18.14
物理できない奴って啓蒙書や科学雑誌読んで自己流に納得して勘違いする奴いるけど、
数学できない奴って、ごまかし効かないからな。コンプレックスも深いんだろさ。
22132人目の素数さん:2012/04/20(金) 14:32:50.13
僅かながら選挙の匂いが漂ってまいりました。

                    _____________
                    ||                      |
                    ||   ちょっと待て .  .     .|
                    ||         .           |
                    ||    その民主党員  . |
                    |l -――-               |
                     '"´: : : : : : : : :`丶 . 帰化鮮人|
                 ':.:.:.:.:.:.:.:.:.:.:.:.:.:.:.:.:.:.:.:.:.:ヽ______|
                  /.::.::./.::.::.::.:j.::.::.:|.:ム;ヘ.::.:ハ ̄ ̄ ̄ ̄ ̄
                  ,'.::.::.::i.::.::.::.:/|.::.:: l/  `|.::./7
                :.::.::.::j:|.:!.:_:/´|_.::_」   くV <|
                   |:ハ_::_ル'´     /⌒丶 j//V|
               |:::::::::i x==ミ     _ 〈/.:|.::|
               |:::::::::i:'"     ´ ゙̄Y}!.::.l.::|
                 八:::::::圦   、' _   "/_ノ.::,'.::j
             /⌒ヽ::::ト{\   _,.ィ__/.::/l:./
               / 丶∧::| 丶 `ニ´ 彡// :厶|∧
            {/  丶ヘ|     ノ / |:/ (こ ハ
                /       }ヽ、 ∧ /  'x┴〈 }_ゝ、
           /         \∨ ∨  /  ニW }  )
             〈       _ノ∧ 厶=7  ,.-、) 人ノ
           }⌒ヽ     `<__,>イ  |__ノ| |/∨
           /   ヘ   /  │  丶ノ.| |   \
             /    ヽ      \__/ | |    ノ
         /       >'"⌒\ 〃⌒\| ト、__/
23132人目の素数さん:2012/04/20(金) 16:34:51.62
>>13
バカオツ
24132人目の素数さん:2012/04/20(金) 16:59:29.42
前スレ996バカオツ
25132人目の素数さん:2012/04/20(金) 21:05:16.02
数学を勉強する意義について議論するスレ
http://uni.2ch.net/test/read.cgi/math/1334923127/
26132人目の素数さん:2012/04/20(金) 21:05:41.54
          __ノ)-'´ ̄ ̄`ー- 、_
        , '´  _. -‐'''"二ニニ=-`ヽ、
      /   /:::::; -‐''"        `ーノ
     /   /:::::/           \
     /    /::::::/          | | |  |
     |   |:::::/ /     |  | | | |  |
      |   |::/ / / |  | ||  | | ,ハ .| ,ハ|
      |   |/ / / /| ,ハノ| /|ノレ,ニ|ル' 
     |   |  | / / レ',二、レ′ ,ィイ|゙/   私は只の数ヲタなんかとは付き合わないわ。
.     |   \ ∠イ  ,イイ|    ,`-' |      頭が良くて数学が出来てかっこいい人。それが必要条件よ。
     |     l^,人|  ` `-'     ゝ  |        さらに Ann.of Math に論文書けば十分条件にもなるわよ。
      |      ` -'\       ー'  人          一番嫌いなのは論文数を増やすためにくだらない論文を書いて
    |        /(l     __/  ヽ、           良い論文の出版を遅らせるお馬鹿な人。
     |       (:::::`‐-、__  |::::`、     ヒニニヽ、         あなたの論文が Ann of Math に accept される確率は?
    |      / `‐-、::::::::::`‐-、::::\   /,ニニ、\            それとも最近は Inv. Math. の方が上かしら?
   |      |::::::::::::::::::|` -、:::::::,ヘ ̄|'、  ヒニ二、 \
.   |      /::::::::::::::::::|::::::::\/:::O`、::\   | '、   \
   |      /:::::::::::::::::::/:::::::::::::::::::::::::::::'、::::\ノ  ヽ、  |
  |      |:::::/:::::::::/:::::::::::::::::::::::::::::::::::'、',::::'、  /:\__/‐、
  |      |/:::::::::::/::::::::::::::::::::::::::::::::::O::| '、::| く::::::::::::: ̄|
   |     /_..-'´ ̄`ー-、:::::::::::::::::::::::::::::::::::|/:/`‐'::\;;;;;;;_|
   |    |/::::::::::::::::::::::\:::::::::::::::::::::::::::::|::/::::|::::/:::::::::::/
    |   /:::::::::::::::::::::::::::::::::|:::::::::::::::::::::O::|::|::::::|:::::::::::::::/
27132人目の素数さん:2012/04/20(金) 21:36:35.58
640 名前:名無しさん@12周年[] 投稿日:2012/02/18(土) 15:05:47.13 ID:sskgsjsc0 [2/2]
『平清盛』プロデューサー在日朝鮮人 磯智明(天皇制度廃止論者)のプロデュース作品

@『かんさほうじん (2008)』反体制・反社会
A『最後の戦犯 (2008)』反日・天皇制度廃止・反体制・反社会
B『リミット -刑事の現場2- (2009)』反体制・反社会

大河の画面が汚いのも、役者が大根なのも、衣装がぼろぼろなのも、役者の下品な立ち回りも、
画面が薄暗いのも、役者が汚いのも全ての原因は




NHKが汚れているから
 史実うんぬんの話ではないのですよ。あの大河は役者、セット、演出等が、いまのNHK内部の汚れ具合を見事に反映しているのです。

薄汚れた空間内で繰り広げられる捏造・妄想(=今年の大河)は、反日・在日の脳内を表しているのだ。
28132人目の素数さん:2012/04/21(土) 00:30:16.26
x^4-8

この式って因数分解できます?
29132人目の素数さん:2012/04/21(土) 00:40:04.71
(x^2+8^(1/2))(x+8^(1/4))(x-8^(1/4))
30132人目の素数さん:2012/04/21(土) 00:41:08.90
ありがとう

因数分解って定数項が整数じゃなくてもいいのかな
31132人目の素数さん:2012/04/21(土) 00:43:23.56
つまり整数係数じゃなくていいのかな
32132人目の素数さん:2012/04/21(土) 00:51:25.33
問題によるんじゃない?
=0の時の解を求める場合なら虚数が出てきてもOKなんだし
33132人目の素数さん:2012/04/21(土) 00:53:27.24
なるほどありがとう


今日テストでx^4-8を因数分解せよってだけ出てて
どう解釈すれば良いか分からなくて焦ったwww
34132人目の素数さん:2012/04/21(土) 01:17:07.72
足し算と掛け算ができる世界(環)の種類を指定しなきゃなあ
有理数、実数、複素数などあるから…
35132人目の素数さん:2012/04/21(土) 05:11:05.00
10本のくじの中に2本の当たりくじがある。当たりくじを3回引くまで繰り返し
くじを引くものとする。ただし、一度引いたくじは毎回元に戻すものとする。
n回目で終わる確立をPnとするとき、次の問に答えよ。

Pnが最大となるnを求めよ
という問題で問題自体の意味が分かりません

Pnはn回目で終わる確率のことを言うのだから最大は3回目で終わる確率じゃないのでしょうか
問題は何を言っているのでしょうか 誰か詳しくお願いします
36132人目の素数さん:2012/04/21(土) 05:34:50.66
にほんごのおべんきょうをやりなおし。
37132人目の素数さん:2012/04/21(土) 05:36:14.82
当たりくじを3回引くまで繰り返し…とあるのだから3回目で終わるためには1/5の確率を3連続で当てるミラクルが必要になる。
「3回目の当たり」を引いたのがちょうどn回目のくじ引きである、という確率がP_n。
38KingMathematician ◆LoZDre77j4i1 :2012/04/21(土) 05:50:12.33
つまり,なおさら若いうちに数学を修得しなくてはならない.

x^4-8を有理数係数整式の範囲で因数分解するとき,一次の因数があるかどうか,無い場合は二次の因数があるかどうか調べればよい.
これに限らず四次式と五次式ではそうなる.二次式と三次式の場合は一次の因数があるかどうか調べればよい.
39132人目の素数さん:2012/04/21(土) 05:52:08.96
>>37
はあ。。。なんとなくは分かりました、、、

解答に自然数nのP_nが最大となるnの値を求めるにはP_n+1-P_nの符号を調べて
P_n+1とP_nの大小を比較する

とあるのですがP_n+1-P_nをする意味って何なのでしょうか・・・馬鹿ですいません
40132人目の素数さん:2012/04/21(土) 06:10:40.57
>>39
A > B を証明するときに A - B > 0 を代わりに示すということを
不等式のところでやっているはず

もっとも,反復試行の最大確率では,
  ・ P[k] と P[k+1] はよく似た式になる
  ・ P[k] は正である
ことを生かして
  P[k] <=> P[k+1] …@ ⇔ ( P[k] )/( P[k+1] ) <=> 1 …A
 (不等号同順,Aで「<」なら@でも「<」)
と,差と0ではなくて「比と1を比較する」ことも多い
約分できるところがたくさんあるので式がすっきりする
41132人目の素数さん:2012/04/21(土) 07:44:24.66
          __ノ)-'´ ̄ ̄`ー- 、_
        , '´  _. -‐'''"二ニニ=-`ヽ、
      /   /:::::; -‐''"        `ーノ
     /   /:::::/           \
     /    /::::::/          | | |  |
     |   |:::::/ /     |  | | | |  |
      |   |::/ / / |  | ||  | | ,ハ .| ,ハ|
      |   |/ / / /| ,ハノ| /|ノレ,ニ|ル' 
     |   |  | / / レ',二、レ′ ,ィイ|゙/   私は只の数ヲタなんかとは付き合わないわ。
.     |   \ ∠イ  ,イイ|    ,`-' |      頭が良くて数学が出来てかっこいい人。それが必要条件よ。
     |     l^,人|  ` `-'     ゝ  |        さらに Ann.of Math に論文書けば十分条件にもなるわよ。
      |      ` -'\       ー'  人          一番嫌いなのは論文数を増やすためにくだらない論文を書いて
    |        /(l     __/  ヽ、           良い論文の出版を遅らせるお馬鹿な人。
     |       (:::::`‐-、__  |::::`、     ヒニニヽ、         あなたの論文が Ann of Math に accept される確率は?
    |      / `‐-、::::::::::`‐-、::::\   /,ニニ、\            それとも最近は Inv. Math. の方が上かしら?
   |      |::::::::::::::::::|` -、:::::::,ヘ ̄|'、  ヒニ二、 \
.   |      /::::::::::::::::::|::::::::\/:::O`、::\   | '、   \
   |      /:::::::::::::::::::/:::::::::::::::::::::::::::::'、::::\ノ  ヽ、  |
  |      |:::::/:::::::::/:::::::::::::::::::::::::::::::::::'、',::::'、  /:\__/‐、
  |      |/:::::::::::/::::::::::::::::::::::::::::::::::O::| '、::| く::::::::::::: ̄|
   |     /_..-'´ ̄`ー-、:::::::::::::::::::::::::::::::::::|/:/`‐'::\;;;;;;;_|
   |    |/::::::::::::::::::::::\:::::::::::::::::::::::::::::|::/::::|::::/:::::::::::/
    |   /:::::::::::::::::::::::::::::::::|:::::::::::::::::::::O::|::|::::::|:::::::::::::::/
42132人目の素数さん:2012/04/21(土) 07:50:46.15
          __ノ)-'´ ̄ ̄`ー- 、_
        , '´  _. -‐'''"二ニニ=-`ヽ、
      /   /:::::; -‐''"        `ーノ
     /   /:::::/           \
     /    /::::::/          | | |  |
     |   |:::::/ /     |  | | | |  |
      |   |::/ / / |  | ||  | | ,ハ .| ,ハ|
      |   |/ / / /| ,ハノ| /|ノレ,ニ|ル' 
     |   |  | / / レ',二、レ′ ,ィイ|゙/   私は只の数ヲタなんかとは付き合わないわ。
.     |   \ ∠イ  ,イイ|    ,`-' |      頭が良くて数学が出来てかっこいい人。それが必要条件よ。
     |     l^,人|  ` `-'     ゝ  |        さらに Ann.of Math に論文書けば十分条件にもなるわよ。
      |      ` -'\       ー'  人          一番嫌いなのは論文数を増やすためにくだらない論文を書いて
    |        /(l     __/  ヽ、           良い論文の出版を遅らせるお馬鹿な人。
     |       (:::::`‐-、__  |::::`、     ヒニニヽ、         あなたの論文が Ann of Math に accept される確率は?
    |      / `‐-、::::::::::`‐-、::::\   /,ニニ、\            それとも最近は Inv. Math. の方が上かしら?
   |      |::::::::::::::::::|` -、:::::::,ヘ ̄|'、  ヒニ二、 \
.   |      /::::::::::::::::::|::::::::\/:::O`、::\   | '、   \
   |      /:::::::::::::::::::/:::::::::::::::::::::::::::::'、::::\ノ  ヽ、  |
  |      |:::::/:::::::::/:::::::::::::::::::::::::::::::::::'、',::::'、  /:\__/‐、
  |      |/:::::::::::/::::::::::::::::::::::::::::::::::O::| '、::| く::::::::::::: ̄|
   |     /_..-'´ ̄`ー-、:::::::::::::::::::::::::::::::::::|/:/`‐'::\;;;;;;;_|
   |    |/::::::::::::::::::::::\:::::::::::::::::::::::::::::|::/::::|::::/:::::::::::/
    |   /:::::::::::::::::::::::::::::::::|:::::::::::::::::::::O::|::|::::::|:::::::::::::::/
43132人目の素数さん:2012/04/21(土) 08:14:13.65
(-2)^(1/3)って、正ですか?負ですか?
44132人目の素数さん:2012/04/21(土) 08:18:03.98
まぁ自分の考えとしては、3乗して-2 つまり奇数乗して負なんだから負なんだろうな
と思ってるんですが
45132人目の素数さん:2012/04/21(土) 08:30:21.41
>>40
アホだなバカオツ
46132人目の素数さん:2012/04/21(土) 08:38:19.07
          __ノ)-'´ ̄ ̄`ー- 、_
        , '´  _. -‐'''"二ニニ=-`ヽ、
      /   /:::::; -‐''"        `ーノ
     /   /:::::/           \
     /    /::::::/          | | |  |
     |   |:::::/ /     |  | | | |  |
      |   |::/ / / |  | ||  | | ,ハ .| ,ハ|
      |   |/ / / /| ,ハノ| /|ノレ,ニ|ル' 
     |   |  | / / レ',二、レ′ ,ィイ|゙/   私は只の数ヲタなんかとは付き合わないわ。
.     |   \ ∠イ  ,イイ|    ,`-' |      頭が良くて数学が出来てかっこいい人。それが必要条件よ。
     |     l^,人|  ` `-'     ゝ  |        さらに Ann.of Math に論文書けば十分条件にもなるわよ。
      |      ` -'\       ー'  人          一番嫌いなのは論文数を増やすためにくだらない論文を書いて
    |        /(l     __/  ヽ、           良い論文の出版を遅らせるお馬鹿な人。
     |       (:::::`‐-、__  |::::`、     ヒニニヽ、         あなたの論文が Ann of Math に accept される確率は?
    |      / `‐-、::::::::::`‐-、::::\   /,ニニ、\            それとも最近は Inv. Math. の方が上かしら?
   |      |::::::::::::::::::|` -、:::::::,ヘ ̄|'、  ヒニ二、 \
.   |      /::::::::::::::::::|::::::::\/:::O`、::\   | '、   \
   |      /:::::::::::::::::::/:::::::::::::::::::::::::::::'、::::\ノ  ヽ、  |
  |      |:::::/:::::::::/:::::::::::::::::::::::::::::::::::'、',::::'、  /:\__/‐、
  |      |/:::::::::::/::::::::::::::::::::::::::::::::::O::| '、::| く::::::::::::: ̄|
   |     /_..-'´ ̄`ー-、:::::::::::::::::::::::::::::::::::|/:/`‐'::\;;;;;;;_|
   |    |/::::::::::::::::::::::\:::::::::::::::::::::::::::::|::/::::|::::/:::::::::::/
    |   /:::::::::::::::::::::::::::::::::|:::::::::::::::::::::O::|::|::::::|:::::::::::::::/
47132人目の素数さん:2012/04/21(土) 09:29:17.85
>>40
なんとなく掴めてきました!
丁寧にありがとうございました
48132人目の素数さん:2012/04/21(土) 12:24:19.74
>>43
負で合ってる
49132人目の素数さん:2012/04/21(土) 13:06:41.65
横から質問お願いします
>>35の質問をちょっとやってみたんですが、途中で躓きました

方法1
「n回引いて3回当たってる確率」=(1/5)^3*(4/5)^(n-3)*C[n,3]
P[n]=「n回目で3回当たる確率」=「n回引いて3回当たっている確率」-「n-1回引いて3回当たっている確率」

=(1/5)^3*(4/5)^(n-3)*C[n,3]-(1/5)^3*(4/5)^(n-4)*C[n-1,3]

方法2
P[n]=「n回目で3回当たる確率」=「n-1回目までに2回当たっていて、n回目で当たる確率」=「n-1回引いて2回当たっている確率」×「当たる確率」

={(1/5)^2*(4/5)^(n-4)*C[n-1,2]}*1/5

確認の為に計算
n=4の時、
方法1
P[4}=(1/5)^3*(4/5)*4-(1/5)^3*1*1=(1/5)^3*11/5
方法2
P[4]=(1/5)^2*1*3*(1/5)=(1/5)^3*3

同じにならないいい
そして両方とも間違ってるっぽい数字
どこが間違っているのでしょうか?
50132人目の素数さん:2012/04/21(土) 14:10:40.90
>43
(-1)^(1/3)*2^(1/3)

>49
方法2
n-1回引き2回当たる確率=(1/5)^2*(4/5)^(n-3)*C[n-1,2]
n=4;(1/5)^2*(4/5)*C[3,2]=12/5^3

方法1
(n回引き3回当たる確率)-(n-1回引き3回当たる確率)*(n回目で外れる確率)
5149:2012/04/21(土) 14:10:46.16
ごめん
計算ミスでした
質問を取り下げます
5249:2012/04/21(土) 14:11:23.92
おお丁度カブッた

>>50
ありがとう
53ようじょ ◆hNziS2E8421X :2012/04/21(土) 14:37:28.89
久しぶりです!こんにちは!

ベータ関数B(p,q)とガンマ関数Γ(p)の間に
B(p,q)=Γ(p)Γ(q)/Γ(p+q)なる等式が成り立つことを示せ

がわかりません・・・おしえてください!
54ようじょ ◆hNziS2E8421X :2012/04/21(土) 14:38:31.91
ごめんなさいまちがえました・・・
55132人目の素数さん:2012/04/21(土) 15:04:57.34
          __ノ)-'´ ̄ ̄`ー- 、_
        , '´  _. -‐'''"二ニニ=-`ヽ、
      /   /:::::; -‐''"        `ーノ
     /   /:::::/           \
     /    /::::::/          | | |  |
     |   |:::::/ /     |  | | | |  |
      |   |::/ / / |  | ||  | | ,ハ .| ,ハ|
      |   |/ / / /| ,ハノ| /|ノレ,ニ|ル' 
     |   |  | / / レ',二、レ′ ,ィイ|゙/   私は只の数ヲタなんかとは付き合わないわ。
.     |   \ ∠イ  ,イイ|    ,`-' |      頭が良くて数学が出来てかっこいい人。それが必要条件よ。
     |     l^,人|  ` `-'     ゝ  |        さらに Ann.of Math に論文書けば十分条件にもなるわよ。
      |      ` -'\       ー'  人          一番嫌いなのは論文数を増やすためにくだらない論文を書いて
    |        /(l     __/  ヽ、           良い論文の出版を遅らせるお馬鹿な人。
     |       (:::::`‐-、__  |::::`、     ヒニニヽ、         あなたの論文が Ann of Math に accept される確率は?
    |      / `‐-、::::::::::`‐-、::::\   /,ニニ、\            それとも最近は Inv. Math. の方が上かしら?
   |      |::::::::::::::::::|` -、:::::::,ヘ ̄|'、  ヒニ二、 \
.   |      /::::::::::::::::::|::::::::\/:::O`、::\   | '、   \
   |      /:::::::::::::::::::/:::::::::::::::::::::::::::::'、::::\ノ  ヽ、  |
  |      |:::::/:::::::::/:::::::::::::::::::::::::::::::::::'、',::::'、  /:\__/‐、
  |      |/:::::::::::/::::::::::::::::::::::::::::::::::O::| '、::| く::::::::::::: ̄|
   |     /_..-'´ ̄`ー-、:::::::::::::::::::::::::::::::::::|/:/`‐'::\;;;;;;;_|
   |    |/::::::::::::::::::::::\:::::::::::::::::::::::::::::|::/::::|::::/:::::::::::/
    |   /:::::::::::::::::::::::::::::::::|:::::::::::::::::::::O::|::|::::::|:::::::::::::::/
56132人目の素数さん:2012/04/21(土) 15:05:57.89
>>45
アホ臭いバカオツ
57132人目の素数さん:2012/04/21(土) 15:59:00.57
「国の借金」について

日頃メディアや、反日工作員が必死になって「国の借金」という単語を使い
財政破綻論を展開させていますが、現実、現在の日本には「政府の借金」1000兆円近く存在いたしますが、
「国の借金」は存在いたしません。

朝日新聞やNHKは、雇い主である中国共産党より日本人に対して不安や政府に対する不信を持たせ、煽るために
局内の共産党員を使用して既に数十年間、「国の借金」を連呼し続けております。

<違和感なく「国の借金が1000兆円もある」という幻想に浸ってしまっている一般の方々は、朝日新聞やNHKに見事に騙されて続けている訳です>

数十年もテレビや新聞から情報を得てきた方々の中には、「メディアが嘘を付く訳ない」と思う、そう思いたい方もいるでしょう。
  しかし、長い目で見れば、もともと戦前から日本を転覆させるために存在してきた報道機関ですから、
これくらいの嘘は朝飯前で御座います。

それでも、「国の借金は1000兆円ある」と考えをお持ちの方は、複式簿記の勉強をしてから、日本のバランスシートをご覧ください。
 
  実質中国の広報機関であるNHK、朝日新聞は、これからも嘘を付き続けます。デマを流し続けます。捏造し続けます。
   
 ---そのニュース 核心はヤラセだ       長文失礼いたしました。---

58132人目の素数さん:2012/04/21(土) 17:55:35.42
          __ノ)-'´ ̄ ̄`ー- 、_
        , '´  _. -‐'''"二ニニ=-`ヽ、
      /   /:::::; -‐''"        `ーノ
     /   /:::::/           \
     /    /::::::/          | | |  |
     |   |:::::/ /     |  | | | |  |
      |   |::/ / / |  | ||  | | ,ハ .| ,ハ|
      |   |/ / / /| ,ハノ| /|ノレ,ニ|ル' 
     |   |  | / / レ',二、レ′ ,ィイ|゙/   私は只の数ヲタなんかとは付き合わないわ。
.     |   \ ∠イ  ,イイ|    ,`-' |      頭が良くて数学が出来てかっこいい人。それが必要条件よ。
     |     l^,人|  ` `-'     ゝ  |        さらに Ann.of Math に論文書けば十分条件にもなるわよ。
      |      ` -'\       ー'  人          一番嫌いなのは論文数を増やすためにくだらない論文を書いて
    |        /(l     __/  ヽ、           良い論文の出版を遅らせるお馬鹿な人。
     |       (:::::`‐-、__  |::::`、     ヒニニヽ、         あなたの論文が Ann of Math に accept される確率は?
    |      / `‐-、::::::::::`‐-、::::\   /,ニニ、\            それとも最近は Inv. Math. の方が上かしら?
   |      |::::::::::::::::::|` -、:::::::,ヘ ̄|'、  ヒニ二、 \
.   |      /::::::::::::::::::|::::::::\/:::O`、::\   | '、   \
   |      /:::::::::::::::::::/:::::::::::::::::::::::::::::'、::::\ノ  ヽ、  |
  |      |:::::/:::::::::/:::::::::::::::::::::::::::::::::::'、',::::'、  /:\__/‐、
  |      |/:::::::::::/::::::::::::::::::::::::::::::::::O::| '、::| く::::::::::::: ̄|
   |     /_..-'´ ̄`ー-、:::::::::::::::::::::::::::::::::::|/:/`‐'::\;;;;;;;_|
   |    |/::::::::::::::::::::::\:::::::::::::::::::::::::::::|::/::::|::::/:::::::::::/
    |   /:::::::::::::::::::::::::::::::::|:::::::::::::::::::::O::|::|::::::|:::::::::::::::/
59132人目の素数さん:2012/04/21(土) 18:00:36.52
          __ノ)-'´ ̄ ̄`ー- 、_
        , '´  _. -‐'''"二ニニ=-`ヽ、
      /   /:::::; -‐''"        `ーノ
     /   /:::::/           \
     /    /::::::/          | | |  |
     |   |:::::/ /     |  | | | |  |
      |   |::/ / / |  | ||  | | ,ハ .| ,ハ|
      |   |/ / / /| ,ハノ| /|ノレ,ニ|ル' 
     |   |  | / / レ',二、レ′ ,ィイ|゙/   私は只の数ヲタなんかとは付き合わないわ。
.     |   \ ∠イ  ,イイ|    ,`-' |      頭が良くて数学が出来てかっこいい人。それが必要条件よ。
     |     l^,人|  ` `-'     ゝ  |        さらに Ann.of Math に論文書けば十分条件にもなるわよ。
      |      ` -'\       ー'  人          一番嫌いなのは論文数を増やすためにくだらない論文を書いて
    |        /(l     __/  ヽ、           良い論文の出版を遅らせるお馬鹿な人。
     |       (:::::`‐-、__  |::::`、     ヒニニヽ、         あなたの論文が Ann of Math に accept される確率は?
    |      / `‐-、::::::::::`‐-、::::\   /,ニニ、\            それとも最近は Inv. Math. の方が上かしら?
   |      |::::::::::::::::::|` -、:::::::,ヘ ̄|'、  ヒニ二、 \
.   |      /::::::::::::::::::|::::::::\/:::O`、::\   | '、   \
   |      /:::::::::::::::::::/:::::::::::::::::::::::::::::'、::::\ノ  ヽ、  |
  |      |:::::/:::::::::/:::::::::::::::::::::::::::::::::::'、',::::'、  /:\__/‐、
  |      |/:::::::::::/::::::::::::::::::::::::::::::::::O::| '、::| く::::::::::::: ̄|
   |     /_..-'´ ̄`ー-、:::::::::::::::::::::::::::::::::::|/:/`‐'::\;;;;;;;_|
   |    |/::::::::::::::::::::::\:::::::::::::::::::::::::::::|::/::::|::::/:::::::::::/
    |   /:::::::::::::::::::::::::::::::::|:::::::::::::::::::::O::|::|::::::|:::::::::::::::/
60132人目の素数さん:2012/04/21(土) 20:25:31.03
まだまだキチガイがいる
バカオツ
61132人目の素数さん:2012/04/21(土) 20:33:15.93
>>60
何してんの?恥ずかしくないの?
バカオツ
62132人目の素数さん:2012/04/21(土) 20:49:45.41
フィボナッチ数列の4番目ごとに
3の倍数が出てくるのはなんで?
63132人目の素数さん:2012/04/21(土) 20:52:29.68
フィボナッチ数列の一般項を考える。
64132人目の素数さん:2012/04/21(土) 21:11:13.59
          __ノ)-'´ ̄ ̄`ー- 、_
        , '´  _. -‐'''"二ニニ=-`ヽ、
      /   /:::::; -‐''"        `ーノ
     /   /:::::/           \
     /    /::::::/          | | |  |
     |   |:::::/ /     |  | | | |  |
      |   |::/ / / |  | ||  | | ,ハ .| ,ハ|
      |   |/ / / /| ,ハノ| /|ノレ,ニ|ル' 
     |   |  | / / レ',二、レ′ ,ィイ|゙/   私は只の数ヲタなんかとは付き合わないわ。
.     |   \ ∠イ  ,イイ|    ,`-' |      頭が良くて数学が出来てかっこいい人。それが必要条件よ。
     |     l^,人|  ` `-'     ゝ  |        さらに Ann.of Math に論文書けば十分条件にもなるわよ。
      |      ` -'\       ー'  人          一番嫌いなのは論文数を増やすためにくだらない論文を書いて
    |        /(l     __/  ヽ、           良い論文の出版を遅らせるお馬鹿な人。
     |       (:::::`‐-、__  |::::`、     ヒニニヽ、         あなたの論文が Ann of Math に accept される確率は?
    |      / `‐-、::::::::::`‐-、::::\   /,ニニ、\            それとも最近は Inv. Math. の方が上かしら?
   |      |::::::::::::::::::|` -、:::::::,ヘ ̄|'、  ヒニ二、 \
.   |      /::::::::::::::::::|::::::::\/:::O`、::\   | '、   \
   |      /:::::::::::::::::::/:::::::::::::::::::::::::::::'、::::\ノ  ヽ、  |
  |      |:::::/:::::::::/:::::::::::::::::::::::::::::::::::'、',::::'、  /:\__/‐、
  |      |/:::::::::::/::::::::::::::::::::::::::::::::::O::| '、::| く::::::::::::: ̄|
   |     /_..-'´ ̄`ー-、:::::::::::::::::::::::::::::::::::|/:/`‐'::\;;;;;;;_|
   |    |/::::::::::::::::::::::\:::::::::::::::::::::::::::::|::/::::|::::/:::::::::::/
    |   /:::::::::::::::::::::::::::::::::|:::::::::::::::::::::O::|::|::::::|:::::::::::::::/
65132人目の素数さん:2012/04/21(土) 23:07:43.36
すみません、螺旋の方程式を知りたいのですが。
サイン曲線の頭とお尻を閉じて、球を作りたいのですが
わかりますでしょうか?
66132人目の素数さん:2012/04/21(土) 23:15:59.74
凾`BCの内部に点Pが存在し
αAP↑ + βBP + γCP = 0

このとき凾oBC:凾oAC:凾oABを求めよ
67132人目の素数さん:2012/04/21(土) 23:16:35.09
あ、二行目ミス
αAP↑ + βBP↑ + γCP↑ = 0↑
68132人目の素数さん:2012/04/21(土) 23:34:32.67
>>62
a[n+4]=a[n+3]+a[n+2]=2a[n+2]+a[n+1]=3a[n+1]+2a[n]
よってa[n]が3で割れる⇒a[n+4]が3で割れる
さらにa[4]=3
69132人目の素数さん:2012/04/21(土) 23:42:14.09
>>65
何が何だか
ランベルト正積円筒図法みたいなもので移すと
y=sin xのグラフが現れるような
球面上の曲線を知りたい、でいいのか?
70132人目の素数さん:2012/04/21(土) 23:51:12.07
>>69 すみません、こんな感じで円の中がサイン波で。
http://up.mugitya.com/img/Lv.1_up36832.jpg
横から見ると↓です。
http://up.mugitya.com/img/Lv.1_up36833.jpg
これのもっと隙間が狭い感じ。
71132人目の素数さん:2012/04/21(土) 23:54:40.84
>>70
ttp://ja.wikipedia.org/wiki/%E5%81%8F%E5%85%89
まずは筒状の正弦波からでどうでしょう
72132人目の素数さん:2012/04/21(土) 23:57:20.22
          __ノ)-'´ ̄ ̄`ー- 、_
        , '´  _. -‐'''"二ニニ=-`ヽ、
      /   /:::::; -‐''"        `ーノ
     /   /:::::/           \
     /    /::::::/          | | |  |
     |   |:::::/ /     |  | | | |  |
      |   |::/ / / |  | ||  | | ,ハ .| ,ハ|
      |   |/ / / /| ,ハノ| /|ノレ,ニ|ル' 
     |   |  | / / レ',二、レ′ ,ィイ|゙/   私は只の数ヲタなんかとは付き合わないわ。
.     |   \ ∠イ  ,イイ|    ,`-' |      頭が良くて数学が出来てかっこいい人。それが必要条件よ。
     |     l^,人|  ` `-'     ゝ  |        さらに Ann.of Math に論文書けば十分条件にもなるわよ。
      |      ` -'\       ー'  人          一番嫌いなのは論文数を増やすためにくだらない論文を書いて
    |        /(l     __/  ヽ、           良い論文の出版を遅らせるお馬鹿な人。
     |       (:::::`‐-、__  |::::`、     ヒニニヽ、         あなたの論文が Ann of Math に accept される確率は?
    |      / `‐-、::::::::::`‐-、::::\   /,ニニ、\            それとも最近は Inv. Math. の方が上かしら?
   |      |::::::::::::::::::|` -、:::::::,ヘ ̄|'、  ヒニ二、 \
.   |      /::::::::::::::::::|::::::::\/:::O`、::\   | '、   \
   |      /:::::::::::::::::::/:::::::::::::::::::::::::::::'、::::\ノ  ヽ、  |
  |      |:::::/:::::::::/:::::::::::::::::::::::::::::::::::'、',::::'、  /:\__/‐、
  |      |/:::::::::::/::::::::::::::::::::::::::::::::::O::| '、::| く::::::::::::: ̄|
   |     /_..-'´ ̄`ー-、:::::::::::::::::::::::::::::::::::|/:/`‐'::\;;;;;;;_|
   |    |/::::::::::::::::::::::\:::::::::::::::::::::::::::::|::/::::|::::/:::::::::::/
    |   /:::::::::::::::::::::::::::::::::|:::::::::::::::::::::O::|::|::::::|:::::::::::::::/
73132人目の素数さん:2012/04/21(土) 23:57:49.65
波形はわかりましたでしょうか?
ちょっと図が手抜きしたもので。
74132人目の素数さん:2012/04/21(土) 23:58:17.34
IDでないのか
75132人目の素数さん:2012/04/22(日) 00:01:28.94
          __ノ)-'´ ̄ ̄`ー- 、_
        , '´  _. -‐'''"二ニニ=-`ヽ、
      /   /:::::; -‐''"        `ーノ
     /   /:::::/           \
     /    /::::::/          | | |  |
     |   |:::::/ /     |  | | | |  |
      |   |::/ / / |  | ||  | | ,ハ .| ,ハ|
      |   |/ / / /| ,ハノ| /|ノレ,ニ|ル' 
     |   |  | / / レ',二、レ′ ,ィイ|゙/   私は只の数ヲタなんかとは付き合わないわ。
.     |   \ ∠イ  ,イイ|    ,`-' |      頭が良くて数学が出来てかっこいい人。それが必要条件よ。
     |     l^,人|  ` `-'     ゝ  |        さらに Ann.of Math に論文書けば十分条件にもなるわよ。
      |      ` -'\       ー'  人          一番嫌いなのは論文数を増やすためにくだらない論文を書いて
    |        /(l     __/  ヽ、           良い論文の出版を遅らせるお馬鹿な人。
     |       (:::::`‐-、__  |::::`、     ヒニニヽ、         あなたの論文が Ann of Math に accept される確率は?
    |      / `‐-、::::::::::`‐-、::::\   /,ニニ、\            それとも最近は Inv. Math. の方が上かしら?
   |      |::::::::::::::::::|` -、:::::::,ヘ ̄|'、  ヒニ二、 \
.   |      /::::::::::::::::::|::::::::\/:::O`、::\   | '、   \
   |      /:::::::::::::::::::/:::::::::::::::::::::::::::::'、::::\ノ  ヽ、  |
  |      |:::::/:::::::::/:::::::::::::::::::::::::::::::::::'、',::::'、  /:\__/‐、
  |      |/:::::::::::/::::::::::::::::::::::::::::::::::O::| '、::| く::::::::::::: ̄|
   |     /_..-'´ ̄`ー-、:::::::::::::::::::::::::::::::::::|/:/`‐'::\;;;;;;;_|
   |    |/::::::::::::::::::::::\:::::::::::::::::::::::::::::|::/::::|::::/:::::::::::/
    |   /:::::::::::::::::::::::::::::::::|:::::::::::::::::::::O::|::|::::::|:::::::::::::::/
76132人目の素数さん:2012/04/22(日) 00:09:24.55
>>70
(x,y,z)=((sqrt t)*sin t,(sqrt t)*cos t, (sqrt t)sin 10*t)from 0 to 100
とかそういう感じか?パラメータいい加減だけど

どこかこれ表示するサービスみたいなのねえかなあ
77132人目の素数さん:2012/04/22(日) 00:10:14.21
78132人目の素数さん:2012/04/22(日) 00:27:41.14
>>76 俺もそれだとわからない。
>>77 おおこれこれ、ありがとうございました。

これ、電子の軌道なんですよね。(俺的に)
79132人目の素数さん:2012/04/22(日) 00:35:19.36
>>66-67
与式を
  「始点を O にしたベクトルで書き換えて整理する(分点公式の形を無理やり作る)」
ことにより
   p = ( αa + βb + γc )/( α+β+γ ) …☆
    =( αa + ( γ+β )(( βb + γc )/( γ+β )))/(( γ+β )+α)
などとなる(位置ベクトルを単に小文字だけで表した)
この式から,辺 BC を γ:βに内分する点を D とすると
   点 P は 線分 AD を (γ+β):α に内分する点
とわかる よって,△ABC と △PBC の高さの比がわかる
他も同様にして,
   △PBC : △PCA : △PAB = α:β:γ

物理的には,点 P は
   3点 A , B , C にそれぞれ α,β,γ の加重があるときの(加重)重心
を表す( P に糸をつけてぶら下げれば,△ABC は水平になる)
高校の物理の教科書にも☆に相当する式が出ている
80132人目の素数さん:2012/04/22(日) 00:45:23.85
ああそういう螺旋なのか…
ParametricPlot3D[{(sqrt t)*sin t,(sqrt t)*cos t, (sqrt (100-t))sin 7.41t)},{t,0,100}]
こんなやつかと思った
81132人目の素数さん:2012/04/22(日) 06:07:39.68
          __ノ)-'´ ̄ ̄`ー- 、_
        , '´  _. -‐'''"二ニニ=-`ヽ、
      /   /:::::; -‐''"        `ーノ
     /   /:::::/           \
     /    /::::::/          | | |  |
     |   |:::::/ /     |  | | | |  |
      |   |::/ / / |  | ||  | | ,ハ .| ,ハ|
      |   |/ / / /| ,ハノ| /|ノレ,ニ|ル' 
     |   |  | / / レ',二、レ′ ,ィイ|゙/   私は只の数ヲタなんかとは付き合わないわ。
.     |   \ ∠イ  ,イイ|    ,`-' |      頭が良くて数学が出来てかっこいい人。それが必要条件よ。
     |     l^,人|  ` `-'     ゝ  |        さらに Ann.of Math に論文書けば十分条件にもなるわよ。
      |      ` -'\       ー'  人          一番嫌いなのは論文数を増やすためにくだらない論文を書いて
    |        /(l     __/  ヽ、           良い論文の出版を遅らせるお馬鹿な人。
     |       (:::::`‐-、__  |::::`、     ヒニニヽ、         あなたの論文が Ann of Math に accept される確率は?
    |      / `‐-、::::::::::`‐-、::::\   /,ニニ、\            それとも最近は Inv. Math. の方が上かしら?
   |      |::::::::::::::::::|` -、:::::::,ヘ ̄|'、  ヒニ二、 \
.   |      /::::::::::::::::::|::::::::\/:::O`、::\   | '、   \
   |      /:::::::::::::::::::/:::::::::::::::::::::::::::::'、::::\ノ  ヽ、  |
  |      |:::::/:::::::::/:::::::::::::::::::::::::::::::::::'、',::::'、  /:\__/‐、
  |      |/:::::::::::/::::::::::::::::::::::::::::::::::O::| '、::| く::::::::::::: ̄|
   |     /_..-'´ ̄`ー-、:::::::::::::::::::::::::::::::::::|/:/`‐'::\;;;;;;;_|
   |    |/::::::::::::::::::::::\:::::::::::::::::::::::::::::|::/::::|::::/:::::::::::/
    |   /:::::::::::::::::::::::::::::::::|:::::::::::::::::::::O::|::|::::::|:::::::::::::::/
82132人目の素数さん:2012/04/22(日) 12:34:15.83
半径rの球面上に異なる4点A,B,C,Dがある。AB=CD=√2,AC=AD=BC=BD=√5であるとき、半径rを求めよ。

図のようになって、△ANOと△DMOが合同になるようなんですが、どの辺とどの角を使って合同であると言ってるのかがわかりません。
それらが合同だからNO=MOになると解答にあるので、∠NAO=∠MDOであることが示せればいいのだと思うのですが、わかりません。

http://i.imgur.com/dBSZA.jpg
83132人目の素数さん:2012/04/22(日) 12:45:23.10
84132人目の素数さん:2012/04/22(日) 12:51:08.38
【前置き】
unicodeでベクトルの記号が無かった為、ベクトル量には上付きの矢印の代わりに
上付きのˢをつけています^ω^;
【本題】
平面上にO₁, O₂, A, B の4点があり

線分O₁A上に任意の点M₁、線分O₂B上に任意の点M₂を
それぞれ O₁M₁:M₁A = m₁:1− m₁ O₂M₂:M₂B = m₂:1−m₂ となるように置く

AM₂とM₁Bの交点をPとして、O₁Aˢ = aˢ, O₂Bˢ = bˢ, O₁Pˢ = pˢ, O₁O₂ˢ = O₁₂ˢとする時
(1)APˢを求めよ

http://i.imgur.com/nSv1R.jpg

という問題なのですが、O₁O₂ˢ= 0ˢ の時は解決しています.....

長くなりましたがオネッシャイシマス!
85132人目の素数さん:2012/04/22(日) 12:55:35.50
【訂正】スイマセン....
× APˢを求めよ

◯ O₁Pˢ = pˢ を求めよ
86132人目の素数さん:2012/04/22(日) 13:03:52.68
>>82です。

>>83ありがとうございます。
そちらの解答の方がスマートですね。

この問題は予備校の授業で扱いました。
ノートをそのまま写しますと、


OはAB,CDの垂直二等分線上にあるから、
△ANO,△DMOは直角三角形である
さらに「△ANO≡△DMOだから」←ここが示せない。
NO=MOつまりOはNMの中点である
△ANMに三平方の定理から、
NM=2
∴NO=MO=1
……
87132人目の素数さん:2012/04/22(日) 13:07:22.41
>>84
>>1を読め
88132人目の素数さん:2012/04/22(日) 13:09:56.58
>>86
直角三角形の合同条件
89132人目の素数さん:2012/04/22(日) 13:19:25.11
>>88ありがとう
結構恥ずかしかった
90132人目の素数さん:2012/04/22(日) 13:24:13.01
文字コードはいつの時代もクソだけど
そのゴミカスみたいな謎の「・」(なか点?)は
UNICODEだと↑に見えるんだろな たぶん
ついでに2ch自体はSHIFT-JIS
専ブラそれぞれの文字コードは知らん
見えるものが常に書けるとは限らない
91132人目の素数さん:2012/04/22(日) 13:27:47.06
>>87
分かってる上で書きました。
いい加減unicodeを使わない•使えない•応用しようとしない人がいる状況をどうにかしてほしいと思います。
失礼な言い方ですが...。

常識ですが
上付き⁰¹²³⁴⁵⁶⁷⁸⁹⁺⁻⁼⁽⁾ ⁿʰʲʳʷʸªˠˡˢˣⁿ
下付き₀₁₂₃₄₅₆₇₈₉₊₋₌₍₎
その他㏒ ㎈㎉㎍ r s ㎐ ㎖ ㎛ o p q
㎟ ㎠ u ㎢
㎣ ㎤ ㎥ ㎦ ㏗ですね。
92すいませんでした...:2012/04/22(日) 13:28:14.83
【本題】【訂正】.......。本当スイマセン....。
平面上にO₁, O₂, A, B の4点があり

線分O₁A上に任意の点M₁、線分O₂B上に任意の点M₂を
それぞれ O₁M₁:M₁A = m₁:1− m₁ O₂M₂:M₂B = m₂:1−m₂ となるように置く

AM₂とM₁Bの交点をPとして、O₁A↑= a↑, O₂B↑ = b↑, O₁P↑ = p↑, O₁O₂↑ = O₁₂↑とする時
(1)AP↑を求めよ
おねがいします...
93すいませんでした...:2012/04/22(日) 13:29:02.68
そしてまたミスる。

求めたいのはp↑でした...
94132人目の素数さん:2012/04/22(日) 13:41:45.13
お前のゴタクはいいから
さっさと>>1に従って書きなおせよゴミ
95132人目の素数さん:2012/04/22(日) 13:48:31.27
>>91
どうにかして欲しいのはわかったから、どうにかなってからやれ
96すいませんでした...:2012/04/22(日) 13:51:30.66
iPadしかないのでunicodeの件は回避できそうにないです
なのでスクショうpします...


http://i.imgur.com/pRwmm.jpg

http://i.imgur.com/nSv1R.jpg

多分これでできたはず....
97すいませんでした...:2012/04/22(日) 13:54:46.86
確認せずにうpしてさっきのを見たら2行目ぐらいに間違いが
なので更に訂正....迷惑かけすぎでスマソです....

http://i.imgur.com/a9Dp5.jpg
98132人目の素数さん:2012/04/22(日) 14:01:29.75
まずさ、ベクトルを表記する時さ
二点を使って表す場合は両方大文字、一文字でおくときは小文字っていう
作法があるんだけど、最後から二番目のベクトルはゼロベクトルでなくてoのベクトルか?
99132人目の素数さん:2012/04/22(日) 14:05:13.33
あと、オナシャスとか煽りみたいな言葉入れるぐらいなら無い方がましだから。

>・質問者は何が分からないのか、どこまで考えたのかを明記しましょう。それがない場合、放置されることがあります。
>  (特に、自分でやってみたのにあわないので教えてほしい、みたいなときは必ず書くように)

書いてあって、ここまでやりましたがわかりません。ってある方が遥かに印象いいよね。
100すいませんでした...:2012/04/22(日) 14:06:00.77
101すいませんでした...:2012/04/22(日) 14:11:05.59
>>99

恒等式たてる考え方でやりましたが

http://i.imgur.com/LcERF.jpg
とはなりましたが正方形で実測したら違う値が出たのでここでの質問に至りました。(この画像でのm1(エムワン)は訂正版でのnのことm2は訂正版でのmのことです)
102132人目の素数さん:2012/04/22(日) 14:19:59.94
>>91
unicodeどうこうではなく、殆どのフォントで文字が
他に比べて細かすぎるから俺個人は嫌いだな
⁰¹²³⁴⁵⁶⁷⁸⁹⁺⁻⁼⁽⁾ ⁿʰʲʳʷʸªˠˡˢˣⁿ
₀₁₂₃₄₅₆₇₈₉₊₋₌₍₎
あたり非常に読みにくい。で、これらにあわせて文字を拡大すると他がでかすぎる
103132人目の素数さん:2012/04/22(日) 14:31:29.03
>>101
他人に見せる様な解答なのに、そんな適当な解答書いてるから出来るようにならないんだよ。
係数比較するときに必須の文言があっただろ。
なんであの文言が必須なのか私は一度も考えた事ない馬鹿です。って答案で自白してる様なもん。
104すいませんでした...:2012/04/22(日) 14:38:01.01
?

具体的にどうすればいいのですか....?!
105132人目の素数さん:2012/04/22(日) 14:40:31.82
>>104
半年ROMれ
とっとと立ち去れ
テンプレにも従わずに人の話もきかねーモン
それなら回答しても聞くとは思えない
106132人目の素数さん:2012/04/22(日) 14:46:06.66
高圧的な教授者の割に解けなかったというオチ(^◇^;)

どっちがバカだよ

ろくに文字式で解いた事ないんだろうな...かわいそうに...
107132人目の素数さん:2012/04/22(日) 15:01:51.45
2次関数y=x^2−2x+1が最大値をもたないことを背理法を用いて証明せよ
108132人目の素数さん:2012/04/22(日) 15:05:05.87
w
109132人目の素数さん:2012/04/22(日) 15:05:32.79
          __ノ)-'´ ̄ ̄`ー- 、_
        , '´  _. -‐'''"二ニニ=-`ヽ、
      /   /:::::; -‐''"        `ーノ
     /   /:::::/           \
     /    /::::::/          | | |  |
     |   |:::::/ /     |  | | | |  |
      |   |::/ / / |  | ||  | | ,ハ .| ,ハ|
      |   |/ / / /| ,ハノ| /|ノレ,ニ|ル' 
     |   |  | / / レ',二、レ′ ,ィイ|゙/   私は只の数ヲタなんかとは付き合わないわ。
.     |   \ ∠イ  ,イイ|    ,`-' |      頭が良くて数学が出来てかっこいい人。それが必要条件よ。
     |     l^,人|  ` `-'     ゝ  |        さらに Ann.of Math に論文書けば十分条件にもなるわよ。
      |      ` -'\       ー'  人          一番嫌いなのは論文数を増やすためにくだらない論文を書いて
    |        /(l     __/  ヽ、           良い論文の出版を遅らせるお馬鹿な人。
     |       (:::::`‐-、__  |::::`、     ヒニニヽ、         あなたの論文が Ann of Math に accept される確率は?
    |      / `‐-、::::::::::`‐-、::::\   /,ニニ、\            それとも最近は Inv. Math. の方が上かしら?
   |      |::::::::::::::::::|` -、:::::::,ヘ ̄|'、  ヒニ二、 \
.   |      /::::::::::::::::::|::::::::\/:::O`、::\   | '、   \
   |      /:::::::::::::::::::/:::::::::::::::::::::::::::::'、::::\ノ  ヽ、  |
  |      |:::::/:::::::::/:::::::::::::::::::::::::::::::::::'、',::::'、  /:\__/‐、
  |      |/:::::::::::/::::::::::::::::::::::::::::::::::O::| '、::| く::::::::::::: ̄|
   |     /_..-'´ ̄`ー-、:::::::::::::::::::::::::::::::::::|/:/`‐'::\;;;;;;;_|
   |    |/::::::::::::::::::::::\:::::::::::::::::::::::::::::|::/::::|::::/:::::::::::/
    |   /:::::::::::::::::::::::::::::::::|:::::::::::::::::::::O::|::|::::::|:::::::::::::::/
110132人目の素数さん:2012/04/22(日) 15:23:15.62
ベクトルの問題なんですが、これ
http://uploader.sakura.ne.jp/src/up95348.jpg
の解き方が分かるかた教えてください。
111132人目の素数さん:2012/04/22(日) 15:31:17.62
俺105じゃねぇし。
まぁどうでもいい事だけど、なんで単純に係数比較出来ないか解ったん?
112132人目の素数さん:2012/04/22(日) 15:52:20.33
定理から公式を導くという表現をよく見かけます。すると例えばピタゴラスの定理「直角三角形の直角を挟んだ二辺の二乗の和は斜辺の二乗に等しい」
よりc^2=b^2+a^2を導けます。では内分点を導ける定理というのは存在しますか?ここら辺で公式や等式といった用語の使い様がよくわかりません。

113132人目の素数さん:2012/04/22(日) 15:56:30.95
内分点の公式を導ける定理、に訂正、
114132人目の素数さん:2012/04/22(日) 16:08:20.47
「内分点の公式を導ける定理」
どんな定理かわからん・・・
115132人目の素数さん:2012/04/22(日) 18:06:09.68
微分積分の基本公式について質問です。参考書では

[In{f(x)}]'=f'(x)/f(x)   ※f(x)>0
∫{f'(x)/f(x)}dx=In{|f(x)|}+C   ※f(x)≠0

と記載されています。積分の証明は右辺を微分するだけとあり、
[In{|f(x)|}]'=f'(x)/f(x) となりますが、なぜ絶対値が外れるのでしょうか?

また、f(x)の範囲は
[In{|f(x)|}]'=f'(x)/f(x)   ※f(x)≠0
としてよいのでしょうか?
116132人目の素数さん:2012/04/22(日) 18:42:45.40
√3*a/(√3+a)=ー6+3bのとき

aとbをそれぞれ求めよという問題
(√3+a)を両辺に掛けて右辺を展開するまでは分かるんですがそれからペンが全く進みません。
何方かヒントでもいいので教えて欲しいです。
117132人目の素数さん:2012/04/22(日) 18:45:35.79
>>115
まず In ではなく ln で自然対数を意味する

で、微分積分の基本公式は関係ない
ln(|x|) の微分や 1/x の積分、合成関数の微分等の問題で、
これらは理解してるのか?


f(x)>0の場合
(ln(|f(x)|))'=(ln(f(x)))'=1/f(x)*f'(x)
f(x)<0の場合
(ln(|f(x)|))'=(ln(-f(x)))'=1/(-f(x))*(-f(x))'=1/f(x)*f'(x)
よって
(ln(|f(x)|))'=f'(x)/f(x) (f(x)≠0)
118132人目の素数さん:2012/04/22(日) 18:57:39.61
>>114
定理の中に公式が存在するのでしょうか?
119132人目の素数さん:2012/04/22(日) 18:58:39.06
>>116
問題の式以外に、a,bについて条件は与えられてないの?
120132人目の素数さん:2012/04/22(日) 19:06:21.19
>>119
a.bは共に有理数である。があります

重要ではないと思ったので省いてました。
121132人目の素数さん:2012/04/22(日) 19:11:37.57
>>120
√3でくくってみろ
122132人目の素数さん:2012/04/22(日) 19:12:29.97
√3*a/(√3+a)は有理数よりa=0
よってb=2
でいいんじゃない?
123132人目の素数さん:2012/04/22(日) 19:14:00.62
>>120
重要すぎる

>   (変に省略するより全文書いた方がいい、また説明なく習慣的でない記号を使わないように)
124132人目の素数さん:2012/04/22(日) 19:15:11.97
それ以前に
「a.bは共に有理数である。」
がこの問題を解く上でどうしても必要である事を
理解する方が先決だと思う。
125132人目の素数さん:2012/04/22(日) 19:15:49.62
          __ノ)-'´ ̄ ̄`ー- 、_
        , '´  _. -‐'''"二ニニ=-`ヽ、
      /   /:::::; -‐''"        `ーノ
     /   /:::::/           \
     /    /::::::/          | | |  |
     |   |:::::/ /     |  | | | |  |
      |   |::/ / / |  | ||  | | ,ハ .| ,ハ|
      |   |/ / / /| ,ハノ| /|ノレ,ニ|ル' 
     |   |  | / / レ',二、レ′ ,ィイ|゙/   私は只の数ヲタなんかとは付き合わないわ。
.     |   \ ∠イ  ,イイ|    ,`-' |      頭が良くて数学が出来てかっこいい人。それが必要条件よ。
     |     l^,人|  ` `-'     ゝ  |        さらに Ann.of Math に論文書けば十分条件にもなるわよ。
      |      ` -'\       ー'  人          一番嫌いなのは論文数を増やすためにくだらない論文を書いて
    |        /(l     __/  ヽ、           良い論文の出版を遅らせるお馬鹿な人。
     |       (:::::`‐-、__  |::::`、     ヒニニヽ、         あなたの論文が Ann of Math に accept される確率は?
    |      / `‐-、::::::::::`‐-、::::\   /,ニニ、\            それとも最近は Inv. Math. の方が上かしら?
   |      |::::::::::::::::::|` -、:::::::,ヘ ̄|'、  ヒニ二、 \
.   |      /::::::::::::::::::|::::::::\/:::O`、::\   | '、   \
   |      /:::::::::::::::::::/:::::::::::::::::::::::::::::'、::::\ノ  ヽ、  |
  |      |:::::/:::::::::/:::::::::::::::::::::::::::::::::::'、',::::'、  /:\__/‐、
  |      |/:::::::::::/::::::::::::::::::::::::::::::::::O::| '、::| く::::::::::::: ̄|
   |     /_..-'´ ̄`ー-、:::::::::::::::::::::::::::::::::::|/:/`‐'::\;;;;;;;_|
   |    |/::::::::::::::::::::::\:::::::::::::::::::::::::::::|::/::::|::::/:::::::::::/
    |   /:::::::::::::::::::::::::::::::::|:::::::::::::::::::::O::|::|::::::|:::::::::::::::/
126あのこうちやんは始皇帝だった:2012/04/22(日) 19:16:47.40

 お前たちは早く定職に就け!!!!!!!!!!!!!!!!!!!!!!!
127132人目の素数さん:2012/04/22(日) 19:18:01.34
          __ノ)-'´ ̄ ̄`ー- 、_
        , '´  _. -‐'''"二ニニ=-`ヽ、
      /   /:::::; -‐''"        `ーノ
     /   /:::::/           \
     /    /::::::/          | | |  |
     |   |:::::/ /     |  | | | |  |
      |   |::/ / / |  | ||  | | ,ハ .| ,ハ|
      |   |/ / / /| ,ハノ| /|ノレ,ニ|ル' 
     |   |  | / / レ',二、レ′ ,ィイ|゙/   私は只の数ヲタなんかとは付き合わないわ。
.     |   \ ∠イ  ,イイ|    ,`-' |      頭が良くて数学が出来てかっこいい人。それが必要条件よ。
     |     l^,人|  ` `-'     ゝ  |        さらに Ann.of Math に論文書けば十分条件にもなるわよ。
      |      ` -'\       ー'  人          一番嫌いなのは論文数を増やすためにくだらない論文を書いて
    |        /(l     __/  ヽ、           良い論文の出版を遅らせるお馬鹿な人。
     |       (:::::`‐-、__  |::::`、     ヒニニヽ、         あなたの論文が Ann of Math に accept される確率は?
    |      / `‐-、::::::::::`‐-、::::\   /,ニニ、\            それとも最近は Inv. Math. の方が上かしら?
   |      |::::::::::::::::::|` -、:::::::,ヘ ̄|'、  ヒニ二、 \
.   |      /::::::::::::::::::|::::::::\/:::O`、::\   | '、   \
   |      /:::::::::::::::::::/:::::::::::::::::::::::::::::'、::::\ノ  ヽ、  |
  |      |:::::/:::::::::/:::::::::::::::::::::::::::::::::::'、',::::'、  /:\__/‐、
  |      |/:::::::::::/::::::::::::::::::::::::::::::::::O::| '、::| く::::::::::::: ̄|
   |     /_..-'´ ̄`ー-、:::::::::::::::::::::::::::::::::::|/:/`‐'::\;;;;;;;_|
   |    |/::::::::::::::::::::::\:::::::::::::::::::::::::::::|::/::::|::::/:::::::::::/
    |   /:::::::::::::::::::::::::::::::::|:::::::::::::::::::::O::|::|::::::|:::::::::::::::/
128132人目の素数さん:2012/04/22(日) 19:29:53.83
>>116 です。
√3で括って
6a-3b=√3(ab-a-6)
a.bは有理数なので√3はそれに矛盾するから右辺は問答無用で0になるってことでしょうか? イマイチ飲め込めない

>>123 以後気をつけます。
129132人目の素数さん:2012/04/22(日) 19:34:04.04
          __ノ)-'´ ̄ ̄`ー- 、_
        , '´  _. -‐'''"二ニニ=-`ヽ、
      /   /:::::; -‐''"        `ーノ
     /   /:::::/           \
     /    /::::::/          | | |  |
     |   |:::::/ /     |  | | | |  |
      |   |::/ / / |  | ||  | | ,ハ .| ,ハ|
      |   |/ / / /| ,ハノ| /|ノレ,ニ|ル' 
     |   |  | / / レ',二、レ′ ,ィイ|゙/   私は只の数ヲタなんかとは付き合わないわ。
.     |   \ ∠イ  ,イイ|    ,`-' |      頭が良くて数学が出来てかっこいい人。それが必要条件よ。
     |     l^,人|  ` `-'     ゝ  |        さらに Ann.of Math に論文書けば十分条件にもなるわよ。
      |      ` -'\       ー'  人          一番嫌いなのは論文数を増やすためにくだらない論文を書いて
    |        /(l     __/  ヽ、           良い論文の出版を遅らせるお馬鹿な人。
     |       (:::::`‐-、__  |::::`、     ヒニニヽ、         あなたの論文が Ann of Math に accept される確率は?
    |      / `‐-、::::::::::`‐-、::::\   /,ニニ、\            それとも最近は Inv. Math. の方が上かしら?
   |      |::::::::::::::::::|` -、:::::::,ヘ ̄|'、  ヒニ二、 \
.   |      /::::::::::::::::::|::::::::\/:::O`、::\   | '、   \
   |      /:::::::::::::::::::/:::::::::::::::::::::::::::::'、::::\ノ  ヽ、  |
  |      |:::::/:::::::::/:::::::::::::::::::::::::::::::::::'、',::::'、  /:\__/‐、
  |      |/:::::::::::/::::::::::::::::::::::::::::::::::O::| '、::| く::::::::::::: ̄|
   |     /_..-'´ ̄`ー-、:::::::::::::::::::::::::::::::::::|/:/`‐'::\;;;;;;;_|
   |    |/::::::::::::::::::::::\:::::::::::::::::::::::::::::|::/::::|::::/:::::::::::/
    |   /:::::::::::::::::::::::::::::::::|:::::::::::::::::::::O::|::|::::::|:::::::::::::::/
130132人目の素数さん:2012/04/22(日) 19:35:45.77
> 6a-3b=√3(ab-a-6)
左辺は有理数
√3は無理数なので右辺が有理数になるのは(ab-a-6)が0の場合
よって右辺は0であり左辺も0である
131132人目の素数さん:2012/04/22(日) 19:37:10.37
          __ノ)-'´ ̄ ̄`ー- 、_
        , '´  _. -‐'''"二ニニ=-`ヽ、
      /   /:::::; -‐''"        `ーノ
     /   /:::::/           \
     /    /::::::/          | | |  |
     |   |:::::/ /     |  | | | |  |
      |   |::/ / / |  | ||  | | ,ハ .| ,ハ|
      |   |/ / / /| ,ハノ| /|ノレ,ニ|ル' 
     |   |  | / / レ',二、レ′ ,ィイ|゙/   私は只の数ヲタなんかとは付き合わないわ。
.     |   \ ∠イ  ,イイ|    ,`-' |      頭が良くて数学が出来てかっこいい人。それが必要条件よ。
     |     l^,人|  ` `-'     ゝ  |        さらに Ann.of Math に論文書けば十分条件にもなるわよ。
      |      ` -'\       ー'  人          一番嫌いなのは論文数を増やすためにくだらない論文を書いて
    |        /(l     __/  ヽ、           良い論文の出版を遅らせるお馬鹿な人。
     |       (:::::`‐-、__  |::::`、     ヒニニヽ、         あなたの論文が Ann of Math に accept される確率は?
    |      / `‐-、::::::::::`‐-、::::\   /,ニニ、\            それとも最近は Inv. Math. の方が上かしら?
   |      |::::::::::::::::::|` -、:::::::,ヘ ̄|'、  ヒニ二、 \
.   |      /::::::::::::::::::|::::::::\/:::O`、::\   | '、   \
   |      /:::::::::::::::::::/:::::::::::::::::::::::::::::'、::::\ノ  ヽ、  |
  |      |:::::/:::::::::/:::::::::::::::::::::::::::::::::::'、',::::'、  /:\__/‐、
  |      |/:::::::::::/::::::::::::::::::::::::::::::::::O::| '、::| く::::::::::::: ̄|
   |     /_..-'´ ̄`ー-、:::::::::::::::::::::::::::::::::::|/:/`‐'::\;;;;;;;_|
   |    |/::::::::::::::::::::::\:::::::::::::::::::::::::::::|::/::::|::::/:::::::::::/
    |   /:::::::::::::::::::::::::::::::::|:::::::::::::::::::::O::|::|::::::|:::::::::::::::/
132132人目の素数さん:2012/04/22(日) 19:42:34.75
>>130
なるほど、やっと理解できました。
あとはなんとか解けそうです、ありがとうございました。
133132人目の素数さん:2012/04/22(日) 21:28:29.93
10^(1/2)×5^(-1/2)×2^(1/2)=

初歩的な問題ですいません
5^(-1/2)が分数でマイナスになっていて分かりません・・・
何方か分かりますか?
134132人目の素数さん:2012/04/22(日) 21:29:15.53
          __ノ)-'´ ̄ ̄`ー- 、_
        , '´  _. -‐'''"二ニニ=-`ヽ、
      /   /:::::; -‐''"        `ーノ
     /   /:::::/           \
     /    /::::::/          | | |  |
     |   |:::::/ /     |  | | | |  |
      |   |::/ / / |  | ||  | | ,ハ .| ,ハ|
      |   |/ / / /| ,ハノ| /|ノレ,ニ|ル' 
     |   |  | / / レ',二、レ′ ,ィイ|゙/   私は只の数ヲタなんかとは付き合わないわ。
.     |   \ ∠イ  ,イイ|    ,`-' |      頭が良くて数学が出来てかっこいい人。それが必要条件よ。
     |     l^,人|  ` `-'     ゝ  |        さらに Ann.of Math に論文書けば十分条件にもなるわよ。
      |      ` -'\       ー'  人          一番嫌いなのは論文数を増やすためにくだらない論文を書いて
    |        /(l     __/  ヽ、           良い論文の出版を遅らせるお馬鹿な人。
     |       (:::::`‐-、__  |::::`、     ヒニニヽ、         あなたの論文が Ann of Math に accept される確率は?
    |      / `‐-、::::::::::`‐-、::::\   /,ニニ、\            それとも最近は Inv. Math. の方が上かしら?
   |      |::::::::::::::::::|` -、:::::::,ヘ ̄|'、  ヒニ二、 \
.   |      /::::::::::::::::::|::::::::\/:::O`、::\   | '、   \
   |      /:::::::::::::::::::/:::::::::::::::::::::::::::::'、::::\ノ  ヽ、  |
  |      |:::::/:::::::::/:::::::::::::::::::::::::::::::::::'、',::::'、  /:\__/‐、
  |      |/:::::::::::/::::::::::::::::::::::::::::::::::O::| '、::| く::::::::::::: ̄|
   |     /_..-'´ ̄`ー-、:::::::::::::::::::::::::::::::::::|/:/`‐'::\;;;;;;;_|
   |    |/::::::::::::::::::::::\:::::::::::::::::::::::::::::|::/::::|::::/:::::::::::/
    |   /:::::::::::::::::::::::::::::::::|:::::::::::::::::::::O::|::|::::::|:::::::::::::::/
135132人目の素数さん:2012/04/22(日) 21:30:23.64
1/√5
136133です:2012/04/22(日) 21:43:28.36
>>135さん
回答有難うございます
出来れば解き方も教えてくれませんか?
ホントすいません・・・
137132人目の素数さん:2012/04/22(日) 21:45:28.88
http://i.imgur.com/3imWv.jpg
この図で、角ACB=角TBAらしいのですがどうしてですか
初歩的な質問ですみません・・・
138132人目の素数さん:2012/04/22(日) 21:47:38.03
>>136
解き方も何も
a^(-n) = 1/(a^n)


5^(-3) = 1/5^3 = 1/125
だけなんだが・・・
139132人目の素数さん:2012/04/22(日) 21:49:47.12
>>137
せつげんていり

教科書より
140132人目の素数さん:2012/04/22(日) 21:51:35.32
>>136
あ、いや>>135は答えじゃなくて、5^(-1/2)の事だよ
つまり、5^(-1/2)=(1/5)^(1/2)

だから>>133
10^(1/2)×5^(-1/2)×2^(1/2)
=10^(1/2)×(1/5)^(1/2)×2^(1/2)
=(10×(1/5)×2)^(1/2)
これを計算するだけ
141132人目の素数さん:2012/04/22(日) 22:23:50.73
>>138 >>140
回答有難うございます・・・
大変参考になりました
142132人目の素数さん:2012/04/22(日) 22:33:25.84
          __ノ)-'´ ̄ ̄`ー- 、_
        , '´  _. -‐'''"二ニニ=-`ヽ、
      /   /:::::; -‐''"        `ーノ
     /   /:::::/           \
     /    /::::::/          | | |  |
     |   |:::::/ /     |  | | | |  |
      |   |::/ / / |  | ||  | | ,ハ .| ,ハ|
      |   |/ / / /| ,ハノ| /|ノレ,ニ|ル' 
     |   |  | / / レ',二、レ′ ,ィイ|゙/   私は只の数ヲタなんかとは付き合わないわ。
.     |   \ ∠イ  ,イイ|    ,`-' |      頭が良くて数学が出来てかっこいい人。それが必要条件よ。
     |     l^,人|  ` `-'     ゝ  |        さらに Ann.of Math に論文書けば十分条件にもなるわよ。
      |      ` -'\       ー'  人          一番嫌いなのは論文数を増やすためにくだらない論文を書いて
    |        /(l     __/  ヽ、           良い論文の出版を遅らせるお馬鹿な人。
     |       (:::::`‐-、__  |::::`、     ヒニニヽ、         あなたの論文が Ann of Math に accept される確率は?
    |      / `‐-、::::::::::`‐-、::::\   /,ニニ、\            それとも最近は Inv. Math. の方が上かしら?
   |      |::::::::::::::::::|` -、:::::::,ヘ ̄|'、  ヒニ二、 \
.   |      /::::::::::::::::::|::::::::\/:::O`、::\   | '、   \
   |      /:::::::::::::::::::/:::::::::::::::::::::::::::::'、::::\ノ  ヽ、  |
  |      |:::::/:::::::::/:::::::::::::::::::::::::::::::::::'、',::::'、  /:\__/‐、
  |      |/:::::::::::/::::::::::::::::::::::::::::::::::O::| '、::| く::::::::::::: ̄|
   |     /_..-'´ ̄`ー-、:::::::::::::::::::::::::::::::::::|/:/`‐'::\;;;;;;;_|
   |    |/::::::::::::::::::::::\:::::::::::::::::::::::::::::|::/::::|::::/:::::::::::/
    |   /:::::::::::::::::::::::::::::::::|:::::::::::::::::::::O::|::|::::::|:::::::::::::::/
143132人目の素数さん:2012/04/22(日) 22:34:08.71
1 名無しさんにズームイン! [] Date:2012/03/28(水) 08:28:15.02 ?ID:NWYs/2ZP Be:
 やらなけゃいけない
 電○の各局への圧力が半端ないんです
 昨日、一昨日前田AKB卒業ネタやった情報番組全てが前田AKB卒業ネタ中の毎分視聴率ダダ下がりしました。
 各局本音では毎分視聴率ダダ下がりするこのネタははやりたくなかったけど原子力村以上に電○からの圧力が凄いんです

なんか海外でも日本でAKBがバカ売れしてるっていう情報に騙されてAKBヲタになった人が最近気付き始めてる件
http://hayabusa3.2ch.net/test/read.cgi/morningcoffee/1334516585/

税金垂れ流しのヤラセブーム
144132人目の素数さん:2012/04/22(日) 23:37:29.29

IをRの区間、→f:I →R’n , →g:I→R’nとする。

◆質点が等速運動するならば(つまり時刻tにおける位置を→f(t)と表すとき、||→f’t(t)||が定数関数となる)、
速度と加速度はつねに直交することを示せ。
145132人目の素数さん:2012/04/22(日) 23:37:50.27
144、教えていただけませんか?
146132人目の素数さん:2012/04/22(日) 23:46:36.36
マルチか
急いでいる問題スレなら、答えてくれる酔狂な人もいるかも
147132人目の素数さん:2012/04/22(日) 23:53:50.44
>>144
等速円運動するならば?
148132人目の素数さん:2012/04/23(月) 00:00:46.91
ω^10+ω^8

誰か教えてくれえ
149132人目の素数さん:2012/04/23(月) 00:02:50.88
>>144
一次元質点モデルを全否定するような問題だな
物理板の電波系スレの方が適切かと
150132人目の素数さん:2012/04/23(月) 00:04:15.39
ω^3=1
151132人目の素数さん:2012/04/23(月) 00:07:32.40
>>150
どう式変形すればええねん
152132人目の素数さん:2012/04/23(月) 00:14:15.05
144、円運動ではないんです。
多変数の微分積分の問題なのですが…。
物理なのでしょうか?
153132人目の素数さん:2012/04/23(月) 00:14:44.12
>>144
速度ベクトルを v ,加速度ベクトルを a ,|v|= u ,
τ = v/u (速度ベクトルと同じ向きの単位ベクトル)とする
v = uτ なので,この両辺を t で微分して
  a = dv/dt = (du/dt)τ + u(dτ/dt) = u(dτ/dt) (∵ 条件より du/dt = 0 )
ここで, (|τ|^2 = ) τ・τ = 1 の両辺を t で微分して,
  2(dτ/dt)・τ = 0
したがって  a・τ = 0

ベクトル解析などではボールド体を用いて表現するが
レスでは無理なのでこういう感じにした
高校生でも背伸びすれば理解できるとは思うが
背伸びしてやってるのならこんなところで質問せずに
もっとちゃんと本を読めといいたい
154132人目の素数さん:2012/04/23(月) 00:17:25.72
>>151
ω^10をどうにかする
155132人目の素数さん:2012/04/23(月) 00:17:50.82
>>151
あと、ω^8もな
指数法則知ってるやろ?
あれを上手く使えや
156132人目の素数さん:2012/04/23(月) 00:19:26.90
ありがとうございます。
数学系の学科で、先生に課題として出されたのですが…。
等速円運動が例に挙げられていて、それは理解出来たのですが、どうしても応用が出来なかったのです。
157132人目の素数さん:2012/04/23(月) 00:33:06.97
>>155
なんとかできたわ。ありがとな
158132人目の素数さん:2012/04/23(月) 01:32:15.57
因数分解してください

a^2(b+c)+b^2(c+a)+c^2(a+b)+3abc
159132人目の素数さん:2012/04/23(月) 01:53:04.68
(a+b+c)(ab+bc+ca)
160132人目の素数さん:2012/04/23(月) 01:58:26.50
>>159
ありがとう

途中式頼む
161132人目の素数さん:2012/04/23(月) 02:03:30.43
数式書いたら書き込む前に検索するぐらいしような。
んなもん覚えるレベルで途中式なんか無い。
159の式バラしてみろよ。そのままだ
162132人目の素数さん:2012/04/23(月) 02:04:52.94
>>161
くぐったんだがな


ありがとうございました
163132人目の素数さん:2012/04/23(月) 02:04:59.62
工夫したやり方あった気がしたけど
aの2次式としてみたらすぐに因数分解できる
164132人目の素数さん:2012/04/23(月) 02:11:44.78
a^2(b+c)+b^2(c+a)+c^2(a+b)+3abc
この式コピペしてググるとすぐにとべる
知恵遅に丁寧に載ってるぞ。まぁaについて整理してもそのたすきがけ気付けるようなら多分質問しないとおもうが
165132人目の素数さん:2012/04/23(月) 02:11:50.17
>>158>>159 の理由なら
神のお告げ、俺天才だから瞬殺、電波キャッチ…でおk
166132人目の素数さん:2012/04/23(月) 02:50:08.11
なんとか理解できたサンクス


記憶ゲーなのか
167132人目の素数さん:2012/04/23(月) 02:55:18.94
別に計算力に自信があるなら毎回その場で計算してもいいんだぜ?
メジャーな恒等式だし、覚えたら。そんなに出ないから数学武器にするつもりでもなきゃ覚えてない奴の方が多いだろうけど。
168132人目の素数さん:2012/04/23(月) 02:56:31.90
>>166
>>163流でおk
これが記憶ゲーなら、記憶ゲーです
169132人目の素数さん:2012/04/23(月) 05:08:25.78
三角比の単元はどうも公式を当てはめるだけの作業の用にしか見えなくて
飽きてくる

もっと考え方を変えれないものか
170132人目の素数さん:2012/04/23(月) 05:29:22.41
>>169
教科書レベルだとそれも仕方がない
やはりそれなりの規模の問題でないと手ごたえを感じないかもしれない
問題の式の図形的意味を調べてみたり
別解が多数出ている問題集で視点を変えてみたりすればいいだろう
171132人目の素数さん:2012/04/23(月) 07:32:01.85
最低次数の文字について整理の仕方がわかりません…
簡単なものなら出来るのですが、
x^2y-2xyz-y-xy^2+x+-2z
=x^2-y-xy^2+x-2(xy+1)z …… @
=x(xy+1)-y(xy+1)-2(xy+1)z…… A
のように、zについて整理するみたいなのですが
何をどうすれば@、Aのようになるのか
高認レベルの私にも分かるように、1から教えてもらえないでしょうか。
よろしくお願いします<m(_ _)m>
172132人目の素数さん:2012/04/23(月) 07:33:49.15
x^2y-2xyz-y-xy^2+x+-2z
じゃなく
x^2y-2xyz-y-xy^2+x-2z です
173132人目の素数さん:2012/04/23(月) 07:38:03.97
>>171
zについて整理すると認識していて@も理解できないのか?
174132人目の素数さん:2012/04/23(月) 08:04:02.93
          __ノ)-'´ ̄ ̄`ー- 、_
        , '´  _. -‐'''"二ニニ=-`ヽ、
      /   /:::::; -‐''"        `ーノ
     /   /:::::/           \
     /    /::::::/          | | |  |
     |   |:::::/ /     |  | | | |  |
      |   |::/ / / |  | ||  | | ,ハ .| ,ハ|
      |   |/ / / /| ,ハノ| /|ノレ,ニ|ル' 
     |   |  | / / レ',二、レ′ ,ィイ|゙/   私は只の数ヲタなんかとは付き合わないわ。
.     |   \ ∠イ  ,イイ|    ,`-' |      頭が良くて数学が出来てかっこいい人。それが必要条件よ。
     |     l^,人|  ` `-'     ゝ  |        さらに Ann.of Math に論文書けば十分条件にもなるわよ。
      |      ` -'\       ー'  人          一番嫌いなのは論文数を増やすためにくだらない論文を書いて
    |        /(l     __/  ヽ、           良い論文の出版を遅らせるお馬鹿な人。
     |       (:::::`‐-、__  |::::`、     ヒニニヽ、         あなたの論文が Ann of Math に accept される確率は?
    |      / `‐-、::::::::::`‐-、::::\   /,ニニ、\            それとも最近は Inv. Math. の方が上かしら?
   |      |::::::::::::::::::|` -、:::::::,ヘ ̄|'、  ヒニ二、 \
.   |      /::::::::::::::::::|::::::::\/:::O`、::\   | '、   \
   |      /:::::::::::::::::::/:::::::::::::::::::::::::::::'、::::\ノ  ヽ、  |
  |      |:::::/:::::::::/:::::::::::::::::::::::::::::::::::'、',::::'、  /:\__/‐、
  |      |/:::::::::::/::::::::::::::::::::::::::::::::::O::| '、::| く::::::::::::: ̄|
   |     /_..-'´ ̄`ー-、:::::::::::::::::::::::::::::::::::|/:/`‐'::\;;;;;;;_|
   |    |/::::::::::::::::::::::\:::::::::::::::::::::::::::::|::/::::|::::/:::::::::::/
    |   /:::::::::::::::::::::::::::::::::|:::::::::::::::::::::O::|::|::::::|:::::::::::::::/
175132人目の素数さん:2012/04/23(月) 08:04:58.06
一番をzについて整理してるって見れるなら充分解っていると思うけどね。
176132人目の素数さん:2012/04/23(月) 08:08:18.38
          __ノ)-'´ ̄ ̄`ー- 、_
        , '´  _. -‐'''"二ニニ=-`ヽ、
      /   /:::::; -‐''"        `ーノ
     /   /:::::/           \
     /    /::::::/          | | |  |
     |   |:::::/ /     |  | | | |  |
      |   |::/ / / |  | ||  | | ,ハ .| ,ハ|
      |   |/ / / /| ,ハノ| /|ノレ,ニ|ル' 
     |   |  | / / レ',二、レ′ ,ィイ|゙/   私は只の数ヲタなんかとは付き合わないわ。
.     |   \ ∠イ  ,イイ|    ,`-' |      頭が良くて数学が出来てかっこいい人。それが必要条件よ。
     |     l^,人|  ` `-'     ゝ  |        さらに Ann.of Math に論文書けば十分条件にもなるわよ。
      |      ` -'\       ー'  人          一番嫌いなのは論文数を増やすためにくだらない論文を書いて
    |        /(l     __/  ヽ、           良い論文の出版を遅らせるお馬鹿な人。
     |       (:::::`‐-、__  |::::`、     ヒニニヽ、         あなたの論文が Ann of Math に accept される確率は?
    |      / `‐-、::::::::::`‐-、::::\   /,ニニ、\            それとも最近は Inv. Math. の方が上かしら?
   |      |::::::::::::::::::|` -、:::::::,ヘ ̄|'、  ヒニ二、 \
.   |      /::::::::::::::::::|::::::::\/:::O`、::\   | '、   \
   |      /:::::::::::::::::::/:::::::::::::::::::::::::::::'、::::\ノ  ヽ、  |
  |      |:::::/:::::::::/:::::::::::::::::::::::::::::::::::'、',::::'、  /:\__/‐、
  |      |/:::::::::::/::::::::::::::::::::::::::::::::::O::| '、::| く::::::::::::: ̄|
   |     /_..-'´ ̄`ー-、:::::::::::::::::::::::::::::::::::|/:/`‐'::\;;;;;;;_|
   |    |/::::::::::::::::::::::\:::::::::::::::::::::::::::::|::/::::|::::/:::::::::::/
    |   /:::::::::::::::::::::::::::::::::|:::::::::::::::::::::O::|::|::::::|:::::::::::::::/
177132人目の素数さん:2012/04/23(月) 09:44:36.87
>>171 はい…
そもそも整理するってなに?って状態です
参考書は、本質の講義をもってるのですが、
整理について書かれていません。これでわかう数学も
立ち読みしましたが、「最低次数を整理して〜なる」
とし書かれてなくて、全くわかりません…
できれば、順を追って説明してもらえないでしょうか?
178132人目の素数さん:2012/04/23(月) 09:54:16.14
>>177
そりゃハイレベルな参考書には書かれてないだろ。それを読む時点ではわかっていて当然のことだから。
つまり、君はその参考書を読むレベルになってないってことだ。
教科書に戻れ。
179132人目の素数さん:2012/04/23(月) 10:02:37.71
>>177
「共通因数でくくる」って表現はわかる?

 入門 ax + bx = ( a + b ) x
 初級 a ( x + y ) + b ( x + y ) = ( a + b ) ( x + y ) ← ( x + y ) が共通因数だったのでそれでくくった
    (よくわからなければ ( x + y ) = z とでも置き換えよ)
180132人目の素数さん:2012/04/23(月) 12:39:51.62
>>178 本質の講義っていう本は、検定教科書に音声をつけた
参考書です。でも、上のような複雑な整理の仕方とかは
説明がありません。
これでわかる数学も、初学者用の参考書なのですが、やはり説明は省かれています。
最も入門的な参考書で有名な、マセマの初めから始める数学も読んだのですが
これには、整理の「せ」の字すらでてなくて…

>>179 はい、これくらいなら理解できてますが、上のように
長く複雑になると混乱してしまいます。
z以外は、ただの数字と認識せよ。と言われたのですが
もう少し、わかりやすく解説してもらえないでしょうか?
181132人目の素数さん:2012/04/23(月) 13:08:03.42
>>180
  ( x^2 )y − 2xyz − y − xy^2 + x − 2z
 = ( x^2 )y + x − y − xy^2− 2z( xy + 1 ) …@
 = x( xy + 1 ) − y( xy + 1 ) − 2( xy + 1 )z …A

「次数の低い文字について整理する」という定石に従って
整理したら@のようになった(項の順番は少し変えてある)
ここまではいいんだな? ( z を特別視するのはとりあえずここまで)
で,@ z の係数に含まれる( xy + 1 )が
「他の部分にも現れるのではないか」と “見当” を付ける
(この部分は数をこなすうちにできるようになる,できないなら練習が不足しているということ)
実際,残りの部分で幾つかをまとめて共通因数でくくってみればAのようになる
多少の試行錯誤は不可欠であり,とにかくいろいろやってみる(手を動かす)ことが重要
182132人目の素数さん:2012/04/23(月) 13:29:29.91
てす
183132人目の素数さん:2012/04/23(月) 13:32:11.10
x,y は実数とする。
a=(3,1,-2) b=(1,x,-1) c=(-2,1,y)
a+c, b+2c が平行となるx,y の値を求めよ。
やり方を忘れてしまった…誰か教えてください。
184132人目の素数さん:2012/04/23(月) 13:32:26.89
ごめんなさい…
@の時点で、何がなんやら分かりません(;>_<;)
最初は、(x^2)yは与式と共通してるのは一目瞭然ですが
次の− 2xyz − y − xy^2 + x − 2z が
+ x − y − xy^2− 2z( xy + 1 )に変形するのか全然理解出来ません…
これが理解できないと、教科書レベルの例題が解けないので
助けてください
185132人目の素数さん:2012/04/23(月) 13:43:34.26
教科書の最初の例題で
xの多項式とみて降べきの順にならべよ
とかあったでしょ

-2z
-2xyz
zの1次式
-2や-2xyを係数と見る
186132人目の素数さん:2012/04/23(月) 13:49:15.30
>>184
さっきとはまた少し変えてある
  ( x^2 )y − 2xyz − y − xy^2 + x − 2z  ←与式
 = −2xyz − 2z + ( x^2 )y − y − xy^2 + x ← z のある項を前に持ってきた(これを「 z について整理」したという)
 = −2z( xy + 1 ) + ( x^2 )y − y − xy^2 + x ←頭の2項を −2z でくくった @
 = −2z( xy + 1 ) + {( x^2 )y + x } − { xy^2 + y } ←少し順番を変えて{ }をつけた A
で,@で見えた( xy + 1 )が出てくるように
Aの2つの{ }をそれぞれ整理(何かでくくる)

ほんとはここまでいちいち書かなくてもできてほしいが
できないならとにかく手を動かせ
ここで質問するまでにチラシ10枚くらいは使ったんだろうな?
187132人目の素数さん:2012/04/23(月) 13:53:07.10
解説を見てもわからないときは
解説に書いてある式の整理を「逆に辿る」(この問題では,結果を展開してみる)ことも
言われなくてもやるようにしてほしい
188132人目の素数さん:2012/04/23(月) 14:10:50.72
>長く複雑になると混乱してしまいます。

注目する文字を鉛筆で丸つけたら?
それの次数が同じ項でまとめる。

こういうだせぇ方法を家庭教師先で教えても、バカな奴程出来ない癖に従わないんだよね。

>z以外は、ただの数字と認識せよ。と言われたのですが
>もう少し、わかりやすく解説してもらえないでしょうか?

これ以上簡単に説明しようがないよ。

y=ax^2+bx+cのabcは数学って捉えるのと同じようなもんだよ。
189132人目の素数さん:2012/04/23(月) 14:29:50.28
>>186 あぁ…霧が晴れました!!
10枚どころか、この問題を2日間、悶々と考え続けましたw
本当にありがとうございます!
a^2-ac-b^2+bcを(-a+b)c+(a^2-b^2)
にするのと、何ら変わらないですね。本当に助かりました
190132人目の素数さん:2012/04/23(月) 14:31:05.96
智者はやるべきことをやる
愚者はやりたいようにやる
191132人目の素数さん:2012/04/23(月) 17:55:40.70
          __ノ)-'´ ̄ ̄`ー- 、_
        , '´  _. -‐'''"二ニニ=-`ヽ、
      /   /:::::; -‐''"        `ーノ
     /   /:::::/           \
     /    /::::::/          | | |  |
     |   |:::::/ /     |  | | | |  |
      |   |::/ / / |  | ||  | | ,ハ .| ,ハ|
      |   |/ / / /| ,ハノ| /|ノレ,ニ|ル' 
     |   |  | / / レ',二、レ′ ,ィイ|゙/   私は只の数ヲタなんかとは付き合わないわ。
.     |   \ ∠イ  ,イイ|    ,`-' |      頭が良くて数学が出来てかっこいい人。それが必要条件よ。
     |     l^,人|  ` `-'     ゝ  |        さらに Ann.of Math に論文書けば十分条件にもなるわよ。
      |      ` -'\       ー'  人          一番嫌いなのは論文数を増やすためにくだらない論文を書いて
    |        /(l     __/  ヽ、           良い論文の出版を遅らせるお馬鹿な人。
     |       (:::::`‐-、__  |::::`、     ヒニニヽ、         あなたの論文が Ann of Math に accept される確率は?
    |      / `‐-、::::::::::`‐-、::::\   /,ニニ、\            それとも最近は Inv. Math. の方が上かしら?
   |      |::::::::::::::::::|` -、:::::::,ヘ ̄|'、  ヒニ二、 \
.   |      /::::::::::::::::::|::::::::\/:::O`、::\   | '、   \
   |      /:::::::::::::::::::/:::::::::::::::::::::::::::::'、::::\ノ  ヽ、  |
  |      |:::::/:::::::::/:::::::::::::::::::::::::::::::::::'、',::::'、  /:\__/‐、
  |      |/:::::::::::/::::::::::::::::::::::::::::::::::O::| '、::| く::::::::::::: ̄|
   |     /_..-'´ ̄`ー-、:::::::::::::::::::::::::::::::::::|/:/`‐'::\;;;;;;;_|
   |    |/::::::::::::::::::::::\:::::::::::::::::::::::::::::|::/::::|::::/:::::::::::/
    |   /:::::::::::::::::::::::::::::::::|:::::::::::::::::::::O::|::|::::::|:::::::::::::::/
192132人目の素数さん:2012/04/23(月) 18:31:03.54
円 x^2+y^2+2x+4y-4=0 の接線で、傾きが2のものの接線の方程式とその接点の座標を求める問題で
解が y=2x+3√5 (-5-6√5/5 , -10+3√5/5) ; y=2x-3√5 (-5+6√5/5 , -10-3√5/5)なのですが、
学校から解答が配られておらず途中の計算がわかりません。
どなたか解法を教えていただけないでしょうか
193132人目の素数さん:2012/04/23(月) 18:41:34.26
>>192
円 x^2+y^2+2x+4y-4=0の中心と半径をまず求めよ
194132人目の素数さん:2012/04/23(月) 18:51:59.27
>>193
中心(-1,-2)、半径3の円になりました
195132人目の素数さん:2012/04/23(月) 18:57:32.28
>>194
そこまでわかったら傾き2の直線との接点なんかグラフ書きゃすぐわかるだろうに。
196132人目の素数さん:2012/04/23(月) 19:08:17.09
>>195
解けましたありがとうございます
197132人目の素数さん:2012/04/23(月) 19:09:57.14
平面において、点A(a↑)を中心とした半径rの円をCとし、
点P0(p0↑)はこの円周上の点でないとする。
点P0を通り単位方向ベクトルe↑の直線lがこの円Cと共有点P1,P2を持つとする。
ただし、点P0(p0↑)が円Cの外部の点のときはP1=P2の場合(直線lが円Cに接する場合)
も許すこととする。P0P1↑=t1*e↑、P0P2↑=t2*e↑とおく。
このとき「t1・t2=|p0↑−a↑|^2−r^2」であることを示しなさい。

よろしくお願いします。

198あのこうちやんは始皇帝だった:2012/04/23(月) 19:32:21.77

 お前たちは、定職に就くのが先決だろがああああ!!!!!!!!!!!!!!1

 ニート・無職の、ゴミ・クズ・カスのクソガキどもがああああ!!!!!!!!!!!!!
199132人目の素数さん:2012/04/23(月) 19:40:03.20
          __ノ)-'´ ̄ ̄`ー- 、_
        , '´  _. -‐'''"二ニニ=-`ヽ、
      /   /:::::; -‐''"        `ーノ
     /   /:::::/           \
     /    /::::::/          | | |  |
     |   |:::::/ /     |  | | | |  |
      |   |::/ / / |  | ||  | | ,ハ .| ,ハ|
      |   |/ / / /| ,ハノ| /|ノレ,ニ|ル' 
     |   |  | / / レ',二、レ′ ,ィイ|゙/   私は只の数ヲタなんかとは付き合わないわ。
.     |   \ ∠イ  ,イイ|    ,`-' |      頭が良くて数学が出来てかっこいい人。それが必要条件よ。
     |     l^,人|  ` `-'     ゝ  |        さらに Ann.of Math に論文書けば十分条件にもなるわよ。
      |      ` -'\       ー'  人          一番嫌いなのは論文数を増やすためにくだらない論文を書いて
    |        /(l     __/  ヽ、           良い論文の出版を遅らせるお馬鹿な人。
     |       (:::::`‐-、__  |::::`、     ヒニニヽ、         あなたの論文が Ann of Math に accept される確率は?
    |      / `‐-、::::::::::`‐-、::::\   /,ニニ、\            それとも最近は Inv. Math. の方が上かしら?
   |      |::::::::::::::::::|` -、:::::::,ヘ ̄|'、  ヒニ二、 \
.   |      /::::::::::::::::::|::::::::\/:::O`、::\   | '、   \
   |      /:::::::::::::::::::/:::::::::::::::::::::::::::::'、::::\ノ  ヽ、  |
  |      |:::::/:::::::::/:::::::::::::::::::::::::::::::::::'、',::::'、  /:\__/‐、
  |      |/:::::::::::/::::::::::::::::::::::::::::::::::O::| '、::| く::::::::::::: ̄|
   |     /_..-'´ ̄`ー-、:::::::::::::::::::::::::::::::::::|/:/`‐'::\;;;;;;;_|
   |    |/::::::::::::::::::::::\:::::::::::::::::::::::::::::|::/::::|::::/:::::::::::/
    |   /:::::::::::::::::::::::::::::::::|:::::::::::::::::::::O::|::|::::::|:::::::::::::::/
200132人目の素数さん:2012/04/23(月) 19:51:19.27
微分についてですが、ライプニッツの記法がニュートンのものより
優れているとされる理由はなんでしょうか。
どちらも同じに思えます。
201132人目の素数さん:2012/04/23(月) 19:55:40.26
>>200
近くの大学図書館にいって
プリンキピアと
ライプニッツ著作集を読んで
どちらが分かりやすいか自分で確かめろ
202132人目の素数さん:2012/04/23(月) 20:05:05.07
わかりました。
近くに大学はないので、市立図書館に行ってみます。
203132人目の素数さん:2012/04/23(月) 20:24:14.71
そもそもプリンキピアに微積出てこない
204132人目の素数さん:2012/04/23(月) 20:35:10.07
横だけど
>>197
(AP1+AP2)*P0P1 = 0
(ベクトル記号略)
は図形的に明らかなんだけどどう示すんだろう・・
205132人目の素数さん:2012/04/23(月) 20:38:36.33
もう仕事退けてる時間だっつーの
206132人目の素数さん:2012/04/23(月) 20:41:27.65
>>197
方べきの定理をベクトルで表現しただけ
直線P0Aを描いて右辺を “因数分解” した式を眺めてみればわかる
207132人目の素数さん:2012/04/23(月) 21:14:57.57
x=cos^4θ y=sin^4θ(0≦θ≦π/2)
の接線の方程式を教えてください
208132人目の素数さん:2012/04/23(月) 21:16:17.27
dy/dx=(dy/dθ)/(dx/dθ)
209132人目の素数さん:2012/04/23(月) 21:17:09.06
>>208
ありがとうございます
210ようじょ ◆hNziS2E8421X :2012/04/23(月) 21:40:08.22
a[n]=Σ[k=1,n]1/√k b[n]=Σ[k=1,n]1/√2k+1とするとき、

(1)lim[n→∞]a[n]

(2)lim[n→∞]b[n]/a[n]を求めよ


ぜんぜんわかりません。
絵みたいに書いてしかくてきに答えをもとめちゃだめですか?
それならいけそうなんですけど、(2)のほうがぜんぜんわかりません・・・
211132人目の素数さん:2012/04/23(月) 21:44:32.93
直線x+2y=0に関して、点A(3.ー4)と対称な点Bを求めよ。
これ誰か教えてください。
212132人目の素数さん:2012/04/23(月) 21:47:00.37
>>210
a[n]=Σ[k=1,n]1/√k≧Σ[k=1,n]1/k
b[n]=Σ[k=1,n]1/√2k+1≧Σ[k=1,n]1/(2k+1)
213ようじょ ◆hNziS2E8421X :2012/04/23(月) 21:50:19.76
>>212
どういうこと・・・?
よくわかんあい
214132人目の素数さん:2012/04/23(月) 21:55:20.38
また失礼します
媒介変数で表された曲線
x=cos^4θ y=sin^4θ (0≦θ≦π/2)
と、その接線およびx軸、y軸で囲まれた、2つの面積の和が1/24であるという
この接線の方程式を求めよ

という問題なのですが、
∫[0,1] y dx
x=cos^4θ
dx/dθ=-4sinθ*cos^3θ
=4∫[π/2,0]sin^5θcos^3θdθ
=1/6
このあと、接点の座標を(cos^4a,sin^4a)とおくのだと思うんですが
ここから先わかりません
よろしくお願いします
215132人目の素数さん:2012/04/23(月) 21:58:41.47
a[n]-b[n]=Σ[k=1,n][1/√k -1/√(2k+1)]>Σ[k=1,n](1/√k -1/√2k)
=(1-1/√2)a[n]
216132人目の素数さん:2012/04/23(月) 22:00:24.14
>>213
k≧1でk^2≧k
k≧√k
1/√k≧1/k
217132人目の素数さん:2012/04/23(月) 22:07:10.47
>>214
>>208で接線の傾きをaで表す
218132人目の素数さん:2012/04/23(月) 22:11:00.10
質問2つあります。どちらも和を求める問題で

@1^2+2^2+…+(2n)^2

A(n+1)^2+(n+2)^2+…+(3n)^2


@はΣk^2の公式でn=2nとおけば良いのかなーとは思うけど、数列のイメージが湧かない
Aは全然わかんない
219132人目の素数さん:2012/04/23(月) 22:13:01.76
>>218
1^2+2^2+3^2+…+n^2+(n+1)^2+(n+2)^2+…+(3n)^2
220132人目の素数さん:2012/04/23(月) 22:16:56.38
三ケタぐらいのでかいたすきがけでいつも5分ぐらい時間取られるんだが、画期的な何かは無いのか?
221132人目の素数さん:2012/04/23(月) 22:17:09.95
>>219
1^2+2^2+…+n^2+(n+1)^2+…+(2n)^2+(2n+1)^2+(2n+2)^2+…+(3n)^2
ってことか!納得!ありがとう!
222ようじょ ◆hNziS2E8421X :2012/04/23(月) 22:18:23.92
>>216
そこまでは理解できたよ
でもその続きがよくわかんない・・・
ごめんなさい・・・
223132人目の素数さん:2012/04/23(月) 22:18:33.25
>>220
時を止める
224132人目の素数さん:2012/04/23(月) 22:19:10.25
>>222
Σ1/k→∞
225ようじょ ◆hNziS2E8421X :2012/04/23(月) 22:22:40.32
>>224
あ、なるほど!わかった!!
なんどもすいませんでした;;
226132人目の素数さん:2012/04/23(月) 22:27:43.22
>>220
インド人に聞け
227132人目の素数さん:2012/04/23(月) 23:33:50.38
>>220
つ解の公式
228132人目の素数さん:2012/04/23(月) 23:58:48.00
229132人目の素数さん:2012/04/24(火) 00:00:59.59
>>228
とりち
230132人目の素数さん:2012/04/24(火) 00:01:10.92
>>228
>>1
>・質問者は何が分からないのか、どこまで考えたのかを明記しましょう。それがない場合、放置されることがあります。
231132人目の素数さん:2012/04/24(火) 02:55:14.34
三つの整数値を入力し、最大値を求めるアルゴリズムをフローチャートで記述して下さい

開始→最大値M=-9999→整数値を入力→終わりか?→イエス→終了
                                 →ノー 
???????????????????????????って感じなんですけど…
232132人目の素数さん:2012/04/24(火) 07:26:45.45
          __ノ)-'´ ̄ ̄`ー- 、_
        , '´  _. -‐'''"二ニニ=-`ヽ、
      /   /:::::; -‐''"        `ーノ
     /   /:::::/           \
     /    /::::::/          | | |  |
     |   |:::::/ /     |  | | | |  |
      |   |::/ / / |  | ||  | | ,ハ .| ,ハ|
      |   |/ / / /| ,ハノ| /|ノレ,ニ|ル' 
     |   |  | / / レ',二、レ′ ,ィイ|゙/   私は只の数ヲタなんかとは付き合わないわ。
.     |   \ ∠イ  ,イイ|    ,`-' |      頭が良くて数学が出来てかっこいい人。それが必要条件よ。
     |     l^,人|  ` `-'     ゝ  |        さらに Ann.of Math に論文書けば十分条件にもなるわよ。
      |      ` -'\       ー'  人          一番嫌いなのは論文数を増やすためにくだらない論文を書いて
    |        /(l     __/  ヽ、           良い論文の出版を遅らせるお馬鹿な人。
     |       (:::::`‐-、__  |::::`、     ヒニニヽ、         あなたの論文が Ann of Math に accept される確率は?
    |      / `‐-、::::::::::`‐-、::::\   /,ニニ、\            それとも最近は Inv. Math. の方が上かしら?
   |      |::::::::::::::::::|` -、:::::::,ヘ ̄|'、  ヒニ二、 \
.   |      /::::::::::::::::::|::::::::\/:::O`、::\   | '、   \
   |      /:::::::::::::::::::/:::::::::::::::::::::::::::::'、::::\ノ  ヽ、  |
  |      |:::::/:::::::::/:::::::::::::::::::::::::::::::::::'、',::::'、  /:\__/‐、
  |      |/:::::::::::/::::::::::::::::::::::::::::::::::O::| '、::| く::::::::::::: ̄|
   |     /_..-'´ ̄`ー-、:::::::::::::::::::::::::::::::::::|/:/`‐'::\;;;;;;;_|
   |    |/::::::::::::::::::::::\:::::::::::::::::::::::::::::|::/::::|::::/:::::::::::/
    |   /:::::::::::::::::::::::::::::::::|:::::::::::::::::::::O::|::|::::::|:::::::::::::::/
233132人目の素数さん:2012/04/24(火) 13:24:08.58
場合の数で教師はみんなコンビネーションやパーミテーションを書いてそれを計算したのが答えってやっているのですが、求める場合の数の計算を書いてそれをまとめたのがコンビネーションって言う教師もいました。どちらが正しいですか?
234132人目の素数さん:2012/04/24(火) 13:37:40.61
>>233
後者が正しい
235馬鹿甜菜:2012/04/24(火) 14:36:04.40
49 名前:132人目の素数さん []: 2012/04/24(火) 08:56:14.49
ブルーバックス社からでてる「新体系・高校(中学)数学」て本は、数学板住人からみるとどう?
あくまでも高校生までに習う範囲の教科書て位置付けなんだが、
体系と定理の証明に重点をおいていて、誤り部分が一切ないらしい
236132人目の素数さん:2012/04/24(火) 14:36:24.35
改行くらい覚えろ
237麒麟:2012/04/24(火) 15:03:39.24
231>>

1つずつ数値を比較すればいいんじゃない? 

最初の数字と最大値を比較→最初の数字が大きければその数字を最大値に
次の数字と最大値を比較→…ってやっていけばおk
238132人目の素数さん:2012/04/24(火) 16:46:11.05
>>234
ありがとうございます
239132人目の素数さん:2012/04/24(火) 19:35:02.34
1対1の二次関数「関数の決定」のところなのですが

2次関数 y=ax^2+bx+c のグラフが x 軸に接し、2点(1, -3), (-5, -75)
を通るとき、a, b, c の値を求めよ。

解答
x 軸との接点を P(p, 0) とおくと、P が頂点であるから、
 y=a(x-p)^2 ・・・@    と表せる
このグラフが 2点(1, -3), (-5, -75) を通るとき
 -3=a(1-p)^2 ・・・A
 -75=a(-5-p)^2 ・・・B
B/Aから
(p+5)^2/(p-1)^2=25 ∴ {(p+5)/(p-1)}^2=5^2
∴ (p+5)/(p-1)=±5

ここで解らないところがあるのですが
{(p+5)/(p-1)}^2=5^2 の2乗を外すとき
左辺は (p+5)/(p-1)
右辺は ±5
となるのですが、なぜ左辺には ±の記号が付かないのでしょうか
初歩的な質問で申し訳ありませんが、お答えいただければ幸いです
240132人目の素数さん:2012/04/24(火) 19:42:03.90
>>239
x^2=2 の解はどう書く?
241132人目の素数さん:2012/04/24(火) 19:46:34.46
±A=±B(複合任意)
⇔A=BまたはA=-Bまたは-A=Bまたは-A=-B
⇔A=BまたはA=-BまたはA=-BまたはA=B
⇔A=BまたはA=-B
⇔A=±B
で結局同じこと
242132人目の素数さん:2012/04/24(火) 19:49:48.45
{(p+5)/(p-1)}^2=5^2
ココだよな

見やすさ分かりやすさを追求して
(p+5)/(p-1) = A
とかおけば

A^2 = 25
A = ±5

1対1とかコーショーなモンやってるレヴェルじゃねーぞww
中学生レベルだw
243132人目の素数さん:2012/04/24(火) 19:50:32.65
(p+5)^2/(p-1)^2=25 ∴ {(p+5)/(p-1)}^2=5^2
∴ (p+5)/(p-1)=±5
こう考えるんじゃなくて
(p+5)^2/(p-1)^2=25
[(p+5)/(p-1)]^2-5^2=0
[(p+5)/(p-1)-5][(p+5)/(p-1)+5]=0
(p+5)/(p-1)=±5
って形で因数分解で解くよう考えよう。
その疑問が浮かぶ奴は直感に頼ると絶対に引っかかるから。
244132人目の素数さん:2012/04/24(火) 19:55:59.37
極限をやっているのですが
∞-∞≠0 になる理由がわかりません
∞-∞が0になるとは限らないという事はわかるのですが
なぜ0にはなれないのでしょうか?
245132人目の素数さん:2012/04/24(火) 19:57:45.55
>>240
なるほど
解りやすい例をありがとうございます
>>241
すごく解りやすいです
助かります
>>242
すいません(;´Д`)
>>243
そのような方法も有ったんですね
どちらかと言えば、こちらの方が自分にはあっているようです

みなさん、お騒がせしました
ありがとうございます
246132人目の素数さん:2012/04/24(火) 19:58:03.31
それは解釈の違いだ。0になるようなケースもある。
247132人目の素数さん:2012/04/24(火) 19:58:04.12
lim_[n→∞](n-n)=0じゃいかんのか
248132人目の素数さん:2012/04/24(火) 20:07:23.77
>>244
1+1+1+1+・・・・=∞
2+2+2+2+・・・・=∞
ゆえに
1+1+1+1+・・・・=2+2+2+2+・・・・

ここで
(2+2+2+2+・・・・)ー(1+1+1+1+・・・・)=1+1+1+1+・・・・=∞

これより
∞ー∞=∞
249132人目の素数さん:2012/04/24(火) 20:11:15.95
>>248
おめぇさんは日本語の勉強したほうがいいな
250132人目の素数さん:2012/04/24(火) 20:39:07.75
数IIIの微積の解き方って言われなきゃ思いつかねーよ的なのあるけど
あれって暗記しかないの?
251132人目の素数さん:2012/04/24(火) 20:46:03.39
試行錯誤。
どうしても自力でできないのなら仕方ないが。
252132人目の素数さん:2012/04/24(火) 20:52:31.09
どんな問題か
挙げてほしいもんだ
253132人目の素数さん:2012/04/24(火) 21:31:34.05
やたらと覚えとかなきゃならん事が多いのは事実。
あと、はさみうちに出てくる、こんなの気付かないよ!っていう比較対象はテーラー展開使ってもってきてる事が多いね。
254132人目の素数さん:2012/04/24(火) 21:36:23.09
>>250
たいていは昔の天才がひねり出したモン
思いつくのはほとんど不可能
255132人目の素数さん:2012/04/24(火) 22:33:28.34
今日高校のテストで出て、問題用紙が回収されちゃったので文章曖昧ですが(´・_・`)

赤、青、黄、緑のボールがたくさんあります。この中から15個取ります(並べないので順番は関係ないです)。
どの色も最低でも1個取るとしたら、取り方は何通りでしょう?


4つのボールは確定に取るから全部で11個と考えて、残りは4通り×4通り×…で
4^11と思ったんですけどこれは違うらしいです

もうひとつ、
14C3っていう解き方が正解らしくて、
11個のボールと3つの仕切りで考えるっていうことまでは理解できたのになんでCを使うのかが分かりません

14! だったら理解できるんですが…



文章長くてすみません
解説して下さる方いたらお願いします。

256132人目の素数さん:2012/04/24(火) 22:38:22.96
簡単な問題ですが、よろしくおねがいします. 
6人がけの円卓に大人用子供用合わせて6コの食器を並べる
(1)大人用4コ、子供用2コの時の並べる方法は何通りか。
(2)大人用3コ、子供用3コの時の並べる方法は何通りか。

(1)を大人用の円順列3!*間に入れる子供用4P2で
区別をなくして1/4と1/2をかけてのですが答え(3通り)に合いません。
(2)も同じです(正答は4通り)

よろしくおねがいします。
257132人目の素数さん:2012/04/24(火) 22:39:24.47
無職透明なボールが11個ある
○○○○○○○○○○○

これを四つの色のエリアに分ける
たとえば下のように

赤   青    黄 緑
○○|○○○|○|○○○○○

そして区分ごとに あとから 色をペンキやらで塗る
258132人目の素数さん:2012/04/24(火) 22:39:26.43
例えば仕切りにA、B、Cと名前をふったとき
赤A青B黄C緑緑 緑緑緑緑緑 緑緑緑緑緑 と
赤B青A黄C緑緑 緑緑緑緑緑 緑緑緑緑緑 と
各色のボールの個数は結局同じだからな
259132人目の素数さん:2012/04/24(火) 22:45:25.75
>>257>>258

わああああ!
納得しました!
ありがとうございます

あと、どうして階乗のやり方だとだめなのかってわかりますか?
260132人目の素数さん:2012/04/24(火) 22:49:58.75
>>259
14!だとボールと仕切り合計14個全部に名前をふって並べた個数と同じ
でも結局区別は色だけでつけるから、重複が大量に出る

>>256
どうやって説明したものかわからんので、とりあえずAA並べてみる
   ●     |     ●     |     ●
○    ●  |  ○    ○  |  ○    ○
○    ○  |  ○    ●  |  ○    ○
   ○     |     ○     |     ●


   ●     |     ●     |     ●     |     ●
○    ●  |  ○    ●  |  ○    ●  |  ○    ○
○    ●  |  ○    ○  |  ●    ○  |  ●    ●
   ○     |     ●     |     ○     |     ○
261132人目の素数さん:2012/04/24(火) 22:50:14.72
あ、階乗だと並べる問題になってしまうからってことですかね?
262132人目の素数さん:2012/04/24(火) 22:53:33.48
>>260
書き間違えてました
11!だとどうなりますか?
263132人目の素数さん:2012/04/24(火) 22:59:43.35
結局重複でますね!
ひとりでたくさん書き込んじゃいましたけど解決しました(^ω^)

教えてくれたみなさんありがとうございました!
264132人目の素数さん:2012/04/24(火) 23:19:21.16
結局同じ事だけど14!/(11!(○だけ並べた)3!(|だけ並べた))でも出るんだぜ?
265132人目の素数さん:2012/04/24(火) 23:30:27.27
(1)大人用4コ、子供用2コの時の並べる方法は何通りか。6!/4!2!6
(2)大人用3コ、子供用3コの時の並べる方法は何通りか。 6!/3!3!6
266132人目の素数さん:2012/04/24(火) 23:38:36.80
>>256
(1)
まず、4つの大人用食器を置く。
次に、大人用の2つの間に子供用の1つを置く置き方を調べる。
残りの1つを、最初に置いた子供のと同じ間に置く。1通り。
異なる間におく。2通り。(残った隙間に左周りに1,2,3と名前を付けると、1に置くのと3に置くのは同じ置き方。)
よって、答えは3

(2)
まず、3つの大人用食器を置く。
大人用の食器の隙間は3つあるから、その3つの隙間に3つの子供用を置くとき
1つの隙間に3つの子供用食器を置く。入れ方は1通り
1つの隙間に2つの子供用食器を置く。残り1つの入れ方は2通り。
どの隙間にも1つの子供用食器を置く。入れ方は1撮り。
よって、答は4。
267132人目の素数さん:2012/04/25(水) 00:24:12.89
素数について質問
ぐぐってみると素数の定義で、「1より大きく、約数が〜」または「1以外で、約数が〜」
と1が素数でないことは注釈的な書き方をされてるものばかりですが

素数の定義としてズバリ「約数が2個である自然数」としたらいけない理由でもあるんですか?
この書き方のほうがスマートだと思うんですが・・・
268132人目の素数さん:2012/04/25(水) 00:25:28.78
いいよ
269132人目の素数さん:2012/04/25(水) 00:34:57.66
1 = 1*1*1
1 = 1*1*1*1*1*1
1 = 1*1*1*1*1*1*1*1*1*1

先人はただしい
1には注釈を付けろ
270132人目の素数さん:2012/04/25(水) 00:40:45.06
簡単な問題ですみませんが
サッパリわかりません
解説お願いします<(_ _)>

http://www.dotup.org/uploda/www.dotup.org2900118.jpg
http://www.dotup.org/uploda/www.dotup.org2900120.jpg
http://www.dotup.org/uploda/www.dotup.org2900125.jpg
271132人目の素数さん:2012/04/25(水) 00:43:44.89
P=P'Q
Qは1次式
272132人目の素数さん:2012/04/25(水) 00:53:48.04
こうも宿題丸投げくさいのも久々だなwww
273132人目の素数さん:2012/04/25(水) 00:54:44.62
>>270
宿題はサッパリ判らない学生を鍛えるためにあるのだよ。
ガンバレ。未来はキミのモノだ。
274132人目の素数さん:2012/04/25(水) 00:56:39.60
ところで、サッパリ分かりませんでした。では何でいけないんだ?バカなんだから仕方ないだろ。
275132人目の素数さん:2012/04/25(水) 01:01:22.18
Q=a(x-1)^2+b(x-1)+c
f(x)+8=(x+1)^3Q
=(x-1+2)^3Q
=(x-1)^3+6(x-1)^2Q+12(x-1)Q+8Q
=(x-1)^3+6(x-1)^2(a(x-1)^2+b(x-1)+c)+12(x-1)(a(x-1)^2+b(x-1)+c)+8(a(x-1)^2+b(x-1)+c)

6c(x-1)^2+12b(x-1)^2+12c(x-1)+8(a(x-1)^2+b(x-1)+c)-16
=(8a+12b+6c)x^2+(-16a-16b)x+8a+4b+2c-16=0
8a+12b+6c=0
-16a-16b=0
8a+4b+2c-16=0
a=2, b=-2, c=4/3
276132人目の素数さん:2012/04/25(水) 01:01:33.18
極限は理系じゃなかったら面倒臭い分野だよね〜
277132人目の素数さん:2012/04/25(水) 01:05:20.80
g(x)=x^2f(x)
x^2-a^2=(x-a)(x+a)
278132人目の素数さん:2012/04/25(水) 01:06:59.27
>>274
ぜんぜんいけなくないよ。
サッパリ判らない学生を鍛えるのが宿題なんだよ。
判っている学生は、簡単に答を出せるんだ、もしかしたら、信じられないかもしれないが。
279132人目の素数さん:2012/04/25(水) 01:08:49.20
>>270 3つ目
  分子 = x^2 f(x) + a^2 f(x)− a^2 f(x) − a^2 f(a)
と, a^2 f(x)を足して引く
このあと,微分係数の定義式の形が出てくるように整理
280132人目の素数さん:2012/04/25(水) 01:11:04.09
>>279
順番を間違えた
  分子 = x^2 f(x) − a^2 f(x) + a^2 f(x) − a^2 f(a)
281132人目の素数さん:2012/04/25(水) 01:38:19.68
>>270
何も書いてない問題用紙をアップして、サッパリわかりませんとか・・・
それじゃいつまでたってもダメだろうね。
最後の問題は確かに発想必要かもしれんけど、
最初の二問は文字で置くなりして表すことぐらいできるでしょ。


>>280
ちなみに
分子 = x^2 f(x) − x^2 f(a) + x^2 f(a) − a^2 f(a)
でも大丈夫だよね?
282132人目の素数さん:2012/04/25(水) 01:49:04.88
2点 A(2,0) B(−2,0) を直径とする円周の上半分の弧上に2点P,Qをとる。
原点をOとし、∠AOP=2θ,∠AOP+∠QOB= π / 2 とする。ただし、0 < θ < π / 4 とする 。
三角形ABPの面積が四角形APQBの面積の半分になるとき、sin2θ の値を求めよ。

合っているかわかりませんが、四角形APQBの面積は 2sin2θ+2cos2θ+2、三角形APBの面積は 4sin2θ
というところまでわかって、条件より
8sin2θ=2sin2θ+2cos2θ+2
という等式を作ったんですが、ここからsin2θの求め方がわかりません。
教えてください。
283132人目の素数さん:2012/04/25(水) 02:16:21.06
>>282
cos(2θ)= c , sin(2θ)= s とおくと,その最後の式は
   −c + 3s = 1
となる これを cs 平面の直線の式と見る
cs 平面の単位円との共有点の s 座標が求めるものである
284132人目の素数さん:2012/04/25(水) 02:19:05.33
cosとsinの二乗話が一になること使おうよ
285132人目の素数さん:2012/04/25(水) 02:39:08.71
>>283,284
わかりません
286132人目の素数さん:2012/04/25(水) 02:56:04.91
>>285

8sin2θ=2sin2θ+2cos2θ+2
を整理して
cos2θ=3sin2θ-1

これを
(sin2θ)^2+(cos2θ)^2=1に代入
287132人目の素数さん:2012/04/25(水) 06:21:43.29
          __ノ)-'´ ̄ ̄`ー- 、_
        , '´  _. -‐'''"二ニニ=-`ヽ、
      /   /:::::; -‐''"        `ーノ
     /   /:::::/           \
     /    /::::::/          | | |  |
     |   |:::::/ /     |  | | | |  |
      |   |::/ / / |  | ||  | | ,ハ .| ,ハ|
      |   |/ / / /| ,ハノ| /|ノレ,ニ|ル' 
     |   |  | / / レ',二、レ′ ,ィイ|゙/   私は只の数ヲタなんかとは付き合わないわ。
.     |   \ ∠イ  ,イイ|    ,`-' |      頭が良くて数学が出来てかっこいい人。それが必要条件よ。
     |     l^,人|  ` `-'     ゝ  |        さらに Ann.of Math に論文書けば十分条件にもなるわよ。
      |      ` -'\       ー'  人          一番嫌いなのは論文数を増やすためにくだらない論文を書いて
    |        /(l     __/  ヽ、           良い論文の出版を遅らせるお馬鹿な人。
     |       (:::::`‐-、__  |::::`、     ヒニニヽ、         あなたの論文が Ann of Math に accept される確率は?
    |      / `‐-、::::::::::`‐-、::::\   /,ニニ、\            それとも最近は Inv. Math. の方が上かしら?
   |      |::::::::::::::::::|` -、:::::::,ヘ ̄|'、  ヒニ二、 \
.   |      /::::::::::::::::::|::::::::\/:::O`、::\   | '、   \
   |      /:::::::::::::::::::/:::::::::::::::::::::::::::::'、::::\ノ  ヽ、  |
  |      |:::::/:::::::::/:::::::::::::::::::::::::::::::::::'、',::::'、  /:\__/‐、
  |      |/:::::::::::/::::::::::::::::::::::::::::::::::O::| '、::| く::::::::::::: ̄|
   |     /_..-'´ ̄`ー-、:::::::::::::::::::::::::::::::::::|/:/`‐'::\;;;;;;;_|
   |    |/::::::::::::::::::::::\:::::::::::::::::::::::::::::|::/::::|::::/:::::::::::/
    |   /:::::::::::::::::::::::::::::::::|:::::::::::::::::::::O::|::|::::::|:::::::::::::::/
288132人目の素数さん:2012/04/25(水) 07:34:00.27
留学生の集まりがあった。集まった留学生が話す言語について次のことが分かっている。
 ・どの留学生も、高々4人の留学生と直接会話ができる。
 ・どの2人の留学生についても、直接会話できるか、1人の“通訳”を介して会話できる。
 (例えば、留学生Aはp語を話せるがq語を話せず、留学生Bはq語を話せるがp語を話せないとき
 p語とq語を共に話せる留学生Cがいるなら、Cを介してAとBは会話ができるものとする。)
このとき、集まった留学生の人数として考えられる最大値はいくらか。

Aと直接会話ができる留学生をD_1, D_2, D_3, D_4 とし、通訳を介して会話できる人をE_1, E_2, ・・・, E_n
とすると、E_1,・・・,E_nはD_1, D_2, D_3, D_4のいずれかとは直接会話ができるので n≦12が必要
ということは分かるのです(つまり答えは17以下)が、「17人」の例は実際可能でしょうか。
正しい答えをご教授ください。
289132人目の素数さん:2012/04/25(水) 07:50:48.47
290132人目の素数さん:2012/04/25(水) 08:26:17.55
          __ノ)-'´ ̄ ̄`ー- 、_
        , '´  _. -‐'''"二ニニ=-`ヽ、
      /   /:::::; -‐''"        `ーノ
     /   /:::::/           \
     /    /::::::/          | | |  |
     |   |:::::/ /     |  | | | |  |
      |   |::/ / / |  | ||  | | ,ハ .| ,ハ|
      |   |/ / / /| ,ハノ| /|ノレ,ニ|ル' 
     |   |  | / / レ',二、レ′ ,ィイ|゙/   私は只の数ヲタなんかとは付き合わないわ。
.     |   \ ∠イ  ,イイ|    ,`-' |      頭が良くて数学が出来てかっこいい人。それが必要条件よ。
     |     l^,人|  ` `-'     ゝ  |        さらに Ann.of Math に論文書けば十分条件にもなるわよ。
      |      ` -'\       ー'  人          一番嫌いなのは論文数を増やすためにくだらない論文を書いて
    |        /(l     __/  ヽ、           良い論文の出版を遅らせるお馬鹿な人。
     |       (:::::`‐-、__  |::::`、     ヒニニヽ、         あなたの論文が Ann of Math に accept される確率は?
    |      / `‐-、::::::::::`‐-、::::\   /,ニニ、\            それとも最近は Inv. Math. の方が上かしら?
   |      |::::::::::::::::::|` -、:::::::,ヘ ̄|'、  ヒニ二、 \
.   |      /::::::::::::::::::|::::::::\/:::O`、::\   | '、   \
   |      /:::::::::::::::::::/:::::::::::::::::::::::::::::'、::::\ノ  ヽ、  |
  |      |:::::/:::::::::/:::::::::::::::::::::::::::::::::::'、',::::'、  /:\__/‐、
  |      |/:::::::::::/::::::::::::::::::::::::::::::::::O::| '、::| く::::::::::::: ̄|
   |     /_..-'´ ̄`ー-、:::::::::::::::::::::::::::::::::::|/:/`‐'::\;;;;;;;_|
   |    |/::::::::::::::::::::::\:::::::::::::::::::::::::::::|::/::::|::::/:::::::::::/
    |   /:::::::::::::::::::::::::::::::::|:::::::::::::::::::::O::|::|::::::|:::::::::::::::/
291132人目の素数さん:2012/04/25(水) 11:02:25.42
とても簡単な質問で申し訳ない。

ある数xに31をかけ、それを100で割ったときの余りを、どう数式で表現できますか?
なぜかパッと出てこない。
292132人目の素数さん:2012/04/25(水) 11:09:54.42
xが非負の整数なら
(31x/100-[31x/100])*100
負の場合どうだったっけかな
http://www.wolframalpha.com/input/?i=%2831x%2F100-int%5B31x%2F100%5D%29*100+from+x%3D-7+to+7
293132人目の素数さん:2012/04/25(水) 11:30:10.36
すいません、[]の演算はどうすればいいのですか?
ネットで数式に関する質問をするのは初めてなので、重ね重ね申し訳ない……
294132人目の素数さん:2012/04/25(水) 12:13:56.09
ttp://ja.wikipedia.org/wiki/%E5%BA%8A%E9%96%A2%E6%95%B0%E3%81%A8%E5%A4%A9%E4%BA%95%E9%96%A2%E6%95%B0

[x] は床関数

プログラミングで言えばfloor(x)
ついでにciel(x)も調べとけ
295132人目の素数さん:2012/04/25(水) 12:38:29.85
>>291
ようは想像しているような、わかりやすい数式で簡単に表現できる方法はないな。

31x=100q+rただしqは整数かつ0≦r<100のrを余りとするとかそんな程度
296132人目の素数さん:2012/04/25(水) 14:55:54.57
みなさんありがとうございます。
ちょっと調べてみます。
297132人目の素数さん:2012/04/25(水) 21:22:56.76
x^2−2x+5の解を計算すると1±2iになる時と1±4iがあるんですが正しいのはどちらでしょうか?
298132人目の素数さん:2012/04/25(水) 21:24:16.49
>>297
代入して検算
299132人目の素数さん:2012/04/25(水) 21:35:28.74
y=ax^2+bx+cが点(1,-3)を通り点(2,6)においてy=x^3+dx
と共通の接線を持つとき定数a,b,c,dの値を求めよ

という問題で代入して地道に解くよりも簡単に解く方法があったのですが
誰かわかりませんか?
300132人目の素数さん:2012/04/25(水) 21:45:28.64
>>297
整式に解なんか無い
301132人目の素数さん:2012/04/25(水) 21:53:38.43
そこは察しろよ
302132人目の素数さん:2012/04/25(水) 21:59:48.35
これは√(-4)が何になるかわからないって事か?
整式に解とか言ってるからこういう基礎が抜けているんだろうな。教科書の複素数の導入部分読んだ方がいいぞ。
303132人目の素数さん:2012/04/25(水) 22:07:04.34
6x^2+x−2がたすき掛けでは解けるのに解の公式を使うと解けない様になってるのは何故ですか?
304132人目の素数さん:2012/04/25(水) 22:07:49.14
どういうときにふたつのこたえがでてくるのだろう?
305288:2012/04/25(水) 22:09:55.21
>>289
ありがとうございます。
僕英語全然読めないんですが、指示されたサイトの論文によると
モーレのグラフというのが関係するんでしょうか。
9ページ(1)を指示されたということは、答は「15人」ということでしょうか。ということは17や16ではダメだということがこの前か後に述べられているんでしょうか。
306132人目の素数さん:2012/04/25(水) 22:25:42.44
>>303
計算間違い
307132人目の素数さん:2012/04/25(水) 22:26:08.38
>>303
気のせいです

x=(-1±√25)/12=-(1/2),1/3
308132人目の素数さん:2012/04/25(水) 22:29:43.40
>>306
>>307
有難う
309132人目の素数さん:2012/04/25(水) 23:29:13.72
a,bは整数で、0でないとする。また、x、yを任意の整数とするとき
ax+by全体の集合をM、整数全体の集合をNとする。
N=Mが成り立つことをつかって、以下を証明せよ。

方程式ax+by=1は整数解x、yをもつ→a、bは互いに素

ax+by=g(a´x+by´)=1

よってg=1
となっているんですがわかりません・・・

理由を教えてくだい。
お願いします
310132人目の素数さん:2012/04/25(水) 23:32:38.55
1の約数
311132人目の素数さん:2012/04/25(水) 23:49:19.36
>>309
a,bが共通素因数p>1を持つと仮定すると
a=pA,b=pB(A,Bは互いに素)とかけて
与方程式はp(Ax+By)=1
つまりAx+By=1/pとなるが
A,B,x,yは整数なのに対し右辺は整数にはならない

背理法によりa,bは互いに素
312132人目の素数さん:2012/04/25(水) 23:51:19.95
>>309
>>1
>   (変に省略するより全文書いた方がいい、また説明なく習慣的でない記号を使わないように)
313132人目の素数さん:2012/04/25(水) 23:54:06.59
>>307
ちょっとまて
314132人目の素数さん:2012/04/25(水) 23:54:50.21
>>310
>>312
ありがとうございます
これからは気をつけます
315132人目の素数さん:2012/04/26(木) 00:01:12.22
N=Mが成り立つ
ということは
ax+by=1を満たす整数解x,yが存在する。
この時a,bの最大公約数をgとすると
ax+by=g(a´x+by´)=1
g×(整数)=1だからg=1
よってa,bは互いに素
316132人目の素数さん:2012/04/26(木) 00:02:16.37
二項定理を用いてy=x^nの導関数を求めよ

解き方が全く分かりません><お願いします
317132人目の素数さん:2012/04/26(木) 00:04:12.83

メディアは全部抑えてある。教団名を出したり逆らえばどうなるかわかってるだろうな?
<           _-=≡:: ;;   ヾ\            >
<         /          ヾ:::\           >
<         |            |::::::|          >
<        ミ|-=≡、 ミ≡==- 、 |;;;;;/          >
<         || <●>| ̄| <◎> |── /\         >
<         |ヽ_/  \_/    > /         >
<        / /(    )\      |_/          >
<        | |  ` ´        ) |           >
<        | \/ヽ/\_/  /  |           >
<        \ \ ̄ ̄ /ヽ  /  /           >
<          \  ̄ ̄   /  /       \    >
  / /        ̄ ̄ ̄ ̄ ̄ ̄ ̄     \\ \ \
     ___
   / ー\ ナンミョウホウレンゲッキョウナンミョウホウレンゲッキョウナンミョウホウレンゲッキョウ
 /ノ  (@)\ ナンミョウホウレンゲッキョウナンミョウホウレンゲッキョウナンミョウホウレンゲッキ
.| (@)   ⌒)\ ナンミョウホウレンゲッキョウナンミョウホウレンゲッキョウナンミョウホウレンゲッ
.|   (__ノ ̄|  |   ///;ト,  ナンミョウホウレンゲッキョウナンミョウホウレンゲッキョウナンミョ
 \   |_/  / ////゙l゙l;  ナンミョウホウレンゲッキョウナンミョウホウレンゲッキョウナンミョ
   \     _ノ   l   .i .! |  ナンミョウホウレンゲッキョウナンミョウホウレンゲッキョウナンミョ
   /´ 公明  `\ │   | .|  ナンミョウホウレンゲッキョウナンミョウホウレンゲッキョウナンミョ
318132人目の素数さん:2012/04/26(木) 00:09:59.25
>>316
まず導関数の定義を書いて、
それからそこに x^n を当てはめてみる。
319132人目の素数さん:2012/04/26(木) 00:12:22.15
>>316
微分係数ってどうやって求めるんだっけ?
320132人目の素数さん:2012/04/26(木) 00:14:58.70
>>319
>>1
>・まずは教科書、参考書、web検索などで調べるようにしましょう。(特に基本的な公式など)
321132人目の素数さん:2012/04/26(木) 00:19:42.52
>>3にあるじゃんw
322132人目の素数さん:2012/04/26(木) 00:29:08.31
>>320
>>319はヒントとしていっているのだと思うが。
323132人目の素数さん:2012/04/26(木) 00:34:02.41
√x+√y≦k√(x+y)
が任意の正の数x,yに対して成り立つような実数kの値の範囲を求めよ。

k>0はわかりました。
そこから2乗して、相加相乗平均使うんだろうなと
思ってやってみましたがうまくいきませんでした。
解説お願いします。
324132人目の素数さん:2012/04/26(木) 00:36:27.19
>>323
x=a^2, y=b^2 と置いた方が見通しがいいかもしれない
325132人目の素数さん:2012/04/26(木) 00:43:09.92
>>324
置いてみましたがわかりません。。。
326132人目の素数さん:2012/04/26(木) 00:45:00.56
数三習ってるならx/y=tと置いたら分かりやすいかも
327132人目の素数さん:2012/04/26(木) 00:45:19.95
>>323
>√x+√y≦k√(x+y)

>そこから2乗して、相加相乗平均使うんだろうなと
>思ってやってみましたが

何をどう二乗したんだかいてくれ
328132人目の素数さん:2012/04/26(木) 00:50:50.14
>>327
両辺をそれぞれ2乗しますた
329132人目の素数さん:2012/04/26(木) 00:56:44.26
a^2 - ab + b^2 ≧ 0 a,b;実数
くらいは示せるだろ。
それと同じ要領

相加平均相乗平均は使わない。
330132人目の素数さん:2012/04/26(木) 01:03:43.78
k≧√2
331132人目の素数さん:2012/04/26(木) 01:07:12.27
>>323
相加相乗でもできる
与式を k ≧ … と整理して両辺2乗
この式の右辺の最大値は相加相乗の不等式から考えることができる

本問は別解が多いことでも有名
K出版の少し前の某問題集には6つの別解が出ていた
332132人目の素数さん:2012/04/26(木) 01:08:14.80
相加相乗使って色々やってたらいけました。どーもです。
>>329
示せるけど、それをどう使えと?
333132人目の素数さん:2012/04/26(木) 01:10:44.65
x=yでk≧√2が必要
あとはこのとき示せばok
334132人目の素数さん:2012/04/26(木) 01:14:01.91
>>328
グダグダ言ってねーで
さっさと数式書けよ
アホちゃうか?
オメーの二乗の仕方が間違ってるからダメダって言ってんだよ
335132人目の素数さん:2012/04/26(木) 01:45:57.56
          __ノ)-'´ ̄ ̄`ー- 、_
        , '´  _. -‐'''"二ニニ=-`ヽ、
      /   /:::::; -‐''"        `ーノ
     /   /:::::/           \
     /    /::::::/          | | |  |
     |   |:::::/ /     |  | | | |  |
      |   |::/ / / |  | ||  | | ,ハ .| ,ハ|
      |   |/ / / /| ,ハノ| /|ノレ,ニ|ル' 
     |   |  | / / レ',二、レ′ ,ィイ|゙/   私は只の数ヲタなんかとは付き合わないわ。
.     |   \ ∠イ  ,イイ|    ,`-' |      頭が良くて数学が出来てかっこいい人。それが必要条件よ。
     |     l^,人|  ` `-'     ゝ  |        さらに Ann.of Math に論文書けば十分条件にもなるわよ。
      |      ` -'\       ー'  人          一番嫌いなのは論文数を増やすためにくだらない論文を書いて
    |        /(l     __/  ヽ、           良い論文の出版を遅らせるお馬鹿な人。
     |       (:::::`‐-、__  |::::`、     ヒニニヽ、         あなたの論文が Ann of Math に accept される確率は?
    |      / `‐-、::::::::::`‐-、::::\   /,ニニ、\            それとも最近は Inv. Math. の方が上かしら?
   |      |::::::::::::::::::|` -、:::::::,ヘ ̄|'、  ヒニ二、 \
.   |      /::::::::::::::::::|::::::::\/:::O`、::\   | '、   \
   |      /:::::::::::::::::::/:::::::::::::::::::::::::::::'、::::\ノ  ヽ、  |
  |      |:::::/:::::::::/:::::::::::::::::::::::::::::::::::'、',::::'、  /:\__/‐、
  |      |/:::::::::::/::::::::::::::::::::::::::::::::::O::| '、::| く::::::::::::: ̄|
   |     /_..-'´ ̄`ー-、:::::::::::::::::::::::::::::::::::|/:/`‐'::\;;;;;;;_|
   |    |/::::::::::::::::::::::\:::::::::::::::::::::::::::::|::/::::|::::/:::::::::::/
    |   /:::::::::::::::::::::::::::::::::|:::::::::::::::::::::O::|::|::::::|:::::::::::::::/
336132人目の素数さん:2012/04/26(木) 05:57:02.95
337132人目の素数さん:2012/04/26(木) 06:22:51.87
とりあえずkはいらないと思う
平行なら(x成分/y成分)が=で結べる
338132人目の素数さん:2012/04/26(木) 06:28:02.85
こたえるなら解いてからにしたら
339132人目の素数さん:2012/04/26(木) 06:31:01.41
というかあってるよ。綺麗な値にならなくて不安なのかもしれんが。適当に作ってある問題なんかそんなもんだよ。
340132人目の素数さん:2012/04/26(木) 06:33:39.11
え、これ答えが-1+2√14、-5+2√14と
-1-2√14、-5-2√14となったのですがよろしいでしょうか??
341132人目の素数さん:2012/04/26(木) 06:36:54.97
うん。自分でx+bの長さ確認してごらん。12になってあってるから
342132人目の素数さん:2012/04/26(木) 06:40:41.66
大変感謝しております。ありがとうございました。
343132人目の素数さん:2012/04/26(木) 06:42:32.54
これ長さが12じゃなくて8だったら整数値になるな。
あと、計算する時だけど(2k+10,2k+2)をそのままやり始めないで
2(k+5,k+1)が12になるって考えると係数が大きくならなくてすむよ
344132人目の素数さん:2012/04/26(木) 11:09:32.07
2点A(0、1) B(1、1)を結ぶ線分ABが円x^2+y^2-2ax-2by-1=0の外部にあるときa、bの満たす条件が表す領域をab平面に図示せよ
という問題で、線分上の点を(t、1)(0<t<1)とおいて
t^2+1^2-2at-2b•1-1>0
がなりたてばよい
となっているのですが、この>0というのは何を表しているのでしょうか
よろしくお願いします
345132人目の素数さん:2012/04/26(木) 11:27:21.42
>>344
円の方程式は基本的には円上の点と中心との距離が一定という式。
それをバラして移行すると=0という式に出来るが、その問題ではこの形で与えられている。

一方、円の外部にある点は、中心との距離が一定よりも大きいという不等式で表される。
これをバラして移行すると>0という不等式が出来るが、
このときの左辺は円の方程式を=0の形にしたときの左辺と同じ。
346132人目の素数さん:2012/04/26(木) 11:29:49.85
>>345
よくわかりました!!
ありがとうございました^^
347132人目の素数さん:2012/04/26(木) 12:51:01.60
アンカつけずにだらだら受け答えしてんじゃねーよバカども>>337-343
348132人目の素数さん:2012/04/26(木) 13:02:12.84
行火?
349132人目の素数さん:2012/04/26(木) 13:49:40.63
Σ[k=1,n](k・2^k)の数列の和の解き方がわかりません。
kに代入していくと、等差数列×等比数列の和になっているのがわかったんですが、求め方を教えてくださいm(_ _)m
350132人目の素数さん:2012/04/26(木) 13:51:18.62
>>349
確かめてないけど、等比数列の和をどうやって求めたかを考えればわかる気がする。
351132人目の素数さん:2012/04/26(木) 13:53:46.44
>>349
> kに代入していくと、等差数列×等比数列の和になっているのがわかった
チャチャ入れだけですまんが、それは代入しなくてもわかるんじゃマイカ?
Σ[k=1,n](k・2^k)ってそれそのものだろ?

たぶん、すでに書かれているように等比数列の和の求め方と似たように出来るんじゃないだろうか。
352132人目の素数さん:2012/04/26(木) 15:09:04.33
昨日、整数の余りに関する質問をした者です。
その節はどうもありがとうございました。

その質問に関連してもう一つ尋ねたいのですが、「0から99の数にそれぞれ31をかけて、さらにそれを100で割ると、その余りは必ず0〜99の範囲で散らばる」……このことを、数学的に証明するにはどういった手法が必要でしょうか?
(昨日からお気づきの方も多いと思いますが、ある簡単なハッシュ関数を作っているところで発生している疑問です)
353132人目の素数さん:2012/04/26(木) 15:11:32.37
>>352
証明も何も、余りの定義ですよそれ
100で目一杯割って残ったものが「余り」なんだから、余りは100より小さいものと定義するべき
354132人目の素数さん:2012/04/26(木) 15:13:24.99
A〜Fの6人がトーナメント(勝ち抜き戦)を戦う。シードか否かの場所はクジ引きで決めるとして
Aが優勝する確率を考える際
 ・どの二人の対戦でも勝つ確率が対等の場合 
は1/6 になるのは明らかですが、これが
 ・Aと、他のどの5人との対戦でも勝つ確率が対等 としか分かっていない場合
(つまり BvsC や DvsF の勝つ確率の対等性が保証されていない場合) でも、
Aが優勝する確率は1/6といえるでしょうか。
355132人目の素数さん:2012/04/26(木) 15:14:26.42
>>349
ふつうの解法は上で他の方がおっしゃったとおりだが
「Σの中身を階差の形に整理」する手もある
  k・2^k = ( pk + q )・2^k − { p( k - 1 )+ q }・2^( k-1 )
が恒等式となるように定数 p , q を決定する
階差の形になれば和はすぐに求まる
356132人目の素数さん:2012/04/26(木) 15:27:18.39
0・31,1・31,…,i・31,…,j・31,…,99・31を100で割った余りは異なることを示す

0≦i<j≦99でi・31とj・31を100で割った余りが等しいと仮定すると
j・31-i・31=(j-i)・31,(1≦j-i≦99)は100で割り切れる
だが(j-i)・31は1≦j-i≦99より明かに100で割り切れないから矛盾

よって割った余りは互いに異なる
357132人目の素数さん:2012/04/26(木) 15:45:32.66
>>354
いえる
優勝に影響するパラメータはシード/非シード率と勝率だけだし


けど、それが答えだと何かおかしな問題だよね
358132人目の素数さん:2012/04/26(木) 15:54:46.86
次の2次方程式をとけ
(1)x2-4=0
(2)x2-3x-4-0
(3)x2+8x+16=0
(4)x2+3x-6=0
359132人目の素数さん:2012/04/26(木) 15:58:47.78
2次方程式を解くのに何か工夫することなんてあるか?
360354:2012/04/26(木) 16:05:52.31
>>357 ありがとうございます。

>それが答えだと何かおかしな問題だよね
どこかの入試問題とかじゃなくて、ふと自分で思いついた疑問だったので。お騒がせしました。
361132人目の素数さん:2012/04/26(木) 16:06:15.10
>>353
例えば最初にかける数が31でなく30なら重複するあまりが出てきてしまうし、これをどう説明すれば……と思いまして。
>>356
ありがとうございます、納得しました。
362片山博文MZ ◆0lBZNi.Q7evd :2012/04/26(木) 16:08:49.80
>>358
(1) x=±4 (2) x=-1,4 (3) x=-4(重解) (4) x=(-3±√33)/2
363132人目の素数さん:2012/04/26(木) 18:09:45.39
6x^2+x−2が解の公式使って解けませんでした、これ解の公式で解けるんですか?
解けるとしたら途中式を詳しく書いてくれたら嬉しいです
364132人目の素数さん:2012/04/26(木) 18:14:49.12
因数分解の問題二つお願いします

(1) (a-b)^2−ac+bc=

(2) (x^2+y^2−z^2)^2−4x^2y^2=

です
解答はあるのですが途中式が載っていなくて自分でとけません
因数分解の公式のどれを使うのでしょうか
365132人目の素数さん:2012/04/26(木) 18:20:17.61
>>364
分かりません
366132人目の素数さん:2012/04/26(木) 18:37:36.98
因数分解なんて公式使ってやるもんじゃない
367132人目の素数さん:2012/04/26(木) 18:41:07.89
>>363
解の公式で解けない二次方程式があったら、世紀の大発見だな。

解の公式のabcにあたる数字はその方程式だと何になんの?
368132人目の素数さん:2012/04/26(木) 18:44:34.88
>>366
>>367
たすき掛けでは解けるのです。しかし解の公式では解けません。
−1±√49/12になってしまいます。
369132人目の素数さん:2012/04/26(木) 18:45:56.78
>>368
そりゃ大変だな
370132人目の素数さん:2012/04/26(木) 18:46:39.72
>>364
(1) (a-b)^2−ac+bc=(a-b)^2-c(a-b)
=(a-b)(a-b-c)

(2) (x^2+y^2−z^2)^2−4x^2y^2=x^4+y^4+z^4+2x^2 y^2 -2x^2 z^2 -2y^2 z^2 -4x^2 y^2
=x^4+y^4+z^4-2x^2 y^2 -2x^2 z^2 -2y^2 z^2
=(x^2-y^2-z^2)^2
371132人目の素数さん:2012/04/26(木) 18:47:36.16
>>370
ひでえ
372132人目の素数さん:2012/04/26(木) 18:48:22.41
6x^2+x−2が解の公式使って解けません!!
373132人目の素数さん:2012/04/26(木) 18:49:01.08
>>368
これはマジレスしてあげたほうがいいの?
374132人目の素数さん:2012/04/26(木) 18:49:59.82
>>373
お願いします
375132人目の素数さん:2012/04/26(木) 18:50:29.08
ネタにマジレスするとか無粋だからやめよう
376132人目の素数さん:2012/04/26(木) 18:50:34.42
√49=7
377132人目の素数さん:2012/04/26(木) 18:53:48.06
>>376
見落としていました……
どうやら疲れ過ぎていた様です…有難うございます
378132人目の素数さん:2012/04/26(木) 19:14:53.25
暖かくして休めwww
379132人目の素数さん:2012/04/26(木) 19:22:31.47
>>370
ありがとうございます

ただ、(2)の答えが解答では
(x+y+z)(x+y-z)(x-y+z)(x-y-z)になってます
380132人目の素数さん:2012/04/26(木) 19:30:14.97
>>364 (2)
A^2 - B^2 = ( A + B ) ( A - B )
の形を見抜く 必要なら置き換え等も活用せよ
381132人目の素数さん:2012/04/26(木) 20:50:56.34
連立方程式の復習で
x-y=-1
x^2+y^2=25
という問題なのですが、上の式をy=の形に直して、下の式を(x+y)^2=25にして
代入する方法であってるでしょうか?
よろしくお願いします。
382132人目の素数さん:2012/04/26(木) 20:52:39.24
>>381
(x+y)^2=25

どっからでてきた?
383132人目の素数さん:2012/04/26(木) 20:52:49.59
>>381
合ってません。
384132人目の素数さん:2012/04/26(木) 20:53:37.99
(x+y)^2=x^2+y^2

ではない
385132人目の素数さん:2012/04/26(木) 20:58:09.57
具体的な数字なら計算できるのに、文字式になると計算できなくなる人って、
たぶん分配法則とかの演算法則を認識していないんだろうな
だから(間違った)勘で (x+y)^2=x^2+y^2 なのかと思ってしまう
386132人目の素数さん:2012/04/26(木) 21:08:05.31
┌─┐
│  │
└─┘
  +
┌──┐
│    │
│    │
└──┘
  ||
┌─┬──┐
│  │    │
├─┼──┤
│  │    │
│  │    │
└─┴──┘

何故これが成り立つと思うのか分からん
387132人目の素数さん:2012/04/26(木) 21:26:54.05
ヒント:標数2の体
388132人目の素数さん:2012/04/26(木) 21:48:19.04
すみません。案の定答えが出ませんでした。
x^2+y^2=25をどうやって処理すればいいかわからないです
389132人目の素数さん:2012/04/26(木) 21:57:53.32
上の式をy=の形にして下の式に代入すればいいだろ。
下の式の謎の変形とかどっから出てきたの?
390132人目の素数さん:2012/04/26(木) 21:58:19.26
>>388
x-y=-1 (1)
x^2+y^2=25 (2)

(1)を変形して
x= y-1とし
これを(2)へ代入します
(y-1)^2 + y^2 = 25
これを整理すればyの2次方程式が出ますので
それを解の公式なりなんなりすれば答えは出てくると思います
391132人目の素数さん:2012/04/26(木) 22:28:43.76
>>390 すみませんでした。ありがとうございます。
392132人目の素数さん:2012/04/26(木) 22:30:23.49
(x-y)^2+(x+y)^2=2(x^2+y^2)に与式を代入して1+(x+y)^2=50として解くこともできる
393132人目の素数さん:2012/04/26(木) 22:35:01.82
円と直線の交点
394 忍法帖【Lv=8,xxxP】 :2012/04/26(木) 22:35:50.58
なんにんじゃ
395132人目の素数さん:2012/04/26(木) 22:36:35.54
馬鹿忍者
396132人目の素数さん:2012/04/26(木) 22:38:34.00
高校数学じゃないと思うけど質問していいですか?
397132人目の素数さん:2012/04/26(木) 22:42:22.88
それを判断しかねるほどにその問題の見分けがつかないような頭脳をお持ちなんだろう、
書けよ
398132人目の素数さん:2012/04/26(木) 22:44:58.30
ゼータ関数の「場」って球体にできるんですか?
399132人目の素数さん:2012/04/26(木) 22:51:15.86
よし、スレチだ
400132人目の素数さん:2012/04/26(木) 22:52:33.43
絶対に数学とエスパー
401132人目の素数さん:2012/04/26(木) 23:05:29.48
時々話題になるな
>>396が本当に高校生相当ならスレタイが悪いと思うけどね
高校数学範囲の質問スレとかにしないのはなんでだろ
402132人目の素数さん:2012/04/26(木) 23:07:34.10
普通の日本の高校生ならどのスレで質問したらいいか分かるだろう
403132人目の素数さん:2012/04/26(木) 23:13:19.08
>>402
すいません高校生ですが分かりませんでした
404132人目の素数さん:2012/04/26(木) 23:17:15.89
>>398
ゼータ関数、場、球体の定義はなんだ?
405132人目の素数さん:2012/04/26(木) 23:18:04.80
どうかね
質問内容は大学以上だが回答が高校レベルであることを期待するなら
やっぱりこのスレになりがちなんじゃなかろうか
406132人目の素数さん:2012/04/26(木) 23:22:04.08
なんかゼータ関数のxを10P進法にすれば場が球体になって、ゼロ点が球の中心になってすべてのゼロ点が一直線上にあることが証明できるとか聞いたんですが・・・
高校生なんでよく分からないです
407132人目の素数さん:2012/04/26(木) 23:23:56.08
聞いた奴に聞け
408132人目の素数さん:2012/04/27(金) 00:23:13.21
p,qは実数とする。xの方程式|x^2+px+q|=|x^2+qx+p|の異なる実数解の個数をnとする
このとき、n=1となる点(p,q)の範囲を図示せよ

よろしくお願いします
409132人目の素数さん:2012/04/27(金) 00:26:59.79
p=qは適さないってすぐ分かるから
まずはp<qで考えるといいんじゃないかな
あとは頂点の位置でパターンを考えて
適切な場合分けを探す
410132人目の素数さん:2012/04/27(金) 00:33:35.37
2乗したら?
411132人目の素数さん:2012/04/27(金) 00:48:14.88
kを自然数としtanθ=1/2のときtan(kθ)≠0を証明せよ。

この問題で数学的帰納法でチャレンジしたら

tan((k+1)θ)=(2tan(kθ)+1)/(2-tan(kθ)

となり詰まりました。

他の証明法があるのでしょうか?
412132人目の素数さん:2012/04/27(金) 01:30:49.77
          __ノ)-'´ ̄ ̄`ー- 、_
        , '´  _. -‐'''"二ニニ=-`ヽ、
      /   /:::::; -‐''"        `ーノ
     /   /:::::/           \
     /    /::::::/          | | |  |
     |   |:::::/ /     |  | | | |  |
      |   |::/ / / |  | ||  | | ,ハ .| ,ハ|
      |   |/ / / /| ,ハノ| /|ノレ,ニ|ル' 
     |   |  | / / レ',二、レ′ ,ィイ|゙/   私は只の数ヲタなんかとは付き合わないわ。
.     |   \ ∠イ  ,イイ|    ,`-' |      頭が良くて数学が出来てかっこいい人。それが必要条件よ。
     |     l^,人|  ` `-'     ゝ  |        さらに Ann.of Math に論文書けば十分条件にもなるわよ。
      |      ` -'\       ー'  人          一番嫌いなのは論文数を増やすためにくだらない論文を書いて
    |        /(l     __/  ヽ、           良い論文の出版を遅らせるお馬鹿な人。
     |       (:::::`‐-、__  |::::`、     ヒニニヽ、         あなたの論文が Ann of Math に accept される確率は?
    |      / `‐-、::::::::::`‐-、::::\   /,ニニ、\            それとも最近は Inv. Math. の方が上かしら?
   |      |::::::::::::::::::|` -、:::::::,ヘ ̄|'、  ヒニ二、 \
.   |      /::::::::::::::::::|::::::::\/:::O`、::\   | '、   \
   |      /:::::::::::::::::::/:::::::::::::::::::::::::::::'、::::\ノ  ヽ、  |
  |      |:::::/:::::::::/:::::::::::::::::::::::::::::::::::'、',::::'、  /:\__/‐、
  |      |/:::::::::::/::::::::::::::::::::::::::::::::::O::| '、::| く::::::::::::: ̄|
   |     /_..-'´ ̄`ー-、:::::::::::::::::::::::::::::::::::|/:/`‐'::\;;;;;;;_|
   |    |/::::::::::::::::::::::\:::::::::::::::::::::::::::::|::/::::|::::/:::::::::::/
    |   /:::::::::::::::::::::::::::::::::|:::::::::::::::::::::O::|::|::::::|:::::::::::::::/
413132人目の素数さん:2012/04/27(金) 02:24:13.96
因数分解の解き方を教えてください
初めて書き込むので数学記号まちがってたらすみません
@x^4+(5*x^2)+9
Ax^4+(4*y^4)
414132人目の素数さん:2012/04/27(金) 02:38:46.18
>>413

>>1
>・質問者は何が分からないのか、どこまで考えたのかを明記しましょう。それがない場合、放置されることがあります。
>  (特に、自分でやってみたのにあわないので教えてほしい、みたいなときは必ず書くように)

加えて
http://ja.wikibooks.org/wiki/中学校数学_3年生-数量/式の計算#.E5.9B.A0.E6.95.B0.E5.88.86.E8.A7.A3_2
ほか、googleに因数分解とでも投げれば解説ページはたくさん見つかるだろう
415132人目の素数さん:2012/04/27(金) 02:48:01.39
@x^4+5x^2+9
=(x^4+6x^2+9)-x^2
=(x^2+3)^2-x^2
=…

Aも同じ感じ
416132人目の素数さん:2012/04/27(金) 03:19:23.02
>>411
整数m, n
tan(m/nπ)が有理数となるのは
m/n=0, -1, 1のみでtan(m/nπ)=0
ある自然数kでtan(kθ)=0とすると
ある整数lがあって
kθ=lπ
θ=l/kπ
tanθ=1/2≠0なので矛盾

あとは最初の命題
整数m, n
tan(m/nπ)が有理数となるのは
m/n=0, -1, 1のみ
を示す
それにはcos(m/nπ)が有理数となるのはn=1, 2, 3のみ
という命題をチェビシェフ多項式の知識とかを使って示す

まあ多分もっと簡潔なやり方があるのだろう
417132人目の素数さん:2012/04/27(金) 03:27:13.20
>>416
> m/n=0, -1, 1のみでtan(m/nπ)=0
m/n=0, -1/4, 1/4のみでtan(m/nπ)=0,-1, 1
418132人目の素数さん:2012/04/27(金) 03:47:11.49
3次式f(x)を(x-1)^2で割った余りが-3,(x+1)^2で割った余りが1のとき,
f(x)を求めなさい。

f(x)をどう立式しても条件式が足りなくなってしまいます
よろしくお願いします
419132人目の素数さん:2012/04/27(金) 04:23:09.81
>>411
角度kθの回転行列は([[2,1],[-1,2]]^k)/(√5^k) ([[縦],[縦]]表記)
だが[[2,1],[-1,2]]^k mod 5 のようなものは3乗ごとにループし、0にならない
…もっといいやりかたがありそうだな
420132人目の素数さん:2012/04/27(金) 04:40:06.14
>>418
f(x)=a(x+b)(x-1)^2-3
f(x)=a(x+c)(x+1)^2+1
421132人目の素数さん:2012/04/27(金) 05:41:32.47
>>48
ありがとー(*´д`*)
422132人目の素数さん:2012/04/27(金) 06:53:54.94
漸く、片思いの彼女とデートできる事になり
4月31日ならOKと言ってくれました。
どんな服を着ていったら良いでしょうか。

何とか、2回目にもつなぎたいので
よろしくお願いします。

なお、彼女は遠洋でとても忙しく
2〜3ヶ月おきの月末にしか時間が取れないようです。

次回があるとすれば、6月で、しかも31日しかダメな模様。

なんとか頑張りたいです。
423132人目の素数さん:2012/04/27(金) 07:13:57.57
すげー誤爆
424132人目の素数さん:2012/04/27(金) 07:17:04.87
>>415
ありがとうございます!
根っから文系なのか
xの係数ををそういう風に分ける発想にならなくて…
とてもよくわかりました

>>414
数学的センスがなく全くわからないから書きようがなかったんです
答えに添えてあるわけでもない、ケチをつけるだけのレスって
あなたの底意地が悪いことを示すだけで無益だと思います
425132人目の素数さん:2012/04/27(金) 07:23:02.24
>>411
・tan(kθ)!= -1/2
・y=(2x+1)/(2-x)のグラフ
に注意して、帰納法
426411:2012/04/27(金) 07:38:40.88
解答くださった方ありがとうございました。力不足で解答がまたかけません。
>>425
!はどういうことでしょうか?
427132人目の素数さん:2012/04/27(金) 07:40:44.91
>>424
>数学的センスがなく全くわからないから書きようがなかったんです
分からなかったら許されると思ってる?
マナー違反咎められて逆切れかよw
428132人目の素数さん:2012/04/27(金) 07:41:57.82
>>426
ちょっとは自分でも考えろ。
429132人目の素数さん:2012/04/27(金) 08:09:43.12
>>424

>>414
>数学的センスがなく全くわからないから書きようがなかったんです
>答えに添えてあるわけでもない、ケチをつけるだけのレスって
>あなたの底意地が悪いことを示すだけで無益だと思います

これ面白いな。パラドックスやトートロジーを使ったジョークみたいだぞ。
430132人目の素数さん:2012/04/27(金) 10:35:48.78
>>426
等号の否定
431132人目の素数さん:2012/04/27(金) 11:01:11.47

>>411
回転行列のn乗を固有値から求めるとかは?
でも固有値は範囲外か?
あとiが出てきて計算も大変かも。
432132人目の素数さん:2012/04/27(金) 13:41:25.97
色々考えてみたが>>419>>425も思いつかんかったなー。大したもんだ。
433132人目の素数さん:2012/04/27(金) 14:23:43.49
×また事故が起こった
○またマスゴミが事故の報道をした

15歳以下の通学中の交通事故死亡者数は、平成20年で127人
3日に1人が死亡しているが、普段はほとんど報道されない
亀岡の事件があったからマスゴミが類似の事件を探してきて報道してるだけ

ちなみに、15歳以下の通学中の交通事故死亡者数の推移をみると、
270人(平成13年)→221人(16年)→158人(18年)→127人(20年)と減り続けている
434132人目の素数さん:2012/04/27(金) 14:33:03.51
この板的に、日本に住む15歳以下の人間の総数、免許所持者数、車両数の推移も出さないと説得力無いよ
435132人目の素数さん:2012/04/27(金) 15:56:10.02
『平清盛』プロデューサー在日朝鮮人 磯智明(天皇制度廃止論者)のプロデュース作品

@『かんさほうじん (2008)』反体制・反社会
A『最後の戦犯 (2008)』反日・天皇制度廃止・反体制・反社会
B『リミット -刑事の現場2- (2009)』反体制・反社会

大河の画面が汚いのも、役者が大根なのも、衣装がぼろぼろなのも、役者の下品な立ち回りも、
画面が薄暗いのも、役者が汚いのも全ての原因は




NHKが汚れているから
 史実うんぬんの話ではないのですよ。あの大河は役者、セット、演出等が、いまのNHK内部の汚れ具合を見事に反映しているのです。

薄汚れた空間内で繰り広げられる捏造・妄想(=今年の大河)は、反日・在日の脳内を表しているのです
436教えろください:2012/04/27(金) 19:05:35.67
教ええろ
437132人目の素数さん:2012/04/28(土) 03:22:35.75
>>420
なるほど ありがとうございました
438132人目の素数さん:2012/04/28(土) 03:38:12.46
センター試験で数学を使うものなのですが平面図形って必要なのでしょうか
平面図形の問題といったらほとんど証明問題という記憶があるのですが
センター試験にも証明問題は出るものなのでしょうか?
439132人目の素数さん:2012/04/28(土) 03:52:45.17
センター試験でぐぐって問題を自分で見てみるという
選択肢を提示してみる
440132人目の素数さん:2012/04/28(土) 04:17:37.48
なんとなくわかるよ。
誰かに大丈夫と言ってもらわなきゃ不安で、
誰かにやれと言われなきゃやる気にならない
441132人目の素数さん:2012/04/28(土) 04:32:42.08
まったくそのとおりでございます

黙って勉強します
442132人目の素数さん:2012/04/28(土) 06:01:20.92
x+y+z=1 ,x>0,y>0,z>0 のとき
(x^2+y^2+z^2)/3≦(x^2+y^2+z^2)^2  を示せ。

解答
X=x^2+y^2+z^2とおくと、
(x^2+y^2+z^2)/3≦(x^2+y^2+z^2)^2
⇔X≦3X^2⇔X(3X-1)≧0⇔X≦0orX≧1/3
x>0,y>0,z>0 のとき、X=x^2+y^2+z^2>0なので
x+y+z-1=0 ,x>0,y>0,z>0…★ のとき、x^2+y^2+z^2≧1/3を示せばよい。
ここでxyz空間において原点(0,0,0)と平面x+y+z-1=0との距離は
公式により|-1|/√(1^2+1^2+1^2)=1/√3…※
★をみたすとき、半径k(>0)の球面x^2+y^2+z^2=k^2 を考えて、
kの最小値は※より1/√3 したがってx^2+y^2+z^2≧1/3が成立。
よって(x^2+y^2+z^2)/3≦(x^2+y^2+z^2)^2 が示された。

これってどうなの?
正しいかいなかではなくて、このレベルで無職板で威張ってるんだけどw
443132人目の素数さん:2012/04/28(土) 06:02:07.79
あー、なか→か
444132人目の素数さん:2012/04/28(土) 06:16:42.25
あと、
「y^2=x^n-x+2 を満たす自然数n,x,yの組をすべて求めよ」
「f(x)=1/x^2 +ax^n  (nは自然数とする)が極小値を持つために、aとnがみたすべき条件を求めよ。」

これが解ける俺はスレ最強、だそうです
検便の方が強そうだけどなw
445132人目の素数さん:2012/04/28(土) 13:12:24.66
京都の事故

44 名前:名無しさん@12周年[] 投稿日:2012/04/12(木) 20:06:17.25 ID:LKXz/REk0
かつてはてんかん患者の自動車免許取得は法的に制限されていたが、
2002年6月の道路交通法改正によって、発作が起きても意識障害を伴わない又は、
発作が就寝中に限るなどの患者は、公安委員会の検査や、医師の診断書を提出するなどの条件付で取得に道が開かれた。
日本てんかん協会の運動で、てんかんでも免許取得が可能になった。
その結果、案の定、2002年以降、てんかん患者のひきおこした重大な事故が急増

2011/04 栃木・鹿沼   6人死亡 (持病を隠蔽、過去に事故2件)
2011/04 島根・松江   1人死亡 (持病を隠蔽、薬飲まず)
2011/05 広島・福山   4人重軽傷 (過去に事故2件)
2011/07 愛知・岩倉   2人死亡 (通院歴なしで不起訴)
2011/10 鹿児島・姶良 1人死亡4人重軽傷 (過去に物損事故、薬飲まず)
2012/02 栃木・宇都宮 6人重軽傷 (昨年7月に事故、運転しないと誓約書)

すると2002年の法改正で可能になった、てんかん患者の自動車免許取得を既得権益とする
共産党が支援している日本てんかん協会は
「てんかん患者の権利を守れ」と法務大臣に要望書を提出
http://uni.2ch.net/test/read.cgi/newsplus/1334225953/

その三日後にまたしても死亡事故発生
2012/04 京都・東山区 7人死亡9人重軽傷     (数日前に発作、姉と相談)

日本てんかん協会 (共産党の支援団体)
住所 東京都新宿区西早稲田2-2-8

>>権利を守れ
>>西早稲田2-2-8    差別利権で食ってる団体だな
446132人目の素数さん:2012/04/28(土) 14:44:27.90
          __ノ)-'´ ̄ ̄`ー- 、_
        , '´  _. -‐'''"二ニニ=-`ヽ、
      /   /:::::; -‐''"        `ーノ
     /   /:::::/           \
     /    /::::::/          | | |  |
     |   |:::::/ /     |  | | | |  |
      |   |::/ / / |  | ||  | | ,ハ .| ,ハ|
      |   |/ / / /| ,ハノ| /|ノレ,ニ|ル' 
     |   |  | / / レ',二、レ′ ,ィイ|゙/   私は只の数ヲタなんかとは付き合わないわ。
.     |   \ ∠イ  ,イイ|    ,`-' |      頭が良くて数学が出来てかっこいい人。それが必要条件よ。
     |     l^,人|  ` `-'     ゝ  |        さらに Ann.of Math に論文書けば十分条件にもなるわよ。
      |      ` -'\       ー'  人          一番嫌いなのは論文数を増やすためにくだらない論文を書いて
    |        /(l     __/  ヽ、           良い論文の出版を遅らせるお馬鹿な人。
     |       (:::::`‐-、__  |::::`、     ヒニニヽ、         あなたの論文が Ann of Math に accept される確率は?
    |      / `‐-、::::::::::`‐-、::::\   /,ニニ、\            それとも最近は Inv. Math. の方が上かしら?
   |      |::::::::::::::::::|` -、:::::::,ヘ ̄|'、  ヒニ二、 \
.   |      /::::::::::::::::::|::::::::\/:::O`、::\   | '、   \
   |      /:::::::::::::::::::/:::::::::::::::::::::::::::::'、::::\ノ  ヽ、  |
  |      |:::::/:::::::::/:::::::::::::::::::::::::::::::::::'、',::::'、  /:\__/‐、
  |      |/:::::::::::/::::::::::::::::::::::::::::::::::O::| '、::| く::::::::::::: ̄|
   |     /_..-'´ ̄`ー-、:::::::::::::::::::::::::::::::::::|/:/`‐'::\;;;;;;;_|
   |    |/::::::::::::::::::::::\:::::::::::::::::::::::::::::|::/::::|::::/:::::::::::/
    |   /:::::::::::::::::::::::::::::::::|:::::::::::::::::::::O::|::|::::::|:::::::::::::::/
447132人目の素数さん:2012/04/28(土) 16:19:02.68
>>442
あんまり凄くはない。

けど、一般的な社会人の9割ぐらいはこれができなくなってると思う
褒めてあげると喜ぶよ
448132人目の素数さん:2012/04/28(土) 16:38:59.84
>>442
お前のレベルが低いことは確定
449132人目の素数さん:2012/04/28(土) 16:47:18.29
>>442
さよなら。
450132人目の素数さん:2012/04/28(土) 17:49:37.69
test
451132人目の素数さん:2012/04/28(土) 17:52:46.82
http://www.dotup.org/uploda/www.dotup.org2912668.jpg
「2つの解がともに」という条件なのにD=0が含まれるのが納得いきません
重解は解が1つってことですよね?
問題集これで合っているんでしょうか?
452132人目の素数さん:2012/04/28(土) 17:54:09.48


2つの解が同じなのが重解だろ?
453451:2012/04/28(土) 17:54:53.48
あれ、何か勘違いしてるかも
見なおしてきます
454132人目の素数さん:2012/04/28(土) 17:57:33.58
見なおしてみたいんですけどやっぱり納得いかないです
2つの解がともに1/2より大であるための〜なのに
D=0のときも含んだら解が1つのときも含まれてしまうと思うんですが
455132人目の素数さん:2012/04/28(土) 17:57:35.59
まあこの手の表現の場合すっかり解の同一値性はスルーということになっちゃってるなあ
456132人目の素数さん:2012/04/28(土) 17:58:25.39
>>454
ぶっちゃけ、慣習として根付いちゃってるから
哀しいかな不服でもスルーしたほうが懸命
457451:2012/04/28(土) 17:59:55.50
最後の答えに影響してないしD=0を含んでも含まなくてもいいってことですか?
D>0って書いてても大丈夫ですよね?
それとも=のほうがいいんでしょうか?
458451:2012/04/28(土) 18:01:37.03
連投すいません
最後の答えに影響してましたね・・・
459132人目の素数さん:2012/04/28(土) 18:06:07.09
え、D>0やろ。
460132人目の素数さん:2012/04/28(土) 18:16:15.33
          __ノ)-'´ ̄ ̄`ー- 、_
        , '´  _. -‐'''"二ニニ=-`ヽ、
      /   /:::::; -‐''"        `ーノ
     /   /:::::/           \
     /    /::::::/          | | |  |
     |   |:::::/ /     |  | | | |  |
      |   |::/ / / |  | ||  | | ,ハ .| ,ハ|
      |   |/ / / /| ,ハノ| /|ノレ,ニ|ル' 
     |   |  | / / レ',二、レ′ ,ィイ|゙/   私は只の数ヲタなんかとは付き合わないわ。
.     |   \ ∠イ  ,イイ|    ,`-' |      頭が良くて数学が出来てかっこいい人。それが必要条件よ。
     |     l^,人|  ` `-'     ゝ  |        さらに Ann.of Math に論文書けば十分条件にもなるわよ。
      |      ` -'\       ー'  人          一番嫌いなのは論文数を増やすためにくだらない論文を書いて
    |        /(l     __/  ヽ、           良い論文の出版を遅らせるお馬鹿な人。
     |       (:::::`‐-、__  |::::`、     ヒニニヽ、         あなたの論文が Ann of Math に accept される確率は?
    |      / `‐-、::::::::::`‐-、::::\   /,ニニ、\            それとも最近は Inv. Math. の方が上かしら?
   |      |::::::::::::::::::|` -、:::::::,ヘ ̄|'、  ヒニ二、 \
.   |      /::::::::::::::::::|::::::::\/:::O`、::\   | '、   \
   |      /:::::::::::::::::::/:::::::::::::::::::::::::::::'、::::\ノ  ヽ、  |
  |      |:::::/:::::::::/:::::::::::::::::::::::::::::::::::'、',::::'、  /:\__/‐、
  |      |/:::::::::::/::::::::::::::::::::::::::::::::::O::| '、::| く::::::::::::: ̄|
   |     /_..-'´ ̄`ー-、:::::::::::::::::::::::::::::::::::|/:/`‐'::\;;;;;;;_|
   |    |/::::::::::::::::::::::\:::::::::::::::::::::::::::::|::/::::|::::/:::::::::::/
    |   /:::::::::::::::::::::::::::::::::|:::::::::::::::::::::O::|::|::::::|:::::::::::::::/
461132人目の素数さん:2012/04/28(土) 18:16:45.91
          __ノ)-'´ ̄ ̄`ー- 、_
        , '´  _. -‐'''"二ニニ=-`ヽ、
      /   /:::::; -‐''"        `ーノ
     /   /:::::/           \
     /    /::::::/          | | |  |
     |   |:::::/ /     |  | | | |  |
      |   |::/ / / |  | ||  | | ,ハ .| ,ハ|
      |   |/ / / /| ,ハノ| /|ノレ,ニ|ル' 
     |   |  | / / レ',二、レ′ ,ィイ|゙/   私は只の数ヲタなんかとは付き合わないわ。
.     |   \ ∠イ  ,イイ|    ,`-' |      頭が良くて数学が出来てかっこいい人。それが必要条件よ。
     |     l^,人|  ` `-'     ゝ  |        さらに Ann.of Math に論文書けば十分条件にもなるわよ。
      |      ` -'\       ー'  人          一番嫌いなのは論文数を増やすためにくだらない論文を書いて
    |        /(l     __/  ヽ、           良い論文の出版を遅らせるお馬鹿な人。
     |       (:::::`‐-、__  |::::`、     ヒニニヽ、         あなたの論文が Ann of Math に accept される確率は?
    |      / `‐-、::::::::::`‐-、::::\   /,ニニ、\            それとも最近は Inv. Math. の方が上かしら?
   |      |::::::::::::::::::|` -、:::::::,ヘ ̄|'、  ヒニ二、 \
.   |      /::::::::::::::::::|::::::::\/:::O`、::\   | '、   \
   |      /:::::::::::::::::::/:::::::::::::::::::::::::::::'、::::\ノ  ヽ、  |
  |      |:::::/:::::::::/:::::::::::::::::::::::::::::::::::'、',::::'、  /:\__/‐、
  |      |/:::::::::::/::::::::::::::::::::::::::::::::::O::| '、::| く::::::::::::: ̄|
   |     /_..-'´ ̄`ー-、:::::::::::::::::::::::::::::::::::|/:/`‐'::\;;;;;;;_|
   |    |/::::::::::::::::::::::\:::::::::::::::::::::::::::::|::/::::|::::/:::::::::::/
    |   /:::::::::::::::::::::::::::::::::|:::::::::::::::::::::O::|::|::::::|:::::::::::::::/
462132人目の素数さん:2012/04/28(土) 18:17:04.93
          __ノ)-'´ ̄ ̄`ー- 、_
        , '´  _. -‐'''"二ニニ=-`ヽ、
      /   /:::::; -‐''"        `ーノ
     /   /:::::/           \
     /    /::::::/          | | |  |
     |   |:::::/ /     |  | | | |  |
      |   |::/ / / |  | ||  | | ,ハ .| ,ハ|
      |   |/ / / /| ,ハノ| /|ノレ,ニ|ル' 
     |   |  | / / レ',二、レ′ ,ィイ|゙/   私は只の数ヲタなんかとは付き合わないわ。
.     |   \ ∠イ  ,イイ|    ,`-' |      頭が良くて数学が出来てかっこいい人。それが必要条件よ。
     |     l^,人|  ` `-'     ゝ  |        さらに Ann.of Math に論文書けば十分条件にもなるわよ。
      |      ` -'\       ー'  人          一番嫌いなのは論文数を増やすためにくだらない論文を書いて
    |        /(l     __/  ヽ、           良い論文の出版を遅らせるお馬鹿な人。
     |       (:::::`‐-、__  |::::`、     ヒニニヽ、         あなたの論文が Ann of Math に accept される確率は?
    |      / `‐-、::::::::::`‐-、::::\   /,ニニ、\            それとも最近は Inv. Math. の方が上かしら?
   |      |::::::::::::::::::|` -、:::::::,ヘ ̄|'、  ヒニ二、 \
.   |      /::::::::::::::::::|::::::::\/:::O`、::\   | '、   \
   |      /:::::::::::::::::::/:::::::::::::::::::::::::::::'、::::\ノ  ヽ、  |
  |      |:::::/:::::::::/:::::::::::::::::::::::::::::::::::'、',::::'、  /:\__/‐、
  |      |/:::::::::::/::::::::::::::::::::::::::::::::::O::| '、::| く::::::::::::: ̄|
   |     /_..-'´ ̄`ー-、:::::::::::::::::::::::::::::::::::|/:/`‐'::\;;;;;;;_|
   |    |/::::::::::::::::::::::\:::::::::::::::::::::::::::::|::/::::|::::/:::::::::::/
    |   /:::::::::::::::::::::::::::::::::|:::::::::::::::::::::O::|::|::::::|:::::::::::::::/
463132人目の素数さん:2012/04/28(土) 18:17:17.16
数学の問題やる時も空気読む必要があるんだよ。
物理とか化学の問題も、理想的なケースだと断りがなくても理想的な条件で解く事を求められてるんだなと空気を読んで答えるわけだし。
464132人目の素数さん:2012/04/28(土) 18:17:33.21
          __ノ)-'´ ̄ ̄`ー- 、_
        , '´  _. -‐'''"二ニニ=-`ヽ、
      /   /:::::; -‐''"        `ーノ
     /   /:::::/           \
     /    /::::::/          | | |  |
     |   |:::::/ /     |  | | | |  |
      |   |::/ / / |  | ||  | | ,ハ .| ,ハ|
      |   |/ / / /| ,ハノ| /|ノレ,ニ|ル' 
     |   |  | / / レ',二、レ′ ,ィイ|゙/   私は只の数ヲタなんかとは付き合わないわ。
.     |   \ ∠イ  ,イイ|    ,`-' |      頭が良くて数学が出来てかっこいい人。それが必要条件よ。
     |     l^,人|  ` `-'     ゝ  |        さらに Ann.of Math に論文書けば十分条件にもなるわよ。
      |      ` -'\       ー'  人          一番嫌いなのは論文数を増やすためにくだらない論文を書いて
    |        /(l     __/  ヽ、           良い論文の出版を遅らせるお馬鹿な人。
     |       (:::::`‐-、__  |::::`、     ヒニニヽ、         あなたの論文が Ann of Math に accept される確率は?
    |      / `‐-、::::::::::`‐-、::::\   /,ニニ、\            それとも最近は Inv. Math. の方が上かしら?
   |      |::::::::::::::::::|` -、:::::::,ヘ ̄|'、  ヒニ二、 \
.   |      /::::::::::::::::::|::::::::\/:::O`、::\   | '、   \
   |      /:::::::::::::::::::/:::::::::::::::::::::::::::::'、::::\ノ  ヽ、  |
  |      |:::::/:::::::::/:::::::::::::::::::::::::::::::::::'、',::::'、  /:\__/‐、
  |      |/:::::::::::/::::::::::::::::::::::::::::::::::O::| '、::| く::::::::::::: ̄|
   |     /_..-'´ ̄`ー-、:::::::::::::::::::::::::::::::::::|/:/`‐'::\;;;;;;;_|
   |    |/::::::::::::::::::::::\:::::::::::::::::::::::::::::|::/::::|::::/:::::::::::/
    |   /:::::::::::::::::::::::::::::::::|:::::::::::::::::::::O::|::|::::::|:::::::::::::::/
465132人目の素数さん:2012/04/28(土) 18:37:55.42
空気読みまくれとか、本当は問題が悪いんだけどな…
466132人目の素数さん:2012/04/28(土) 18:44:57.24
大学入試なら心配ないだろうけど、高校の定期試験だと空気読む必要あるかもね
467132人目の素数さん:2012/04/28(土) 18:54:35.54
突っ込み出すと小学生の問題とか酷いぞ
図の様に箱にボールが入ってますとか書いてあるだけで
ピッタリ入ってるとかボールが箱に接しているとか一切断ってなくて、斜め上から見た図だから実際はどうなってるか分からないけど、接してるもんだとして考える。
いちいち覚えちゃないけど、此の手の条件が不十分だけど、まぁこう言いたいんだよなぁ的問題は結構あるからな。
468132人目の素数さん:2012/04/28(土) 19:20:17.69
★ある2ケタの自然数がある。十の位の数字と一の位の数字を入れかえた数を作り、もとの数を足すと、11の倍数になることを説明しなさい。

これの解答の仕方がわかりません。教えて頂けませんでしょうか。
469132人目の素数さん:2012/04/28(土) 19:28:37.88
>>468
例えば 35 = 3×10 + 5×1
470132人目の素数さん:2012/04/28(土) 19:29:57.33
中学生レベルだな
471132人目の素数さん:2012/04/28(土) 19:46:47.28
元の数の十の位をa、一の位をbとすると、
元の数と入れかえた数の和は次のようになる。
10a+b + 10b+a = 11a+11b = 11(a+b)
故に和は11の倍数になる。
472132人目の素数さん:2012/04/28(土) 19:50:19.03
a,bが9以下で共に0にはならんって書いてなきゃ点は貰えないがな
473132人目の素数さん:2012/04/28(土) 19:57:06.16
知らんよそんなのwww
二桁の数のそれぞれの桁の数がどうやって9以上になったり
一緒に0になったりすんだよwww
474132人目の素数さん:2012/04/28(土) 20:02:04.68
>>472
-0.5にはならんって書いてなきゃ点は貰えないがな
475132人目の素数さん:2012/04/28(土) 20:04:56.44
証明問題なら文字の条件は詳しく書く必要あるだろ。
476132人目の素数さん:2012/04/28(土) 20:06:29.85
>>472
2桁の自然数が「こういう形」で表される、という話であって、
「こういう形」がいつも2桁の自然数になる必要はない
477132人目の素数さん:2012/04/28(土) 20:08:13.99
>>472
そんなことが書いてあったら、むしろコイツは必要条件と十分条件の使い分けが分かっていないのかも、と思うよ俺は。
478132人目の素数さん:2012/04/28(土) 21:16:53.71
いや、書いておいた方が良いよ
479132人目の素数さん:2012/04/28(土) 21:28:57.78
ベクトルの係数比較する時に一次独立だって書くのと同じで百も承知だけどかかなきゃならん事ってのがあるんだよ
480132人目の素数さん:2012/04/28(土) 21:30:34.15
>>468です。
>>469さん、>>471さん、>>472さん、>>473さん、>>474さん、>>475さん、>>476さん、>>477さん教えてくれてありがとうございます。
中学生レベルの質問ですみません。
参考書見たのですが記述のばあい、どこまで書けばよいのかわからなかったので質問しました。


481132人目の素数さん:2012/04/28(土) 21:53:06.01
>>479
それは必要な条件なんだから書いて当たり前
今のは余分な条件だから、書かなくてよい
482132人目の素数さん:2012/04/28(土) 23:08:54.68
          __ノ)-'´ ̄ ̄`ー- 、_
        , '´  _. -‐'''"二ニニ=-`ヽ、
      /   /:::::; -‐''"        `ーノ
     /   /:::::/           \
     /    /::::::/          | | |  |
     |   |:::::/ /     |  | | | |  |
      |   |::/ / / |  | ||  | | ,ハ .| ,ハ|
      |   |/ / / /| ,ハノ| /|ノレ,ニ|ル' 
     |   |  | / / レ',二、レ′ ,ィイ|゙/   私は只の数ヲタなんかとは付き合わないわ。
.     |   \ ∠イ  ,イイ|    ,`-' |      頭が良くて数学が出来てかっこいい人。それが必要条件よ。
     |     l^,人|  ` `-'     ゝ  |        さらに Ann.of Math に論文書けば十分条件にもなるわよ。
      |      ` -'\       ー'  人          一番嫌いなのは論文数を増やすためにくだらない論文を書いて
    |        /(l     __/  ヽ、           良い論文の出版を遅らせるお馬鹿な人。
     |       (:::::`‐-、__  |::::`、     ヒニニヽ、         あなたの論文が Ann of Math に accept される確率は?
    |      / `‐-、::::::::::`‐-、::::\   /,ニニ、\            それとも最近は Inv. Math. の方が上かしら?
   |      |::::::::::::::::::|` -、:::::::,ヘ ̄|'、  ヒニ二、 \
.   |      /::::::::::::::::::|::::::::\/:::O`、::\   | '、   \
   |      /:::::::::::::::::::/:::::::::::::::::::::::::::::'、::::\ノ  ヽ、  |
  |      |:::::/:::::::::/:::::::::::::::::::::::::::::::::::'、',::::'、  /:\__/‐、
  |      |/:::::::::::/::::::::::::::::::::::::::::::::::O::| '、::| く::::::::::::: ̄|
   |     /_..-'´ ̄`ー-、:::::::::::::::::::::::::::::::::::|/:/`‐'::\;;;;;;;_|
   |    |/::::::::::::::::::::::\:::::::::::::::::::::::::::::|::/::::|::::/:::::::::::/
    |   /:::::::::::::::::::::::::::::::::|:::::::::::::::::::::O::|::|::::::|:::::::::::::::/
483132人目の素数さん:2012/04/29(日) 08:42:30.08
>>480
> 参考書見たのですが記述のばあい、どこまで書けばよいのかわからなかったので質問しました。
あんた、それは失礼だよ。
>>1
> ・質問者は何が分からないのか、どこまで考えたのかを明記しましょう。それがない場合、放置されることがあります。
>   (特に、自分でやってみたのにあわないので教えてほしい、みたいなときは必ず書くように)
484132人目の素数さん:2012/04/29(日) 10:58:13.26
書き方の程度がわからん、といっているんだから、そうとしか書きようがないな。
485132人目の素数さん:2012/04/29(日) 11:02:44.17
>>454
重解は文字通り解が"重なった"状態
だから解はふたつ
486451:2012/04/29(日) 11:11:07.01
皆さんレスありがとうございます
解の個数を聞かれてる訳じゃないし空気を読むってことなんですかね
487132人目の素数さん:2012/04/29(日) 11:51:09.56
解は1つ、根が2つ…なのだが、ごっちゃにされてるよなあ
488132人目の素数さん:2012/04/29(日) 15:15:14.87
1 名無しさんにズームイン! [] Date:2012/03/28(水) 08:28:15.02 ?ID:NWYs/2ZP Be:
 やらなけゃいけない
 電○の各局への圧力が半端ないんです
 昨日、一昨日前田AKB卒業ネタやった情報番組全てが前田AKB卒業ネタ中の毎分で視聴率がダダ下がりしました。
 各局本音では毎分視聴率ダダ下がりするこのネタははやりたくなかったけど原子力村以上に電○からの圧力が凄いんです

ブーム捏造、枕営業、自社買い、サクラの動員そして
AKBの捏造ブームのために税金が大量に使われている証拠がこちら

やっと気付いた「AKBに電通が絡んでる」ではなく「AKBの正体が電通」な件 その127
http://hayabusa3.2ch.net/test/read.cgi/mor●ningcoffee/1335468718/

AKBの宣伝に税金を使い、その税金が民主党に流れている
テレビの捏造に騙されるな
489132人目の素数さん:2012/04/29(日) 16:50:03.29
(a+b)(b+c)(c+a)+abc
これと
ab(a-b)+bc(b-c)+ca(c-a)
これの因数分解がわかりません
490132人目の素数さん:2012/04/29(日) 16:51:56.83
>>489
展開してaについて降べきの順に並べろ
491132人目の素数さん:2012/04/29(日) 17:02:30.56
2つ目やってみた
-(a-b)(b-c)(c-a)
やり方は>>490さんの言う通り。
展開してとりあえずaでまとめて、
まとめた中身とまとめられなかった部分で
共通の因数を見つけてまとめて最後合体
492132人目の素数さん:2012/04/29(日) 17:02:33.52
両方ですか?
493132人目の素数さん:2012/04/29(日) 17:10:13.41
ありがとうございます
494132人目の素数さん:2012/04/29(日) 18:11:09.22
ルベーグ積分て何ですか?
495132人目の素数さん:2012/04/29(日) 18:13:31.37
測度
496132人目の素数さん:2012/04/29(日) 18:18:01.52
測度?
497132人目の素数さん:2012/04/29(日) 18:25:53.31
測度。
498132人目の素数さん:2012/04/29(日) 18:28:12.62
そくど!
499132人目の素数さん:2012/04/29(日) 18:34:21.87
そくどそくど!
500132人目の素数さん:2012/04/29(日) 19:06:18.30
ルベーグ測度で積分したもの
501132人目の素数さん:2012/04/29(日) 19:09:17.43
ルベーグさんが発案した積分の体系
502132人目の素数さん:2012/04/29(日) 19:33:32.37
          __ノ)-'´ ̄ ̄`ー- 、_
        , '´  _. -‐'''"二ニニ=-`ヽ、
      /   /:::::; -‐''"        `ーノ
     /   /:::::/           \
     /    /::::::/          | | |  |
     |   |:::::/ /     |  | | | |  |
      |   |::/ / / |  | ||  | | ,ハ .| ,ハ|
      |   |/ / / /| ,ハノ| /|ノレ,ニ|ル' 
     |   |  | / / レ',二、レ′ ,ィイ|゙/   私は只の数ヲタなんかとは付き合わないわ。
.     |   \ ∠イ  ,イイ|    ,`-' |      頭が良くて数学が出来てかっこいい人。それが必要条件よ。
     |     l^,人|  ` `-'     ゝ  |        さらに Ann.of Math に論文書けば十分条件にもなるわよ。
      |      ` -'\       ー'  人          一番嫌いなのは論文数を増やすためにくだらない論文を書いて
    |        /(l     __/  ヽ、           良い論文の出版を遅らせるお馬鹿な人。
     |       (:::::`‐-、__  |::::`、     ヒニニヽ、         あなたの論文が Ann of Math に accept される確率は?
    |      / `‐-、::::::::::`‐-、::::\   /,ニニ、\            それとも最近は Inv. Math. の方が上かしら?
   |      |::::::::::::::::::|` -、:::::::,ヘ ̄|'、  ヒニ二、 \
.   |      /::::::::::::::::::|::::::::\/:::O`、::\   | '、   \
   |      /:::::::::::::::::::/:::::::::::::::::::::::::::::'、::::\ノ  ヽ、  |
  |      |:::::/:::::::::/:::::::::::::::::::::::::::::::::::'、',::::'、  /:\__/‐、
  |      |/:::::::::::/::::::::::::::::::::::::::::::::::O::| '、::| く::::::::::::: ̄|
   |     /_..-'´ ̄`ー-、:::::::::::::::::::::::::::::::::::|/:/`‐'::\;;;;;;;_|
   |    |/::::::::::::::::::::::\:::::::::::::::::::::::::::::|::/::::|::::/:::::::::::/
    |   /:::::::::::::::::::::::::::::::::|:::::::::::::::::::::O::|::|::::::|:::::::::::::::/
503132人目の素数さん:2012/04/29(日) 19:36:30.37
数学3で質問です。
y=x^3/(x-1)^2
のグラフを書く問題の途中経過で、疑問が出ました。
直線x=1が漸近線となる際、参考書には、
lim_[x→1]y=∞
と一行だけ書かれているのですが、
この、lim_[x→1]y=∞であるというのは、どうやって示すんでしょうか。
勉強不足で申し訳ないですが、どなたかご教示願います。
504132人目の素数さん:2012/04/29(日) 19:41:43.82
>>503
xが限りなく1に近づくとき、(x-1)^2は正の値をとって、限りなく0に近づく。
また、x^3は正の値をとって、有限の値1に限りなく近づく。
よって、x^3/(x-1)^2は限りなく大きくなる。
505132人目の素数さん:2012/04/29(日) 19:50:21.44
>>503
分母は常に正

感覚的には
(ものすごい1に近い数)/(ものすごく0に近い数)

1.001/0.001
1.00001/0.00001


数学的には>>504
506132人目の素数さん:2012/04/29(日) 20:19:28.69
x^3/(x-1)^2
=x+(3x-2)/(x-1)^2
507132人目の素数さん:2012/04/29(日) 21:23:59.62
(2a+b)(a+2/b)≧9
を証明せよ。(a>0,b>0)
という問題がとけないです。
相加相乗を使ってもぐちゃぐちゃになってしまいました。
ご教授お願い致します
508132人目の素数さん:2012/04/29(日) 21:25:20.59
>>507
一応聞くけど
問題は正確か?
509132人目の素数さん:2012/04/29(日) 21:30:30.48
>>507
a=1
b=2
510132人目の素数さん:2012/04/29(日) 21:30:49.01
>>508
学校の先生が作った問題です。
宿題になっているので答えは提示されていません。
展開して他のくくり方で試してみたりしてはいるのですが
なかなか答えが見えてこないです。
511132人目の素数さん:2012/04/29(日) 21:35:01.66
人の話を聞かない馬鹿は放っておくか
512132人目の素数さん:2012/04/29(日) 21:36:40.81
>>509
問題自体がおかしいみたいですね。
くだらない質問をしてすみませんでした。
513503:2012/04/29(日) 21:42:12.30
>>504-506
ありがとうございます。
514132人目の素数さん:2012/04/29(日) 22:05:09.59
>>507
>(2a+b)(a+2/b)≧9
これが
(2a+b)(1/a+2/b)≧8

(2/a+b)(a+2/b)≧8
なら相加相乗の問題なんだがなw
515仙石14:2012/04/29(日) 22:19:41.14
(2a+b)(a+2/b)=2 a^2+a(b+4/b)+2>=2a^2+4 a 2=2(a^2+2a+1)>=2(a+1)^2 > 2
for a=0 &b=2

(a+b)(a+2/b)>2
516132人目の素数さん:2012/04/29(日) 22:35:42.75
>>512
>>507において
先生が書いた通りにはキミが書き写していないのかもしれない。
517132人目の素数さん:2012/04/29(日) 22:39:35.01
独学じゃ極限とかの概念の理解に苦しむ
518132人目の素数さん:2012/04/29(日) 23:35:33.49
1から12までの自然数が1ずつ書かれた12個の玉が入ってる袋がある
この中から無作為に玉を一個取り出しその玉に書かれている自然数を記録してから
袋の中に戻すという操作を5回繰り返す

このとき2つ同じ数が2組あるのは何通り?という問題で

(5C2×3C2÷2)×12×11×10というのが正解なんですが
なぜ2で割るんでしょうか(´・ω・`)?

教えてください
519132人目の素数さん:2012/04/29(日) 23:44:50.02
aabbc bbaac とか 
cabba cbaab とかを見て
ちょっと考えてみよ。
520132人目の素数さん:2012/04/29(日) 23:47:35.65
解答が間違っているから
521132人目の素数さん:2012/04/29(日) 23:51:26.95
aabbcとbbaacは1通りですか?

例えばa=1b=2とすれば1122Cと2211Cこれは2通りですよね
だって最初に1が出た場合と最初に2が出た場合ってちがう場合ですよね(´・ω・`)?
522132人目の素数さん:2012/04/29(日) 23:52:09.92
まあ具体的にはすぐイメージできないかもしれないが
同じ数字となっている組のところで数え方にダブリがないかを考えてみる
523132人目の素数さん:2012/04/29(日) 23:52:31.21
>>521
一通り

この手の問題文なら
同じ場合として考える
524132人目の素数さん:2012/04/29(日) 23:54:44.10
>>523
お前適当なこと抜かすな
525132人目の素数さん:2012/04/29(日) 23:54:59.00

極限の問題?で質問です
問題集の解答で

α≠0のときθ→0で
tanαθ/θ= sinαθ/αθ・α/cosαθ →α  *ここまではわかる
 
これはα=0のときにも成り立つ            *ここがわからない

と簡潔にあるのですが
なんでα=0のとき成り立つとしてるんでしょうか?
526132人目の素数さん:2012/04/29(日) 23:55:00.07
あんた5C2×3C2の意味が分かってないからそんな事いうのよ
何表してるとおもってるよ5C2×3C2
527132人目の素数さん:2012/04/29(日) 23:57:07.70
>>525
a=0なら
tanαθ/θ= ?
528132人目の素数さん:2012/04/29(日) 23:58:05.96
あaじゃなくてアルファだったか…
529132人目の素数さん:2012/04/29(日) 23:59:13.80
5C2×3C2でダブりは解消してますよね(´・ω・`)?

('ω')ううわからない・・・なぜ2で割るの?
530132人目の素数さん:2012/04/29(日) 23:59:42.42
同じ数の組の数字として
×12が最初の数字の選び方
5C2がその数字が出る回の選び方
×11がもう一つの数字の選び方
3C2がその数字が出る回の選び方
とみると例えば
最初に3と選んで次に4と選んだ場合と
最初に4と選んで次に3と選んだ場合と
この2つを数え上げてしまっている
531132人目の素数さん:2012/04/30(月) 00:06:06.52
>>529
だから数字のはまる場所を選んでるの。
お前さんそれわかってないんだよ。
532132人目の素数さん:2012/04/30(月) 00:06:47.10
>>530
補足
だから重複無く数え上げるには
同じ数の組の数字として
最初に選ぶ数<次に選ぶ数
とかそういった条件がいる
最初に選ぶ数>次に選ぶ数
として数えても同じで
こういう制限なしで数えたら2倍となる
533132人目の素数さん:2012/04/30(月) 00:09:53.86
(5C2×3C2)×12C2×11×10

こう言い換えてもよい
この方がわかりやすいんでない?
534132人目の素数さん:2012/04/30(月) 00:10:46.10
ごめん間違えた
正しくは (5C2×3C2)×12C2×10
535132人目の素数さん:2012/04/30(月) 00:15:53.68
>>534
(`・ω・´) !!

何かもやが晴れてきたような気がします
皆さんのご教示をもとにじっくり考えてみます!!

ほんとうに丁寧にお答えいただいてありがとうございます(^ω^)
536132人目の素数さん:2012/04/30(月) 00:19:03.18
頂角をA、底角をB、Cとする二等辺三角形があり、底辺BC上の任意の点をDとするとき
BDの長さを求める場合、△ABDと△ACDの辺ADの長さが等しいので
BD=xとおいて余弦定理を使って解こうと思ったのですが、xがどんな値でも等式が成り立ってしまいました
なぜそうなるのか教えてください
537132人目の素数さん:2012/04/30(月) 00:20:23.60
あっちなみにこれは岩手大学(教育・農)の過去問でした
場合の数徹底的に鍛えなおそう・・・・(`・ω・´)
538132人目の素数さん:2012/04/30(月) 00:20:53.16
>>533
最初からカード12枚でやるから自分のバカさに気付かないんだよ。三枚とか四枚とか五枚で同じ操作考えてみろって
539132人目の素数さん:2012/04/30(月) 00:21:31.21
アンカ間違えたすいません
もう寝ます
540132人目の素数さん:2012/04/30(月) 00:21:47.22
アンタのデッチ上げた式をあげてくれれば
なんでそうなるのかが少しは分かるかもナ
541132人目の素数さん:2012/04/30(月) 00:27:46.56
>>537
2011年の問題か
改変されてもとの問題とはずいぶん違うけど
542132人目の素数さん:2012/04/30(月) 00:34:43.60
>>536
自分で任意の点Dって言ってるじゃないの
それでxが定まるわけないでしょ
543525:2012/04/30(月) 00:37:42.45
>>527
α=0なら
tanαθ/θ= 0 /θ
で極限とるまでもなくこの時点で0と確定するから
極限とった結果 分子分母ともに 0/0 となる不定形とは違う
だから結果として>>525はα=0のときでも成り立つ
ってことでいいのでしょうかね?
544132人目の素数さん:2012/04/30(月) 00:41:17.04
>>543
はいそうです

数列に良く出てくるn≧2に成り立つ式がn=1について調べてみたら(出どころは違っても結果が同じだから)一つにまとめちゃってもいいよね!
ってのと同じ。
545132人目の素数さん:2012/04/30(月) 00:42:12.53
これを教えて下さいお願いしますhttp://beebee2see.appspot.com/i/azuYrf6lBgw.jpg
546132人目の素数さん:2012/04/30(月) 00:43:57.73
>>545
(k+1)!-k!=(k+1)k!-k!=k*k!
547132人目の素数さん:2012/04/30(月) 00:50:26.06
>>512
不等式の右辺の9を生かせば、こんな問題だったのかも
(2+b/a)(2+a/b)≧9 或いは ((2a+b)/a)((2b+a)/b)≧9
548132人目の素数さん:2012/04/30(月) 01:22:56.47
ここに質問を書くヤツって、
式を正確に書くことになんか気を使っているとは思えないヤツばっか。
549132人目の素数さん:2012/04/30(月) 04:14:52.01
_____Q
|_|_|_|_|_|
|_|_|_|_|_|
|_|_|_|_|_|
P

上のような縦、横全てが等間隔の道筋があります。
太郎はP(左下)からQ(右上)へ最短距離を進み、花子はQからPへ最短距離を進みます。
ただし、各分岐点で進む方向は、等確率で選ぶものとします。
太郎と花子は同時に出発し、太郎と花子の速さは等しく、一定である時、太郎と花子の出会う確率を求めよ。

という問いですが、私が思ったのは、太郎は四マスずつ進んで、
D 左から二番目、上から一番目
C 左から三番目、上から二番目
B 左から四番目、上から三番目
A 左から五番目、上から四番目
の四点へ到達すると考えました。それぞれについて、道順は上から
4通り、6通り、4通り、1通りの計15通りなので、分母は15×15になると思います。
従って出会うのは、
Dで出会う 4/15×1/15(花子はDに達するには1通り)
Cで出会う 6/15×4/15(花子は4通り)
Bで出会う 4/15×6/15(花子は6通り)
Aで出会う 1/15×4/15(花子は4通り)
これらを足して56/225だと考えました。
しかし解答では、
Dで出会う 5/16・(1/2)^4
Cで出会う 6×(1/2)^4・4×(1/2)^4
Bで出会う 4×(1/2)^4・6×(1/2)^4
Aで出会う (1/2)^4・5/16
(5/16は、1-(1/2)^4-6×(1/2)^4-4×(1/2)^4の余事象で出しています)
を全て足して29/128になっています。確かに解答のやり方も分かるのですが、なぜ上の方法ではいけないのかが分からないでいます
よろしくお願いします
550549:2012/04/30(月) 04:18:13.73
すいません、はじめの図が少しずれました
Qは格子上、右上の端です
551132人目の素数さん:2012/04/30(月) 04:30:45.82
左上の曲がり角を考慮してないんだと思う
上上上右→(1/2)*(1/2)*(1/2)*1
上上右上→(1/2)*(1/2)*(1/2)*(1/2)
上右上上→同上
右上上上→同上
4通りだけど上上上右だけ重みが違う
552132人目の素数さん:2012/04/30(月) 04:33:35.34
ああそれだけじゃつじつま合わないね
ごめんちゃんとやってみる
553132人目の素数さん:2012/04/30(月) 04:37:49.12
つじつま合うな。度々ごめん
太郎の上上上右と花子の下下下左だけ重みが2倍だから
分母に+1して、太郎のDと花子のAに+1する
Dで出会う 5/16×1/16(花子はDに達するには1通り)
Cで出会う 6/16×4/16(花子は4通り)
Bで出会う 4/16×6/16(花子は6通り)
Aで出会う 1/16×5/16(花子は4通り)
これで合算58/256=29/128
554132人目の素数さん:2012/04/30(月) 05:00:15.32
>>551-553
ありがとうございます
なるほど、道順は15通りでも重みが違う、と考えるのですね
納得できました。助かりましたm(__)m
555132人目の素数さん:2012/04/30(月) 07:44:58.37
何だよ、重みって
556132人目の素数さん:2012/04/30(月) 08:11:00.33
Z/5Z では、多項式関数 x^5 - x はつねに0を吐きますが、
「Z/5Z では x^5 - x = 0 が恒等式になる」 とみてはいけないと聞きました。
どうしてでしょう。
557132人目の素数さん:2012/04/30(月) 08:32:58.28
          __ノ)-'´ ̄ ̄`ー- 、_
        , '´  _. -‐'''"二ニニ=-`ヽ、
      /   /:::::; -‐''"        `ーノ
     /   /:::::/           \
     /    /::::::/          | | |  |
     |   |:::::/ /     |  | | | |  |
      |   |::/ / / |  | ||  | | ,ハ .| ,ハ|
      |   |/ / / /| ,ハノ| /|ノレ,ニ|ル' 
     |   |  | / / レ',二、レ′ ,ィイ|゙/   私は只の数ヲタなんかとは付き合わないわ。
.     |   \ ∠イ  ,イイ|    ,`-' |      頭が良くて数学が出来てかっこいい人。それが必要条件よ。
     |     l^,人|  ` `-'     ゝ  |        さらに Ann.of Math に論文書けば十分条件にもなるわよ。
      |      ` -'\       ー'  人          一番嫌いなのは論文数を増やすためにくだらない論文を書いて
    |        /(l     __/  ヽ、           良い論文の出版を遅らせるお馬鹿な人。
     |       (:::::`‐-、__  |::::`、     ヒニニヽ、         あなたの論文が Ann of Math に accept される確率は?
    |      / `‐-、::::::::::`‐-、::::\   /,ニニ、\            それとも最近は Inv. Math. の方が上かしら?
   |      |::::::::::::::::::|` -、:::::::,ヘ ̄|'、  ヒニ二、 \
.   |      /::::::::::::::::::|::::::::\/:::O`、::\   | '、   \
   |      /:::::::::::::::::::/:::::::::::::::::::::::::::::'、::::\ノ  ヽ、  |
  |      |:::::/:::::::::/:::::::::::::::::::::::::::::::::::'、',::::'、  /:\__/‐、
  |      |/:::::::::::/::::::::::::::::::::::::::::::::::O::| '、::| く::::::::::::: ̄|
   |     /_..-'´ ̄`ー-、:::::::::::::::::::::::::::::::::::|/:/`‐'::\;;;;;;;_|
   |    |/::::::::::::::::::::::\:::::::::::::::::::::::::::::|::/::::|::::/:::::::::::/
    |   /:::::::::::::::::::::::::::::::::|:::::::::::::::::::::O::|::|::::::|:::::::::::::::/
558132人目の素数さん:2012/04/30(月) 08:53:28.62
ちょっとなにいってんのかわからない
559132人目の素数さん:2012/04/30(月) 09:41:12.74
>>556
係数が0でないから
560132人目の素数さん:2012/04/30(月) 09:53:44.68
5∧7=5、5∨7=7というように、∧,∨がそれぞれminとmaxを表してる時に

(x∧9)∨(y∧9)∨(z∧10)=6
(x∧6)∨(y∧8)∨(z∧4)=6
(x∧6)∨(y∧4)∨(z∧6)=5

この3つの式がある時にx,y,zはどのようにして求められますか?
561132人目の素数さん:2012/04/30(月) 10:18:39.13
あれ・・xが確定しないけど
俺 間違ってるのかな・・
562132人目の素数さん:2012/04/30(月) 10:20:10.83
>>559
ということは
「恒等式 ・・・ どのようなxを代入しても成立する式 」という定義は、ここでは正しくないということでしょうか。
563132人目の素数さん:2012/04/30(月) 10:46:08.06
          __ノ)-'´ ̄ ̄`ー- 、_
        , '´  _. -‐'''"二ニニ=-`ヽ、
      /   /:::::; -‐''"        `ーノ
     /   /:::::/           \
     /    /::::::/          | | |  |
     |   |:::::/ /     |  | | | |  |
      |   |::/ / / |  | ||  | | ,ハ .| ,ハ|
      |   |/ / / /| ,ハノ| /|ノレ,ニ|ル' 
     |   |  | / / レ',二、レ′ ,ィイ|゙/   私は只の数ヲタなんかとは付き合わないわ。
.     |   \ ∠イ  ,イイ|    ,`-' |      頭が良くて数学が出来てかっこいい人。それが必要条件よ。
     |     l^,人|  ` `-'     ゝ  |        さらに Ann.of Math に論文書けば十分条件にもなるわよ。
      |      ` -'\       ー'  人          一番嫌いなのは論文数を増やすためにくだらない論文を書いて
    |        /(l     __/  ヽ、           良い論文の出版を遅らせるお馬鹿な人。
     |       (:::::`‐-、__  |::::`、     ヒニニヽ、         あなたの論文が Ann of Math に accept される確率は?
    |      / `‐-、::::::::::`‐-、::::\   /,ニニ、\            それとも最近は Inv. Math. の方が上かしら?
   |      |::::::::::::::::::|` -、:::::::,ヘ ̄|'、  ヒニ二、 \
.   |      /::::::::::::::::::|::::::::\/:::O`、::\   | '、   \
   |      /:::::::::::::::::::/:::::::::::::::::::::::::::::'、::::\ノ  ヽ、  |
  |      |:::::/:::::::::/:::::::::::::::::::::::::::::::::::'、',::::'、  /:\__/‐、
  |      |/:::::::::::/::::::::::::::::::::::::::::::::::O::| '、::| く::::::::::::: ̄|
   |     /_..-'´ ̄`ー-、:::::::::::::::::::::::::::::::::::|/:/`‐'::\;;;;;;;_|
   |    |/::::::::::::::::::::::\:::::::::::::::::::::::::::::|::/::::|::::/:::::::::::/
    |   /:::::::::::::::::::::::::::::::::|:::::::::::::::::::::O::|::|::::::|:::::::::::::::/
564132人目の素数さん:2012/04/30(月) 12:38:23.03
>>556
恒等式には2通りの定義がある。
広い意味では「定義域内のどのような値をとっても、つねに成り立つ等式」だから Z/5Z で x^5−x=0 は恒等式。
整式に限定した狭い意味では「次数が等しい」条件がついている。
565132人目の素数さん:2012/04/30(月) 12:51:46.98
>>564
そんな定義はないよ
566132人目の素数さん:2012/04/30(月) 12:52:02.68
>>560
(x∧9)∨(y∧9)∨(z∧10)=6 → x∧9, y∧9, z∧10 ≦ 6 で x∧9, y∧9, z∧10 のどれかは 6 → x, y, z ≦ 6 で x, y, z のどれかは 6
(x∧6)∨(y∧8)∨(z∧4)=6 → x∧6, y∧8, z∧4 ≦ 6 で x∧6, y∧8, z∧4 のどれかは 6 → x, y ≦ 6 で x, y のどれかは 6
つまり x, y, z ≦ 6 で x, y のどれかは 6
(x∧6)∨(y∧4)∨(z∧6)=5 → x∧6, y∧4, z∧6 ≦ 5 で x∧6, y∧4, z∧6 のどれかは 5 → x, z ≦ 5 で x, z のどれかは 5
つまり y=6 で x, z ≦ 5 で x, z のどれかは 5
567132人目の素数さん:2012/04/30(月) 12:55:00.50
568132人目の素数さん:2012/04/30(月) 13:00:27.84
>>567
ないってことだね
569132人目の素数さん:2012/04/30(月) 13:04:43.72
>>567
ないってことだなwww
570132人目の素数さん:2012/04/30(月) 13:28:26.01
無いことを証明された(T_T)
571132人目の素数さん:2012/04/30(月) 13:40:19.91
個々の整式が等しいことと、整式から定まる関数が等しい(定義域内のどのような値をとっても常に等しい)こと
(係数体が無限体であれば、論理的に同値)
この二つをどちらも「恒等式」だと勘違いしてるわけね
572132人目の素数さん:2012/04/30(月) 14:09:59.21
定義域をx=0、1としたとき、x^2=xを恒等式と呼ぶかというと呼ばないとするのが普通ってことか?
573132人目の素数さん:2012/04/30(月) 15:23:59.42
 やらなけゃいけない
 電○の各局への圧力が半端ないんです
 昨日、一昨日前田AKB卒業ネタやった情報番組全てが前田AKB卒業ネタ中の毎分で視聴率がダダ下がりしました。
 各局本音では毎分視聴率ダダ下がりするこのネタははやりたくなかったけど原子力村以上に電○からの圧力が凄いんです

AKBの宣伝に税金が使われ、その税金が民主党に流れている。
ブーム捏造、枕営業、自社買い、サクラの動員そして
AKBの捏造ブームのために税金が大量に使われている証拠がこちら

やっと気付いた「AKBに電通が絡んでる」ではなく「AKBの正体が電通」な件 その127
http://hayabusa3.2ch.net/test/read.cgi/morningcoffee/133∵5468718/     ∵を外して貼り付け

テレビのやらせブームに騙されるな
574132人目の素数さん:2012/04/30(月) 16:13:33.16
>>566
やっぱりそういう風に地道に解くしかないんですかね?
普通の連立方程式みたいに式同士を足し合わせたりとかは出来ないのかな
575132人目の素数さん:2012/04/30(月) 16:39:55.44
クソな3-SAT問題に似てる気がする
576132人目の素数さん:2012/04/30(月) 17:00:45.22
max(x,y)=(x+y+|x-y|)/2
min(x,y)=(x+y-|x-y|)/2
でなんとかなる場合があるかもしれない。やってないけど。
577132人目の素数さん:2012/04/30(月) 17:46:38.41
質問です。

5a=7b は a=7/5b になると問題分の解説に書いてあるのですが
5a=7b がどういう理屈で a=7/5b になるのでしょうか?
a=5/7b でも b=7/5a でもなく a=7/5b になる理屈が知りたいです。

あと 7/5 を帯小数に直すと 1.4になるのですが
自分がやってる直し方は、どうも効率が悪いような気がするのです。

まず 7/5 を帯分数の 1と2/5 に直しておいてから
次に 2/5 を小数に直すために 
5/5=1 だから 1/5 を導くために 10÷5=2 と計算します。
それで 1/5=0.2 だということが分かるから 2/5=0.2x2=0.4 なので
7/5=1.04 ということが、やっと分かるという次第です。

分数を小数に直す効率的な方法ありましたら、教えてください。

ちなみのその問題文はこれです。↓
一定の速さの動く歩道がある。ある人がこの動く歩道上を歩道の動きと反対方向に
端から端まで歩いたところ、動く歩道上を歩道と同じ方向に端から端まで歩いた時と比べて、
6倍の時間がかかった。歩く速さはどちらの場合も同じで一定であるとすると、歩く速さは動く歩道の速さの何倍か?

その問題分の解説文↓
人の歩く速さをa、動く歩道の速さをbとすると、動く歩道上を反対方向に歩くときの速さは(a-b)、
動く歩道上を歩道と同じ方向に歩くときの速さは(a+b)である。反対方向に歩くと6倍の時間がかかるのだから、
歩道と同じ方向に歩くときの速さは反対方向に歩くときの速さの6倍ということになる。したがって、
(a+b):(a-b)=6:1
6(a-b)=(a+b)
6a-6b=a+b
より、5a=7bとなり、a=7/5bである。5/7=1.4だから、歩く速さは動く歩道の速さの1.4倍である

答え=1.4倍
578132人目の素数さん:2012/04/30(月) 17:47:56.84
釣りは他所でやって下さい
579132人目の素数さん:2012/04/30(月) 17:49:05.57
>>577
算数からやり直し
580577:2012/04/30(月) 17:57:24.44
>>578
すみません釣りじゃないです。。。

>>579
算数から中学の数学までやり直しましたが、どうしてもわかりません。
581132人目の素数さん:2012/04/30(月) 18:01:25.60
それがわからないレベルでは「算数から中学の数学までやり直した」とは言えない
「算数から中学の数学までやったつもりでいたが実際は何一つ分からなかった」と言うべき
582132人目の素数さん:2012/04/30(月) 18:05:24.18
人間には向き・不向きというものがあるのだから、諦めて他の分野で頑張ったらどうだろう
君にも出来ることはきっとあるはずだから
583132人目の素数さん:2012/04/30(月) 18:48:28.10
          __ノ)-'´ ̄ ̄`ー- 、_
        , '´  _. -‐'''"二ニニ=-`ヽ、
      /   /:::::; -‐''"        `ーノ
     /   /:::::/           \
     /    /::::::/          | | |  |
     |   |:::::/ /     |  | | | |  |
      |   |::/ / / |  | ||  | | ,ハ .| ,ハ|
      |   |/ / / /| ,ハノ| /|ノレ,ニ|ル' 
     |   |  | / / レ',二、レ′ ,ィイ|゙/   私は只の数ヲタなんかとは付き合わないわ。
.     |   \ ∠イ  ,イイ|    ,`-' |      頭が良くて数学が出来てかっこいい人。それが必要条件よ。
     |     l^,人|  ` `-'     ゝ  |        さらに Ann.of Math に論文書けば十分条件にもなるわよ。
      |      ` -'\       ー'  人          一番嫌いなのは論文数を増やすためにくだらない論文を書いて
    |        /(l     __/  ヽ、           良い論文の出版を遅らせるお馬鹿な人。
     |       (:::::`‐-、__  |::::`、     ヒニニヽ、         あなたの論文が Ann of Math に accept される確率は?
    |      / `‐-、::::::::::`‐-、::::\   /,ニニ、\            それとも最近は Inv. Math. の方が上かしら?
   |      |::::::::::::::::::|` -、:::::::,ヘ ̄|'、  ヒニ二、 \
.   |      /::::::::::::::::::|::::::::\/:::O`、::\   | '、   \
   |      /:::::::::::::::::::/:::::::::::::::::::::::::::::'、::::\ノ  ヽ、  |
  |      |:::::/:::::::::/:::::::::::::::::::::::::::::::::::'、',::::'、  /:\__/‐、
  |      |/:::::::::::/::::::::::::::::::::::::::::::::::O::| '、::| く::::::::::::: ̄|
   |     /_..-'´ ̄`ー-、:::::::::::::::::::::::::::::::::::|/:/`‐'::\;;;;;;;_|
   |    |/::::::::::::::::::::::\:::::::::::::::::::::::::::::|::/::::|::::/:::::::::::/
    |   /:::::::::::::::::::::::::::::::::|:::::::::::::::::::::O::|::|::::::|:::::::::::::::/
584132人目の素数さん:2012/04/30(月) 18:49:33.16
>>582
何こいつ
585132人目の素数さん:2012/04/30(月) 18:50:39.83
調子に乗った大学生か何かだろう
586560:2012/04/30(月) 18:52:38.62
また似たような質問ですみません
>>560と同じ条件(さっきは書き忘れてたんですが、x,y,z,wは0〜10の値)で、
(x∧2) ∨ (y∧3) ∨ (z∧1) ∨ (w∧0)=1
(x∧4) ∨ (y∧1) ∨ (z∧4) ∨ (w∧3)=1
(x∧5) ∨ (y∧6) ∨ (z∧6) ∨ (w∧0)=2
(x∧7) ∨ (y∧8) ∨ (z∧7) ∨ (w∧10)=2

という4つの式があります。
>>566さんに教えてもらったのと同じようにしていった場合
1つ目の式で、x≦1,y≦1ということがわかり、2つ目の式でz≦1,w≦1ということがわかります。
しかし、そうすると3つ目と4つ目の式の右辺に1より大きい値が出るのはおかしいと思って行き詰まってしまいました。
どういうことなのでしょう。問題が誤植なのか、自分の考えで間違ってるところがあるのか、教えてください。
587132人目の素数さん:2012/04/30(月) 18:59:02.30
>>586
解なしじゃない?と言いたいけど
先の問題でも確定してないところがあるから
何か間違ってる気がする。
588560:2012/04/30(月) 19:14:33.48
>>587
いえ、先の問題は、x,z≦5でxとzのいずれかが5、y=6という条件を満たしているx,y,zの組であれば
検算しても正しい結果でした。
x,y,zの組合せは複数あるみたいです。
589132人目の素数さん:2012/04/30(月) 19:33:01.52
A,Bが有限集合である時、
|(A-B)|∪|(B-A)|=|A|+|B|-2|A∩B|
であることを示せという問題があるのですが
全くわかりません。
対象差についての証明だと思うのですが、何も出てこないです。
590132人目の素数さん:2012/04/30(月) 19:36:05.09
>>589
ヴェン図を描く
厳密に証明するなら、全単射を構成して1対1対応がつくことを示すことになるけど、
高校数学ならそこまでする必要なし
591132人目の素数さん:2012/04/30(月) 19:41:17.30

xについての2次方程式
ax^2+bx+(a+b)=0を考える。
この2次方程式は異なる2つの解α,βをもち、以下の条件を満たす。

・α+β=αβ
・α<1<β

このとき、条件を満たす実数a,bの存在する領域の面積を求めよ。

これってどうやるんですか?
592132人目の素数さん:2012/04/30(月) 19:42:17.54
>>590
-は差集合だとわかるのですが、+というのは和集合と考えていいのでしょうか?
593132人目の素数さん:2012/04/30(月) 19:44:22.39
>>591
解と係数の関係を使って
> ・α+β=αβ
> ・α<1<β
をa,bで表す
594132人目の素数さん:2012/04/30(月) 19:44:52.09
>>592
正しくは |(A-B)||+|(B-A)|=|A|+|B|-2|A∩B|
もしくは |(A-B)∪(B-A)|=|A|+|B|-2|A∩B|
だろう

∪は和集合、+は自然数の加法
595132人目の素数さん:2012/04/30(月) 19:45:46.06
  Λ_Λ  
 <=( ´∀`)  
 (    )  朝鮮人は宇宙一ニダ
 | | |   
 〈_フ__フ


  Λ_Λ    
 < ;`Д´>  あ…
 (    )ポロ 
 | | |  ヽヽ
 (__フ_フ =( ´∀`)
朝鮮人だらけの東京のテレビ局が日夜流す、デマや歪曲に騙されないようにしましょう。
596132人目の素数さん:2012/04/30(月) 19:48:16.94
>>593
ありがとうございます
やってみます
597132人目の素数さん:2012/04/30(月) 19:53:04.97
>>594
たぶん後者の式ですね。ありがとうございます。
598132人目の素数さん:2012/04/30(月) 19:55:28.64
自分がもし家庭教師をやったとして>>577のような質問をされたら怖くなるな
何言っても分かんないって言われそう
599132人目の素数さん:2012/04/30(月) 20:13:18.59
>>577
とりあえず"移項"でgoogle検索してでたページを
10頁くらい巡って自習してなさい
600132人目の素数さん:2012/04/30(月) 20:14:19.24
あ、移項じゃなかったけどまあいいや
考え方似てるから
601132人目の素数さん:2012/04/30(月) 20:27:36.35
          __ノ)-'´ ̄ ̄`ー- 、_
        , '´  _. -‐'''"二ニニ=-`ヽ、
      /   /:::::; -‐''"        `ーノ
     /   /:::::/           \
     /    /::::::/          | | |  |
     |   |:::::/ /     |  | | | |  |
      |   |::/ / / |  | ||  | | ,ハ .| ,ハ|
      |   |/ / / /| ,ハノ| /|ノレ,ニ|ル' 
     |   |  | / / レ',二、レ′ ,ィイ|゙/   私は只の数ヲタなんかとは付き合わないわ。
.     |   \ ∠イ  ,イイ|    ,`-' |      頭が良くて数学が出来てかっこいい人。それが必要条件よ。
     |     l^,人|  ` `-'     ゝ  |        さらに Ann.of Math に論文書けば十分条件にもなるわよ。
      |      ` -'\       ー'  人          一番嫌いなのは論文数を増やすためにくだらない論文を書いて
    |        /(l     __/  ヽ、           良い論文の出版を遅らせるお馬鹿な人。
     |       (:::::`‐-、__  |::::`、     ヒニニヽ、         あなたの論文が Ann of Math に accept される確率は?
    |      / `‐-、::::::::::`‐-、::::\   /,ニニ、\            それとも最近は Inv. Math. の方が上かしら?
   |      |::::::::::::::::::|` -、:::::::,ヘ ̄|'、  ヒニ二、 \
.   |      /::::::::::::::::::|::::::::\/:::O`、::\   | '、   \
   |      /:::::::::::::::::::/:::::::::::::::::::::::::::::'、::::\ノ  ヽ、  |
  |      |:::::/:::::::::/:::::::::::::::::::::::::::::::::::'、',::::'、  /:\__/‐、
  |      |/:::::::::::/::::::::::::::::::::::::::::::::::O::| '、::| く::::::::::::: ̄|
   |     /_..-'´ ̄`ー-、:::::::::::::::::::::::::::::::::::|/:/`‐'::\;;;;;;;_|
   |    |/::::::::::::::::::::::\:::::::::::::::::::::::::::::|::/::::|::::/:::::::::::/
    |   /:::::::::::::::::::::::::::::::::|:::::::::::::::::::::O::|::|::::::|:::::::::::::::/
602132人目の素数さん:2012/04/30(月) 20:30:42.23
http://m.chiebukuro.yahoo.co.jp/detail/q1244640032

この問題で「三角形OPMをOを中心に座標平面内で1回転させた」図形のイメージがわかないのですがどなたか文字でも画像でもいいので説明してくださるとありがたいです
よろしくお願いします
603132人目の素数さん:2012/04/30(月) 20:34:42.27
>>602
円って書いてあるじゃん
604132人目の素数さん:2012/04/30(月) 20:36:58.23
>>603
というか座標平面内で一回転っていう意味がわからないです・・・
図形を回転させたら立体になっちゃわないんですか?
605132人目の素数さん:2012/04/30(月) 20:39:42.83
「座標平面内で」一回転だから立体にはならない
三角形のOの部分に針を刺してぐるんと一回転させた感じ
606132人目の素数さん:2012/04/30(月) 20:42:56.83
>>605
はっとしましたw
なんで気づかなかったんだろうってかんじです・・・
ありがとうございました
607132人目の素数さん:2012/04/30(月) 23:37:50.48
某国政府著「民間防衛」より転載


某国元首:費用のかからない方法で敵(国)を滅ぼすことができる。
魅力でひきつける宣伝は効果的な武器だ。我々の意図を美しい装飾で包み隠そう。
文化は立派な隠れ蓑になる。音楽・芸術・旅行などの口実で仲間をつくり、一方的な文化交流(聞こえは良いが実際は押し付け→韓流)をしよう。
彼らは徐々に罠にはまっていく。


韓流、AKB商法の正体
もちろん、韓流、AKBは捏造ブームである。

ヨン様ファン?
もちろん在日ババアの動員ですよ。
テレビは愚民の思考を止めるために存在し、そのためにテレビ業界が役人、政治家により手厚く保護されていることを忘れないでください。
608132人目の素数さん:2012/05/01(火) 00:55:30.21
xy^2+y+1-x の因数分解教えてください

最低次数はxなんだろうけど整理のやり方がわかりません
609132人目の素数さん:2012/05/01(火) 00:58:04.31
>>608
(y^2-1)x+y+1
=(y+1)(y-1)x+y+1
=(y+1)((y-1)x+1)
610132人目の素数さん:2012/05/01(火) 01:18:09.70
>>609
=(y+1)(y-1)x+1(y+1)
=(y+1)[(y-1)x+1]

ありがとうございます。2行目からですが、上のような理解であってますか?
611132人目の素数さん:2012/05/01(火) 01:23:53.83
>>610
まあそうですね
612132人目の素数さん:2012/05/01(火) 01:27:43.01
>>611
ありがとうございました。
613132人目の素数さん:2012/05/01(火) 04:01:58.22
614132人目の素数さん:2012/05/01(火) 04:20:21.04
↑糞まみれ
615132人目の素数さん:2012/05/01(火) 05:42:32.49
二次関数y=-x^2+ax+bの、すべてのxにおける最大値は7、x≦0における最大値は3である。
このとき、定数a,bの値を求めよ。

という質問について少し質問です。

解答に
x≦0における最大値が3であるから、このグラフの軸x=pについてはp>0である。

とあるのですが、よくわかりません p<0では何がいけないのでしょうか。
616132人目の素数さん:2012/05/01(火) 06:23:42.86
>>615
軸での値が定義域全体での最大値なので
軸が負だと条件に反することになる
617132人目の素数さん:2012/05/01(火) 07:50:47.28
>>615
> x≦0における最大値が3であるから、このグラフの軸x=pについてはp>0である。
本当にそう書かれているなら、おかしい。
グラフの軸x=pについてはp>0である理由は、
・すべてのxにおける最大値は7(→グラフが上に凸で軸におけるyの値が7)
・x≦0における最大値は3(→上の条件と合わせると、この区間に軸が存在しない)
が必要で、「x≦0における最大値が3であるから」だけで「このグラフの軸x=pについてはp>0」と言っているのはおかしい。
618132人目の素数さん:2012/05/01(火) 09:04:28.78
>>615
略図で構わないので上に凸である二次関数のグラフを
p>0の場合と p<0の場合に分けてy軸辺りで書いて考えて見たら?

>>617
定数a,bの値を求めることが主眼なので設問の条件にあった「すべてのxにおける最大値は7」が
当然の前提であるから説明での記載は省かれてるのでしょう。スペース節約側面もあるし。
619132人目の素数さん:2012/05/01(火) 09:10:19.45
>>618
「x≦0における最大値は3」も設問の条件にある当然の前提だが。
「このグラフの軸x=pについてはp>0」の理由を書くなら、「すべてのxにおける最大値は7」は省略しちゃいかん条件だと思うよ。
620132人目の素数さん:2012/05/01(火) 09:19:49.73
>>619
あなたと論議しても結論でないと思う。あなたにアンカー付けて失敗した。
621132人目の素数さん:2012/05/01(火) 09:28:41.47
両方ないと成立しないからなあ。
答案なら減点されるだろう。
622132人目の素数さん:2012/05/01(火) 09:34:23.91
この設問では、定数a,bの値が合えば満点です。
623132人目の素数さん:2012/05/01(火) 09:55:15.46
その説明を読んで?となった質問者の方がマシだなあ。
そこで止まっちゃったのは低レベルではあるけれど。
624132人目の素数さん:2012/05/01(火) 10:11:17.00
求める解答が「定数a,bの値を求めよ」なのに解答には
前提を全て書いてないので「減点」とかは素晴らしい発想です。

流石に蛇足を書くような人間の性格が滲みでているレスでです。
625132人目の素数さん:2012/05/01(火) 11:16:37.89
蛇足でトンチンカンなことを書いちゃった自分に言ってんのか?
626132人目の素数さん:2012/05/01(火) 11:23:03.87
おかしな茶々入れして失敗だったと思ったくせにスルー出来ずに逆ギレしてんのか。
宅浪スレの住民か?
627132人目の素数さん:2012/05/01(火) 11:30:08.00
質問者が勝手に省略してんじゃねえか?
解説の全文を見ないとはっきりせんな。
質問者の疑問についてはここの回答にあるとおりだけど、
質問者がその部分を勝手に見落としてるだけなんじゃないかと。
628132人目の素数さん:2012/05/01(火) 12:09:27.74
先に軸についての言及があるんだろうな、たぶん。
629132人目の素数さん:2012/05/01(火) 12:33:45.59
>>120のような例があるからな。
630525:2012/05/01(火) 12:57:34.91
>>544
遅くなってすみませんが
どうもありがとうございました!
631132人目の素数さん:2012/05/01(火) 13:07:39.44
>>625-629
自演乙。

蛇足までならまだしも蛇足を書かないと減点、ってアホ丸出し。それとも真性のアホですか?
よほど悔しかったと見える。恥の上塗り、面白くてお腹の皮が痛くなってます。

自分は>>616氏の趣旨をグラフにしたら理解しやすいと言ってるに過ぎない。
632132人目の素数さん:2012/05/01(火) 13:13:31.77
↑なにこれ?
633132人目の素数さん:2012/05/01(火) 13:25:31.88
>>632
↑なにこれ? と書くお前と五十歩百歩の(ry

いゃしかし流石に蛇足までならまだしも「蛇足を書かないと減点」とか言わないし。
「蛇足を書かないと減点」って考えると、お腹の皮が痛いけど低周波なくても腹筋の鍛錬できてる。
アリガトウとお礼を言いたい。
634132人目の素数さん:2012/05/01(火) 16:30:27.97
lim_(x→π) √(a+cosx)-b/(x-π)^2 =1/4
となる定数a,bの値は?
b=√(a-1)以降わかりません。よろしくお願いします。
635132人目の素数さん:2012/05/01(火) 16:34:03.81
>>634
分数やルートはどの部分に掛かってるの?
636132人目の素数さん:2012/05/01(火) 16:44:33.27
y=xとy=x^2で囲まれる部分を直線y=xを中心に回転させた図形の体積を求められますか?x軸、y軸を中心にした図形以外は教科書の範囲外ですが…。
637132人目の素数さん:2012/05/01(火) 16:49:26.22
3C課す様な大学ならふつうにでるよ
638132人目の素数さん:2012/05/01(火) 16:52:34.68
何も考えずに>>634をWolfram|Alphaに投げてみた
なんかでてきた
639132人目の素数さん:2012/05/01(火) 17:00:35.43
考えろ
640132人目の素数さん:2012/05/01(火) 17:07:48.80
いや俺いま来た第三者だから
641634:2012/05/01(火) 17:36:04.03
すいません ルートはa+cosxにかかっています。
分子は√(a+cosx)
分母は(x-π)の2乗です。わかりにくくてごめんなさい
642132人目の素数さん:2012/05/01(火) 17:45:17.49
>>641
> 分子は√(a+cosx)
> 分母は(x-π)の2乗です。
bはどこ行った?
643132人目の素数さん:2012/05/01(火) 17:57:26.96
>>641
>すいません ルートはa+cosxにかかっています。

そんな事はさすがにわかる
644132人目の素数さん:2012/05/01(火) 17:58:49.06
>>641
取り敢えずは
x-πをXとでもおいてみろ
645132人目の素数さん:2012/05/01(火) 18:03:15.76
>>636
計算に自信があるなら曲線を-45°回転させて
x軸回転
典型的解法なら
ハイ理か新数演に載っている
646132人目の素数さん:2012/05/01(火) 18:05:00.74
>>634
まず分母0になるから分子0の不定形の形にならないと有限の値は出てこない
647634:2012/05/01(火) 18:17:40.47
たびたびすいません -bまでが分子です
648640:2012/05/01(火) 18:21:36.05
Wolfram|Alphaも
lim_(x→π) [{(√(a+cosx))-b}/{(x-π)^2}] =1/4
は解けない、あるいは解かない…のかな?
まあ手計算で解いたけど

>>647
このレス2行目の式でいいってことね
649634:2012/05/01(火) 19:06:57.21
>648
2行目の式でいいです
650あのこうちやんは始皇帝だった:2012/05/01(火) 19:26:39.67

 お前たちは、定職に就くのが先決だろがああああああ!!!!!!!!!

 ニート・無職の、ゴミ・クズ・カスのクソガキどもがあああああ!!!!!!!!
651132人目の素数さん:2012/05/01(火) 19:40:29.98
GW中ですけどー(笑)
652132人目の素数さん:2012/05/01(火) 19:52:35.37
bはわかるというあたり努力してないわけじゃないだろうし
それにしてはヒントが出なさすぎるようなんでヒント

高校レベルなら三角関数まじりの極限は
lim_[x→0] (sinx)/x = 1
に持っていくことが多い(というか他にあったかな)
この問題では少々長手順だとは思うけど、持っていけないことはない
653132人目の素数さん:2012/05/01(火) 21:02:16.12
分母分子に√(a+cosx)+√(a-1)をかける
654634:2012/05/01(火) 21:05:35.60
ありがとうございました ヒントのおかげでわかりました
a=2 b=1ですね

655132人目の素数さん:2012/05/01(火) 21:11:20.70
>>653
(cosx+1)/(x-π)^2*(1/√(a+cosx)+√(a-1))
cosx+1=2cos^2(x/2)=2sin^2((x-π)/2)
(cosx+1)/(x-π)^2*(1/√(a+cosx)+√(a-1))
=2sin^2((x-π)/2)/(x-π)^2*(1/√(a+cosx)+√(a-1))
→1/2*(1/√(a-1)+√(a-1))=1/4
656132人目の素数さん:2012/05/01(火) 23:12:35.02
正三角形ABCにおいて、辺BC、CA、ABを3:(n-3)に内分する点をそれぞれD,E,Fとする(ただしn>6)。
線分AD,BE,CFの交点の作る三角形の面積が元の三角形の4/49のとき、nを求めろ。
657132人目の素数さん:2012/05/01(火) 23:13:12.35
いやだ
658132人目の素数さん:2012/05/01(火) 23:13:56.13
そういわずに
659132人目の素数さん:2012/05/01(火) 23:15:17.11
そうかー
660132人目の素数さん:2012/05/01(火) 23:15:56.06
そうだよー
661132人目の素数さん:2012/05/01(火) 23:18:01.28
考えろよ
662132人目の素数さん:2012/05/01(火) 23:18:34.70
ふぇぇ…おにいちゃんたち、イジワルだよぉ…
663132人目の素数さん:2012/05/01(火) 23:29:24.97
>>656
すみません。自己解決しました。
664132人目の素数さん:2012/05/01(火) 23:32:54.49
教えてほしいんですけど、1>>xのとき1/(1-x)=1+x+x^2+・・・って何展開ですか?
665132人目の素数さん:2012/05/01(火) 23:38:51.69
とりあえずgoogleに「1>>xのとき1/(1-x)=1+x+x^2+・・・」とぶちこんでみたのか?
666132人目の素数さん:2012/05/01(火) 23:40:15.59
べき級数展開
667132人目の素数さん:2012/05/01(火) 23:42:30.68
まぁ僕はぶち込まれるほうですから
668132人目の素数さん:2012/05/01(火) 23:43:12.43
なんでホモが湧いてるんですかねぇ・・・
669132人目の素数さん:2012/05/01(火) 23:46:21.62
僕っ娘
670132人目の素数さん:2012/05/01(火) 23:49:25.55
8
671132人目の素数さん:2012/05/01(火) 23:53:10.56
>>664
テイラー展開かマクローリン展開
672132人目の素数さん:2012/05/02(水) 00:33:00.60
関数 f(x)=3x-1、g(x)=x^2+1について、

f^-1(x)=xを満たすx

ありがちな問題なのですが、解法を忘れてしまいました。
初歩的なものですが解答と解説お願いしますm(_ _)m
673132人目の素数さん:2012/05/02(水) 00:36:01.58
>>672
y=3x-1をxについて解いてxとyを入れ替えたら逆関数
674132人目の素数さん:2012/05/02(水) 00:40:33.52
>>673
逆関数までは出せましたが、その後はどうすればよいのでしょうか。
675132人目の素数さん:2012/05/02(水) 00:43:06.84
>>674
=xとして1次方程式になると思うけどそれを解いてxを求めてください
676132人目の素数さん:2012/05/02(水) 00:44:19.59
          __ノ)-'´ ̄ ̄`ー- 、_
        , '´  _. -‐'''"二ニニ=-`ヽ、
      /   /:::::; -‐''"        `ーノ
     /   /:::::/           \
     /    /::::::/          | | |  |
     |   |:::::/ /     |  | | | |  |
      |   |::/ / / |  | ||  | | ,ハ .| ,ハ|
      |   |/ / / /| ,ハノ| /|ノレ,ニ|ル' 
     |   |  | / / レ',二、レ′ ,ィイ|゙/   私は只の数ヲタなんかとは付き合わないわ。
.     |   \ ∠イ  ,イイ|    ,`-' |      頭が良くて数学が出来てかっこいい人。それが必要条件よ。
     |     l^,人|  ` `-'     ゝ  |        さらに Ann.of Math に論文書けば十分条件にもなるわよ。
      |      ` -'\       ー'  人          一番嫌いなのは論文数を増やすためにくだらない論文を書いて
    |        /(l     __/  ヽ、           良い論文の出版を遅らせるお馬鹿な人。
     |       (:::::`‐-、__  |::::`、     ヒニニヽ、         あなたの論文が Ann of Math に accept される確率は?
    |      / `‐-、::::::::::`‐-、::::\   /,ニニ、\            それとも最近は Inv. Math. の方が上かしら?
   |      |::::::::::::::::::|` -、:::::::,ヘ ̄|'、  ヒニ二、 \
.   |      /::::::::::::::::::|::::::::\/:::O`、::\   | '、   \
   |      /:::::::::::::::::::/:::::::::::::::::::::::::::::'、::::\ノ  ヽ、  |
  |      |:::::/:::::::::/:::::::::::::::::::::::::::::::::::'、',::::'、  /:\__/‐、
  |      |/:::::::::::/::::::::::::::::::::::::::::::::::O::| '、::| く::::::::::::: ̄|
   |     /_..-'´ ̄`ー-、:::::::::::::::::::::::::::::::::::|/:/`‐'::\;;;;;;;_|
   |    |/::::::::::::::::::::::\:::::::::::::::::::::::::::::|::/::::|::::/:::::::::::/
    |   /:::::::::::::::::::::::::::::::::|:::::::::::::::::::::O::|::|::::::|:::::::::::::::/
677132人目の素数さん:2012/05/02(水) 00:48:53.28
>>675
ありがとうございます。こんな単純な計算だったとは;
感謝です
678132人目の素数さん:2012/05/02(水) 00:56:49.63
>>677
もっと単純には
f(f^{-1}(x))=x
を利用する
f^{-1}(x)=x
f(f^{-1}(x))=f(x)
x=3x-1
679132人目の素数さん:2012/05/02(水) 00:58:21.25
半角の公式について
証明の終盤で
角をa→a/2に置き換える部分で
どうも引っかかってしまいます。
何故成り立つのでしょうか
680132人目の素数さん:2012/05/02(水) 00:58:53.15
>>678
> f(f^{-1}(x))=f(x)
> x=3x-1
え?
681132人目の素数さん:2012/05/02(水) 00:59:58.19
>>679
> 角をa→a/2に置き換える部分で
式を省略せずに書かないと、どこが疑問なのかが伝わらない。
682132人目の素数さん:2012/05/02(水) 01:00:25.28
>>679
だったら最初からa/2使え
683132人目の素数さん:2012/05/02(水) 01:10:43.18
>>681、2レスありがとうございます。
2倍角の公式より、
sin*a=1-cos2a/2

ここでa=a/2と置き換えて、
sin*a/2=1-cosa/2

置き換えること自体に違和感というか納得がいってません。数学的センスが皆無なのですが、どうかご回答よろしくお願いします。
684132人目の素数さん:2012/05/02(水) 01:21:56.15
cos(2a)=1-2sin^2(a)
2sin^2(a)=1-cos(2a)
a→a/2
2sin^2(a/2)=1-cos(2*(a/2))

2sin^2(a)=1-cos(2a)にa=b/2を代入
2sin^2(b/2)=1-cos(b)
685132人目の素数さん:2012/05/02(水) 01:23:45.00
>>680
f(x)=f(x)の左辺にx=f^-1(x)を代入するとf(f^{-1}(x))=f(x)。
f(f^{-1}(x))は往って来いでxに戻るので=x。
f(f^{-1}(x))=f(x)の左辺はxで、右辺は f(x)=3x-1なので、結局
x=3x-1

f^-1(x)=xを満たすxはy=f^-1(x)とy=xとの交点のx座標。
y=f(x)とその逆関数はy=xについて対称なので、上記の交点は当然y=f(x)との交点でもある。
だから、求めるxはf(x)=xの解ってこと。
686質問:2012/05/02(水) 01:38:39.32
お邪魔します。円周率を求める計算で「内接正n角形の周 < 円周 < 外接正n角形の周」という手法が古くからよく知られています。
内接正n角形の周が円周より短いことは、2点間の最短距離が線分で与えられることからすぐにわかります。
しかし外接正n角形の周が円周より長いことはどのように説明できるのでしょうか?
できれば初等的な説明をつけたいのですが上手く行きません。
どなたかよろしくおねがいしますm(_ _)m
687132人目の素数さん:2012/05/02(水) 01:41:47.87
>>684
わかりました!
ありがとうございます。
688132人目の素数さん:2012/05/02(水) 01:42:21.19
>>686
y=sinx と y=x の上下関係を考えればわかる
689132人目の素数さん:2012/05/02(水) 01:44:03.47
AB=AC=1、∠A=90°である直角二等辺三角形ABCの辺AB、BC上にそれぞれ点P,Qをとり、m
AP=BQとする。2点P,Q間の距離が最小となるときのAPの長さを求めよ。

解答にPH:BH:PB:1:1:√2
PB=1-xであるから
PH=BH=1/√2PB=1-x/√2
ゆえに QH=lBH-BQl=・・・・・

と続くのですがここで絶対値を付けるのにはどういった意味があるのでしょうか。
またなぜ絶対値を付けるのでしょうか

誰か詳しくお願いします。
690132人目の素数さん:2012/05/02(水) 01:47:18.09
>>689
問題を正確に
691132人目の素数さん:2012/05/02(水) 01:49:40.03
>>686
よく考えたら y=tanx と y=x だった
692686:2012/05/02(水) 01:50:35.07
>>688
早速ありがとうございます。
半径1の円に外接する正n角形を考えて
正n角形の周 = 2n*tan(π/n)
ですよね。
sinx < x をどのように使えばよいのでしょうか?
飲み込みが悪くてすみません。
693132人目の素数さん:2012/05/02(水) 01:51:55.44
>>686
面積が一定の図形の外周の長さが最も短くなるのは真円の時
→円よりも外接n角形の面積の方が大きい
→外接n角形の外周が円周の長さを下回る事はない

う〜んどうだろう?
数学的にこれで証明になるのかどうかは知らないけど、理解はできると思う
694686:2012/05/02(水) 01:52:06.85
>>691
解決しました、ありがとうございました!
695132人目の素数さん:2012/05/02(水) 01:53:59.63
>>694
y=sinx と y=x を比較するのは、円周と内接正多角形を比べるときね
「最短距離」を考えるよりも、より厳密だと思うよ
696686:2012/05/02(水) 01:55:30.00
>>693
なるほど、こんな説明もできるのですね、ありがとうございます。
697686:2012/05/02(水) 01:57:14.41
>>695
「2点間の最短距離を実現するのが線分である」を用いた証明も厳密だと思いますが。。。
外接円のほうもこのくらいわかりやすく説明できないものかと思ったのですがそれはどうやら無理そうですね。
698132人目の素数さん:2012/05/02(水) 01:59:22.15
>>697
距離が最小とか、面積が最小とか…
最小であることを証明するためには、変分法という全然初等的でない理論を使うことになるんだよ実は
699132人目の素数さん:2012/05/02(水) 02:03:35.15
三角不等式で十分だべ
700686:2012/05/02(水) 02:06:44.43
>>698
三角不等式を使えば変分法なしでも示せると思いますが如何ですか?
面積一定図形の周長最小化は変分法使わないと証明できなさそうですが…
701132人目の素数さん:2012/05/02(水) 02:06:52.06
ABCの比について4A=9B=Cという計算結果のあと、したがって比は9:4:36であると記述してあるのですが4:9:1じゃないのですか?
どうして9:4:36になるのでしょうか?よろしくお願いします
702132人目の素数さん:2012/05/02(水) 02:08:12.09
>>701
4A=9B=Cの全ての辺を36で割ってA/9=B/4=C/36
703132人目の素数さん:2012/05/02(水) 02:09:50.07
>>701
4A=9B=C=k とおいてみると分かりやすい
704132人目の素数さん:2012/05/02(水) 02:13:11.98
>>700
すまん確かに三角不等式でいけるね
円周を折れ線で近似したものと、内接正多角形を、不等式で比較してから、折れ線を細かく分割して極限をとる
705132人目の素数さん:2012/05/02(水) 02:15:13.59
>>702
4:9:1じゃなんでだめなんですか?
706686:2012/05/02(水) 02:15:44.04
>>704
やはりそうですよね。
色々とすっきりしました、ありがとうございました!
707132人目の素数さん:2012/05/02(水) 02:16:41.74
>>705
4A=9B=Cやろ?
A=4, B=9, C=1 を代入したときイコール成り立ってるか?
708132人目の素数さん:2012/05/02(水) 02:20:44.49
>>707
なるほど!ありがとうございました
709132人目の素数さん:2012/05/02(水) 05:35:33.90
ごめんなさい先走っちゃいました 詳しくはこんな感じです

AB=AC=1、∠A=90°である直角二等辺三角形ABCの辺AB、BC上にそれぞれ点P,Qをとり、m
AP=BQとする。2点P,Q間の距離が最小となるときのAPの長さを求めよ。

解答に
AP=BQ=xとおくと 0<x<1
点Pから辺BCに下ろした垂線をPHとすると
∠PBH=45°、∠PHB=90°
よってPH:BH:PB:1:1:√2
PB=1-xであるから
PH=BH=1/√2PB=1-x/√2
ゆえに QH=lBH-BQl=・・・・・

と続くのですがここで絶対値を付けるのにはどういった意味があるのでしょうか。
またなぜ絶対値を付けるのでしょうか

誰か詳しくお願いします。
710132人目の素数さん:2012/05/02(水) 06:11:50.03
>>709
Q と H の位置関係で BH ,BQ のどっちが長くなるかが変わってくるので
QH をひとつの式で表現しようと思ったら絶対値が必要になる
x が0に近いとき,1に近いときを図にすればわかると思う
711132人目の素数さん:2012/05/02(水) 06:23:28.45
>>710
バカ発見
712132人目の素数さん:2012/05/02(水) 06:26:41.19
たまに、一言「アホ」「バカ」とだけ言う奴がいるな
数学苦手なのかな
713132人目の素数さん:2012/05/02(水) 06:33:42.60
まあバカには指摘しても自覚しないか
714132人目の素数さん:2012/05/02(水) 06:41:55.00
>>713
そうだな
無駄だから、もうこういうことは止めにしような
715132人目の素数さん:2012/05/02(水) 06:54:10.14
x,y,zは異なる数で、x(1-2y)=y(1-2z)=z(1-2x)を満たしている。
(1) x(1-2y)の値を求めよ。

x(1-2y)=y(1-2z)=z(1-2x)=kとおくことは分かるのですが、その後どうすればいいでしょう。
716132人目の素数さん:2012/05/02(水) 07:12:56.91
>>710 どうもです!
717132人目の素数さん:2012/05/02(水) 07:26:53.32
>>715
x(1-2y)=y(1-2z)
y(1-2z)=z(1-2x)
z(1-2x)=x(1-2y)
この連立方程式を解く
718132人目の素数さん:2012/05/02(水) 07:37:54.53
>>715
>>717
基本方針としては
yとzをxで表す
719132人目の素数さん:2012/05/02(水) 07:50:26.10
>>717 おいw
720132人目の素数さん:2012/05/02(水) 08:28:24.77
>>719
とりあえず分母に出てくる式は0でないとします
3つ目の式からz=x(1-2y)/(1-2x)
とまずzをx, yで表す
それで上の2つの式に代入すればzは消去できます
代入するところの少ない1つ目の式で考えて
yの2次式とみて整理して因数分解したら
(1)は1/2と求まる
721132人目の素数さん:2012/05/02(水) 16:24:39.46
今高2でチャート式で確立と数列同時に進めているんですが、解説みても納得しないこととが多々あります。正直、時間もないんで、理解よりこの問題はこんな風に解くんだと頭に叩き込んでますがやっぱり理解してからの方がいいでしょうか?
722132人目の素数さん:2012/05/02(水) 16:35:33.45
確率なんて理解しないと解きようがないだろ
もっと単純な確率や場合の数の問題をやるといい
723132人目の素数さん:2012/05/02(水) 16:37:58.20
確率問題の8割は文章題
どんな文脈をどんな式に直すかで点数が決まる
文脈を式に変換するところなんて公式があるわけがないので
ゼヒとも理解しないとキツい
724132人目の素数さん:2012/05/02(水) 17:05:11.87
高校の数列が難しいって言ってる奴って池沼なんじゃないかなって思う。
数列でも極限とかに足突っ込むと分からなくなるのとかは同情出来るけど、文系もやらされる範囲の数列とか計算するだけじゃんね。
計算うぜぇぐらいで勉強しても出来ないってのが一番謎だわ。
725132人目の素数さん:2012/05/02(水) 17:13:51.44
別に数列に限ったことじゃない
726132人目の素数さん:2012/05/02(水) 17:18:14.10
極限も計算だべ
727132人目の素数さん:2012/05/02(水) 17:38:28.96
22 :可愛い奥様:2012/02/19(日) 20:10:14.60 ID:BAiCWuwH0

AKBを全面に出したバスが走ってて
http://www.akb-sg.net/akbus01.jpg

バスに書かれたリンクに行ったらAKBのページがあって
クールジャパンとは関係ないショップやカフェの広告が出てて
http://www.cooljapan.com.sg/akb48_live.html

クールジャパンとは関係ない現地のAKB企業HPへのリンクが貼ってあり
経済産業省の文字の真上で是非リンク先に訪れて下さいねと書いてある Please visit the official AKB48 Singapore website for the latest schedule of highlights!
www.akb48.com.sg

これのどこが日本の利益になるの?
税金使ってAKBと秋元康のバックにいる民主党への利益誘導をしてるだけでしょ
728132人目の素数さん:2012/05/02(水) 18:38:32.82
x^2+y^2=25は
x=3のとき、y=±4と
2つの値をとってしまいます
これは関数なんですか?
まぁ、y=±√(25-x^2)と書けば、
x=3のとき、y=4「または」y=-4で2つの値はとってないんですが、図形的に2つとっています。
1対1対応が崩れてるきが...。
それとも、これは関数ではない?
729132人目の素数さん:2012/05/02(水) 18:54:03.01
お前知ってて聞いてるだろ(笑)
一体一の時意外も関数っていうよ高校でも、陰伏関数とか陰関数とか言うし
多価関数とか言う事もある。厳密に突っ込み出すとニュアンスとか使う文脈とか色々あって面倒だけど
高校の時は細かいこと考えない方がいいと思うよ。
730132人目の素数さん:2012/05/02(水) 19:29:33.55
y=x^2も1対1ではないが
731132人目の素数さん:2012/05/02(水) 20:48:08.30
二次方程式の解の公式の分子部分の b^2-4ac に二次関数を代入(?)すると
x軸の共有点になると言うのはなんとなく分かったのですが

そもそもb^2-4acに代入するとなぜx軸の共有点になるのでしょうか

b^2-4acって式は何を表してるのでしょうか
732132人目の素数さん:2012/05/02(水) 20:53:56.46
たぶん中学校の教科書の最後らへんに書いてる
733132人目の素数さん:2012/05/02(水) 20:57:02.98
>>731
日本語が大分おかしいぞ
テストで減点くらうかもしれんから気をつけて
734132人目の素数さん:2012/05/02(水) 20:57:46.63
>>731
まず2次方程式 ax^2+bx+c=0 の解の導き方は分かる?
735132人目の素数さん:2012/05/02(水) 21:00:42.75
ぜってーわかってねーよコイツ
> 判別式にブチ込むと共有点に「なる」
「ナル」って言いきってんだぜ

言う通りに中学の教科書からやり直した方が良い
決定的な何かが分かってない
留年の危険がある
736132人目の素数さん:2012/05/02(水) 21:03:24.05
解の公式を丸暗記してるんだと思う
公式を使わずに、平方完成を使ってax^2+bx+c=0を解いてみると分かる
737132人目の素数さん:2012/05/02(水) 21:15:53.17
全く分かってないんだなってのは良くわかるんだけど
何を言っているのかが全くわからん。
具体例でも出して自分の主張を分かりやすく説明してくれないか?
738132人目の素数さん:2012/05/02(水) 22:03:41.45
http://beebee2see.appspot.com/i/azuYh-unBgw.jpg
n>=2とあらやますがなぜ1項目はもとめきれないんでしょうか?
739132人目の素数さん:2012/05/02(水) 22:06:08.82
>>738
第0群ってなんでしょうか?
740132人目の素数さん:2012/05/02(水) 22:07:30.75
まさかここまで叩かれるとは思わなかった;;

共有点になるじゃなくて個数が分かるんでしたね・・・

仰る通り解の公式の丸暗記でやってました

中学の復習します
741132人目の素数さん:2012/05/02(水) 22:07:40.40
お前一行上の文章読んでなんもおもわないの?
n-1群までの総数の和を出してるのにnに1突っ込んだら0群目までの和とかよくわからないもの考えることになるだろ。
742132人目の素数さん:2012/05/02(水) 22:10:15.84
>>740
叩かれることは気にするな。
でもわかってなさ杉。
743132人目の素数さん:2012/05/02(水) 22:10:40.35
奇数Nの約数の総和が奇数になるのは、Nが平方数のとき
といえるでしょうか
744132人目の素数さん:2012/05/02(水) 22:13:37.82
>>731

叩くっていうか意味不明

>二次方程式の解の公式の分子部分の b^2-4ac に

ここまではわかる

二次関数を代入(?)すると

は?二次関数を代入って概念始めてきいたぞしかもxに代入とか特定の文字に入れるとかわかるけどb^2-4acに代入って
まるまるb^2-4acを二次関数に入れ替えるの?でも二次関数って何さしてんの??意味が全くわからないぞ

>x軸の共有点になると言うのはなんとなく分かったのですが

共有点?何とx軸の??

>そもそもb^2-4acに代入するとなぜx軸の共有点になるのでしょうか

えっ???

>b^2-4acって式は何を表してるのでしょうか

二次関数のb^2-4acを取ることについてきいてるのかな?それとも二次方程式の解の公式のb^2-4acの部位について聞いてるのかな???

って正直言ってる意味がわからなすぎるけど、こんな意味不明な文書くだけあって何も分かってないって事だけが強烈に理解できる。
745132人目の素数さん:2012/05/02(水) 22:16:54.08
判別式で二次関数の位置関係がわかる理由ってのは教科書にくどいぐらいの説明がされていると思うのだけど、教科書読んだ事ありますか?
746132人目の素数さん:2012/05/02(水) 22:18:54.86
確立で区別するとかしないとか意味わからない。だれか俺に理解さしてくれー
747132人目の素数さん:2012/05/02(水) 22:20:55.57
仮説を確立する、確率はどうしにはならない
748132人目の素数さん:2012/05/02(水) 22:22:01.73
>>741
和は0。
749132人目の素数さん:2012/05/02(水) 22:28:53.92
>>746
確率は全て区別つくって覚えておいて問題ない。

袋の中に黒玉が99こ白玉が1こ入ってるとしよう。
取り出した時には黒が出るか白が出るかの二通りしかないように見えるけど、実際には全然違うってのはわかるだろ?
確認は出やすさを考えるから黒か白かで適当にわけるわけにはいかないってのがわかるよな?
色を忘れて、どの玉も一個選ぶ場合の選ばれ方は偏りが無いってのもわかるよな?(これが確率の問題に書いてある同様に確かってこと)
んじゃ100のどの玉も確かなんだから白より黒が99倍あるから99倍出やすいってのもわかるよね?

この黒玉は99こあるよねって考えることを同じ色で見た目では区別がつかないようにみえるけど実際には確かに違うから「区別する」って言ってるだけ
当たり前の事。
750132人目の素数さん:2012/05/02(水) 22:34:26.95
>>743
簡略的に
N = p^a*q^b*r^c
p,q,r:奇数
a,b,c:自然数
として
約数の個数は
(a+1)*(b+1)*(c+1)個

総和の奇遇は約数の個数で決まり、
総和が奇数となるのはa,b,cが偶数の時で
Nは平方数


これを一般化すればいいかと・・・
751132人目の素数さん:2012/05/02(水) 22:36:08.20
ダブリが多いということは
そうなる確率が高いということなのだ!!!!!!!!!!!!!!!!1
752132人目の素数さん:2012/05/02(水) 22:37:13.13
ダブリー
753132人目の素数さん:2012/05/02(水) 22:54:12.71
>>731
皆が言ってるように用語が不適切、トンチンカン。したがって文章が理解不能。
で、トンチンカンな質問を勝手に解釈しトンチンカンな内容で応えようとした自分がいた。

>中学の復習します
二次方程式の解の個数は高校の範囲なので中学まで戻る必要はない。

1)二次関数「y = ax^2+bx+c」とx軸の共有点を求めるには「y = 0」を代入することは理解してるか?
理由はx軸の式は「y = 0」であるので、代入するとその交点、例えば( 、0)を求めることができるから。

したがって二次関数「y = ax^2+bx+c」とx軸の共有点の個数は二次方程式ax^2+bx+c=0の解の個数に一致する
ただし↓で説明する 数TTでの b^2-4ac<0の時を除く

2)通常、二次方程式の判別式は D=b^2-4ac と記されている。因みに「D」discriminantの略かな。
二次関数を標準形に変形した時に現れるアレの一部だけど。
@この判別式D=b^2-4ac=0の時の解の個数は1個なのは解るか? 
解らなければ x^2+6x+9=0 や x^2-4x+4=0 の解を求めて確かめてみてくれ。
AD=b^2-4ac>0の時は解の個数は2個なのは解るか?
解らなければ x^2+7x+12=0 の解を求めて確かめてみてくれ。

ところでこちらからの質問。何年生だ? で解らないのは数Tだろうか数TTだろうか?
754132人目の素数さん:2012/05/02(水) 22:58:24.90
>>744
全く難解な文章だけど意訳する。

>二次関数を代入(?)すると
二次関数の係数「a, b, c」を判別式 D=b^2-4ac に代入ってことだろう。
755132人目の素数さん:2012/05/03(木) 00:18:27.66
>>753何年生かは勘弁してください。。。まあ高1じゃないです それと数Tに関してです

丁寧にありがとうございました

>>754
はいそれです
756132人目の素数さん:2012/05/03(木) 00:22:58.72
>>755
それよりなにより、
「二次関数を代入する」、とかいて>>754の意味になると思った過程を知りたい。
757132人目の素数さん:2012/05/03(木) 00:24:09.83
>>755
マジで教科書読めよ。
758132人目の素数さん:2012/05/03(木) 00:25:15.29
>>755
二次方程式とか中学までもどる必要が無いとかそういうことじゃあない、

> b^2-4ac に二次関数を代入(?)すると
本人も?付きで書いてる通り、「係数」の概念がコイツの頭には無いんだよ
だから1次関数見せられても2次関数見せられてもn次関数見せられても
「係数」ってな概念に行きつかネーはず。

んで概念が無いから「日本語がおかしい」ってなハナシも出んのさ。
そりゃあ、概念が頭ン中に無いんなら表現しようがない、
だから文章にするといろいろとおかしくなるんだよ

だから中学まで戻って
テメーの頭に「係数」とかいう概念を植え付けて来い
そこまで戻れ

あと係数も分かんねーつーなら きっと変数の概念も分かんねーよ コイツ
高校とか何年生とかそういうレヴェルじゃねー
今すぐ中学校の教科書ひっぱり出して今すぐ復習しろ
759132人目の素数さん:2012/05/03(木) 00:26:35.13
チャートの黄色を買いましょう
760132人目の素数さん:2012/05/03(木) 00:29:33.58
質問の仕方さえ分からないレベルの人はちょくちょく見かけるけど、
今回に限って、何でみんなそんなに攻撃的なんだ?
761132人目の素数さん:2012/05/03(木) 00:33:03.88
>>760
本当に分かってねーヤツは
マジで言わないと本気にならないから
762132人目の素数さん:2012/05/03(木) 00:33:31.50
>>760
長々と書いているのは一人だろ
763132人目の素数さん:2012/05/03(木) 00:37:44.57
チャート買って例題を1問ずつちゃんと解いていけ
解答前にある説明はゴミなんで
分からないところは解答のほうだけを読め
764132人目の素数さん:2012/05/03(木) 01:01:53.16
>>755
何年生れ(なんねんうまれ)ではなくて何年生(なんねんせい)か? って聞いたんだけど。
高1か高2か高3か、あるいはオーバーでもアドバイスが違うので聞いた。

>b^2-4acって式は何を表してるのでしょうか
既に説明済みだけど二次方程式の解の公式に出てくるルートの中の式で
1)二次方程式の解の個数を調べる時に使用する。態々、解を求めなくてその一部でわかるので便利。
2)二次関数とx軸との共有点の個数を調べる時に使用する。態々、二次方程式を解かなくても(ry
3)その他↑を使った派生問題とか。
765132人目の素数さん:2012/05/03(木) 01:04:01.37
>>760
日本語としては正しいのに、数学的には全くナンセンスだから。
766132人目の素数さん:2012/05/03(木) 01:07:39.39
東大生以外は他人にアドバイスできる立場にないので
勘違いしないように
767132人目の素数さん:2012/05/03(木) 01:08:41.95
質問です。
三角関数の弧度法の単位ラジアンは何時頃から数TTでの履修になったのでしょうか?
数十年前は数TTTでの三角関数の微分で必要だったのでその時の履修でした。
数TTでは三角関数の微分は出てこないので何故、数TTで必要なのかと疑問に思った次第。
768132人目の素数さん:2012/05/03(木) 01:16:07.99
>>767
頑張って探すんだな
http://www.nier.go.jp/guideline/
769132人目の素数さん:2012/05/03(木) 01:32:51.03
>>767
弧度法がIIIに入ったのは1994〜2002だけらしいが、数十年前とはいつのこと?
770132人目の素数さん:2012/05/03(木) 01:41:25.79
>>767
レスありがとうございます。

>数十年前とはいつのこと?
正確な年数は勘弁してほしいけど、もう少し遡る年代です。
771770:2012/05/03(木) 01:43:17.89
失礼アンカミス
>>767>>769氏へのレスです。
772132人目の素数さん:2012/05/03(木) 02:12:17.02
>>771
>>768のリンク先を見ると、指導要領上は弧度法が現れるのは
1960〜 数学I
1982〜 基礎解析
1994〜 数学III
2003〜 数学II
で、数学IIIのときだけ三角関数の微分のところ、
他は一般角のところで出ることになっている。
773132人目の素数さん:2012/05/03(木) 07:45:59.74
          __ノ)-'´ ̄ ̄`ー- 、_
        , '´  _. -‐'''"二ニニ=-`ヽ、
      /   /:::::; -‐''"        `ーノ
     /   /:::::/           \
     /    /::::::/          | | |  |
     |   |:::::/ /     |  | | | |  |
      |   |::/ / / |  | ||  | | ,ハ .| ,ハ|
      |   |/ / / /| ,ハノ| /|ノレ,ニ|ル' 
     |   |  | / / レ',二、レ′ ,ィイ|゙/   私は只の数ヲタなんかとは付き合わないわ。
.     |   \ ∠イ  ,イイ|    ,`-' |      頭が良くて数学が出来てかっこいい人。それが必要条件よ。
     |     l^,人|  ` `-'     ゝ  |        さらに Ann.of Math に論文書けば十分条件にもなるわよ。
      |      ` -'\       ー'  人          一番嫌いなのは論文数を増やすためにくだらない論文を書いて
    |        /(l     __/  ヽ、           良い論文の出版を遅らせるお馬鹿な人。
     |       (:::::`‐-、__  |::::`、     ヒニニヽ、         あなたの論文が Ann of Math に accept される確率は?
    |      / `‐-、::::::::::`‐-、::::\   /,ニニ、\            それとも最近は Inv. Math. の方が上かしら?
   |      |::::::::::::::::::|` -、:::::::,ヘ ̄|'、  ヒニ二、 \
.   |      /::::::::::::::::::|::::::::\/:::O`、::\   | '、   \
   |      /:::::::::::::::::::/:::::::::::::::::::::::::::::'、::::\ノ  ヽ、  |
  |      |:::::/:::::::::/:::::::::::::::::::::::::::::::::::'、',::::'、  /:\__/‐、
  |      |/:::::::::::/::::::::::::::::::::::::::::::::::O::| '、::| く::::::::::::: ̄|
   |     /_..-'´ ̄`ー-、:::::::::::::::::::::::::::::::::::|/:/`‐'::\;;;;;;;_|
   |    |/::::::::::::::::::::::\:::::::::::::::::::::::::::::|::/::::|::::/:::::::::::/
    |   /:::::::::::::::::::::::::::::::::|:::::::::::::::::::::O::|::|::::::|:::::::::::::::/
774132人目の素数さん:2012/05/03(木) 13:42:59.80
http://beebee2see.appspot.com/i/azuYj7CtBgw.jpg

(2)の赤文字で書かれてある部分って部分分数に分けて途中で消すと書いてありますが意味がわかりません。だれか教えてください
775132人目の素数さん:2012/05/03(木) 13:54:13.07
>>774
よくみて+と-でペアになっている部分を探せ
776132人目の素数さん:2012/05/03(木) 14:00:20.47
某RPGで、
「5人のパーティーをつくる。
各メンバーはそれぞれ10種の職業を選べる。」
というのがあるのですが、
そのRPGではメンバーを特に区別しません。
つまり、「剣士 剣士 剣士 剣士 僧侶」と
「剣士 剣士 剣士 僧侶 剣士」は同じとして扱われます。
この場合、全部で組み合わせはいくつになりますか?

メンバーを区別する場合は、10^5になるのはわかるのですが・・・。
777132人目の素数さん:2012/05/03(木) 14:05:34.36
>>776
重複組み合わせ でぐぐれ
778132人目の素数さん:2012/05/03(木) 14:05:39.01
P
779132人目の素数さん:2012/05/03(木) 14:06:37.41
極限の問題で質問です
lim   1/n・log(a^n + a^2n) (0<a≦1)
n→∞

上の問題の解答は 0<a≦1 のとき a^2n≦a^nであるから
log(a^n + a^2n)≦ log (a^n + a^n) = log 2a^n = log2 + n log a
log(a^n + a^2n)≧ log a^n = n log a        ・・・・・・・・・・・・・・・・(*) 
log a ≦ 1/n・log(a^n + a^2n) ≦log2/n + log a
これではさみうちの原理を使って求める極限は log a
です

ここで質問なんですが
(*)の行で log(a^n + a^2n) ≧ log(a^2n + a^2n) = log(2a^2n)
としないのはどういう感覚から来るのでしょうか?
(a^n + a^2n)との差を考えた場合
log a^nとしたほうがlog(a^2n + a^2n)より小さいから?
780779:2012/05/03(木) 14:15:48.33
すみません自己解決しました
781132人目の素数さん:2012/05/03(木) 14:21:55.11
>>776
異なる10個から重複を許して5個を選ぶと考えればよい
782132人目の素数さん:2012/05/03(木) 14:26:31.76
          __ノ)-'´ ̄ ̄`ー- 、_
        , '´  _. -‐'''"二ニニ=-`ヽ、
      /   /:::::; -‐''"        `ーノ
     /   /:::::/           \
     /    /::::::/          | | |  |
     |   |:::::/ /     |  | | | |  |
      |   |::/ / / |  | ||  | | ,ハ .| ,ハ|
      |   |/ / / /| ,ハノ| /|ノレ,ニ|ル' 
     |   |  | / / レ',二、レ′ ,ィイ|゙/   私は只の数ヲタなんかとは付き合わないわ。
.     |   \ ∠イ  ,イイ|    ,`-' |      頭が良くて数学が出来てかっこいい人。それが必要条件よ。
     |     l^,人|  ` `-'     ゝ  |        さらに Ann.of Math に論文書けば十分条件にもなるわよ。
      |      ` -'\       ー'  人          一番嫌いなのは論文数を増やすためにくだらない論文を書いて
    |        /(l     __/  ヽ、           良い論文の出版を遅らせるお馬鹿な人。
     |       (:::::`‐-、__  |::::`、     ヒニニヽ、         あなたの論文が Ann of Math に accept される確率は?
    |      / `‐-、::::::::::`‐-、::::\   /,ニニ、\            それとも最近は Inv. Math. の方が上かしら?
   |      |::::::::::::::::::|` -、:::::::,ヘ ̄|'、  ヒニ二、 \
.   |      /::::::::::::::::::|::::::::\/:::O`、::\   | '、   \
   |      /:::::::::::::::::::/:::::::::::::::::::::::::::::'、::::\ノ  ヽ、  |
  |      |:::::/:::::::::/:::::::::::::::::::::::::::::::::::'、',::::'、  /:\__/‐、
  |      |/:::::::::::/::::::::::::::::::::::::::::::::::O::| '、::| く::::::::::::: ̄|
   |     /_..-'´ ̄`ー-、:::::::::::::::::::::::::::::::::::|/:/`‐'::\;;;;;;;_|
   |    |/::::::::::::::::::::::\:::::::::::::::::::::::::::::|::/::::|::::/:::::::::::/
    |   /:::::::::::::::::::::::::::::::::|:::::::::::::::::::::O::|::|::::::|:::::::::::::::/
783132人目の素数さん:2012/05/03(木) 15:37:25.20
>>772
詳細な調査報告ありがとうございます。
今はあえて正確に調べる必然性がなかったので他力本願になってしまいました。

自分の経験と指導要領とを勘案すると1960以降は履修する教科書にも関係しそうですけど
数学Iで弧度法を履修する内容として差し支えなかった時期があったと言うことですね。

現在、数学IIで履修するのはその後に続く、数学III(微積)、数学C(行列と1次変換)
との兼ね合いで、そのタイミングが効率的なのかと自分では結論に達しました。
784132人目の素数さん:2012/05/03(木) 17:27:53.17
次の等式を満たす有理数p,qの値を求めよ
(1) (√2 -1)p+q√2 = 2+√2
(2) p/(√2 -1) +q/√2 = 1

このタイプの問題は初めてで、
式変形したり√2でくくったりして考えてみたのですが
おそらくそういう解き方じゃないんだと思います
よろしくお願いします

785132人目の素数さん:2012/05/03(木) 17:29:07.49
α = √2 -1
とかおいてみたらどうYO
786132人目の素数さん:2012/05/03(木) 17:31:06.62
>>784
> 式変形したり√2でくくったり
他になにがあんねん
787132人目の素数さん:2012/05/03(木) 17:31:53.55
>>784
有理数 p, q について
p√2 + q = 0 ならば p = q = 0
788132人目の素数さん:2012/05/03(木) 17:55:23.76
(1) (√2 -1)p+q√2 = 2+√2

√2で整理して
(p+q-1)*√2 + (-p-2) = 0
p,qは有理数だから

p+q-1=0 -p-2=0でないとすると√2 = (p+2)/(p+q-1) = 有理数となり題意に反する  (ここの説明はいらない。気があれば自分で厳密に書いて)

p+q-1=0
-p-2=0

p=3 , q=-2

(2)は自力で
789132人目の素数さん:2012/05/03(木) 17:59:36.21
>>785
同じようなことはすでにしましたが分かりませんでした

>>786
何かあると思ったんです

>>787
その式を利用するのだと思ったのですがなかなかうまくいきません
もう少しヒントをお願いします
790132人目の素数さん:2012/05/03(木) 18:12:48.66
あ、多分いけそうです
791132人目の素数さん:2012/05/03(木) 18:16:42.90
>>787の等式を使って連立して解いてみたのですが
qが無理数になってしまったのでだめでした…
>>788
すみません、レスあったの気付いてませんでした
今からやってみます
792132人目の素数さん:2012/05/03(木) 18:16:43.51
787以上はヒントじゃなくて答えだろ(笑)
793132人目の素数さん:2012/05/03(木) 18:27:25.53
わかりました、皆さんありがとうございました。
>>787は大ヒントでしたね
なのに>>788ぐらいまでやってもらわないと気付けなかったなんて
自分のレベルの低さを改めて思い知りました・・・
794132人目の素数さん:2012/05/03(木) 18:40:32.89
解けない、気づけないといっている人は、
こういう問題を解くのに、どのくらい文字や記号を書いて考えているんだろうね。
文字通り、字数のこと。
一行、数文字を書いて、あとは眺めているだけのような気がする。
795132人目の素数さん:2012/05/03(木) 18:44:18.57
説教くさいのはやめとこうや
796132人目の素数さん:2012/05/03(木) 18:47:26.01
最近調子乗った学生がいるな
797132人目の素数さん:2012/05/03(木) 18:49:25.65
春だしな
もうしばらくしたら落ち着くだろう
798132人目の素数さん:2012/05/03(木) 18:54:20.24
おりこうさんはいらないよ
799132人目の素数さん:2012/05/03(木) 19:11:59.08
馬鹿はいらないよ
800132人目の素数さん:2012/05/03(木) 19:28:42.17
東大生以外はこのスレで他人に説教できる立場にないので
勘違いしないように
801132人目の素数さん:2012/05/03(木) 20:08:31.28
東大卒だからと言ってノーベル賞とれるとは限らないし
東大卒でないから言ってノーベル賞とれないとも限らないし。

まぁノーベル賞をとれるような人は2chをやっていないことは確かだと思う。
802132人目の素数さん:2012/05/03(木) 20:09:23.71
a>0で f(p) = p^(-a) exp(-1/p) とします。
pを0に近づけたときのf(p)の極限を求めたいです。
よろしくお願いします。
803132人目の素数さん:2012/05/03(木) 20:11:12.29
          __ノ)-'´ ̄ ̄`ー- 、_
        , '´  _. -‐'''"二ニニ=-`ヽ、
      /   /:::::; -‐''"        `ーノ
     /   /:::::/           \
     /    /::::::/          | | |  |
     |   |:::::/ /     |  | | | |  |
      |   |::/ / / |  | ||  | | ,ハ .| ,ハ|
      |   |/ / / /| ,ハノ| /|ノレ,ニ|ル' 
     |   |  | / / レ',二、レ′ ,ィイ|゙/   私は只の数ヲタなんかとは付き合わないわ。
.     |   \ ∠イ  ,イイ|    ,`-' |      頭が良くて数学が出来てかっこいい人。それが必要条件よ。
     |     l^,人|  ` `-'     ゝ  |        さらに Ann.of Math に論文書けば十分条件にもなるわよ。
      |      ` -'\       ー'  人          一番嫌いなのは論文数を増やすためにくだらない論文を書いて
    |        /(l     __/  ヽ、           良い論文の出版を遅らせるお馬鹿な人。
     |       (:::::`‐-、__  |::::`、     ヒニニヽ、         あなたの論文が Ann of Math に accept される確率は?
    |      / `‐-、::::::::::`‐-、::::\   /,ニニ、\            それとも最近は Inv. Math. の方が上かしら?
   |      |::::::::::::::::::|` -、:::::::,ヘ ̄|'、  ヒニ二、 \
.   |      /::::::::::::::::::|::::::::\/:::O`、::\   | '、   \
   |      /:::::::::::::::::::/:::::::::::::::::::::::::::::'、::::\ノ  ヽ、  |
  |      |:::::/:::::::::/:::::::::::::::::::::::::::::::::::'、',::::'、  /:\__/‐、
  |      |/:::::::::::/::::::::::::::::::::::::::::::::::O::| '、::| く::::::::::::: ̄|
   |     /_..-'´ ̄`ー-、:::::::::::::::::::::::::::::::::::|/:/`‐'::\;;;;;;;_|
   |    |/::::::::::::::::::::::\:::::::::::::::::::::::::::::|::/::::|::::/:::::::::::/
    |   /:::::::::::::::::::::::::::::::::|:::::::::::::::::::::O::|::|::::::|:::::::::::::::/
804132人目の素数さん:2012/05/03(木) 20:16:04.02
>>802
p→0の極限値は無い。p→+0なら1/p=xとおいてx→∞
805802:2012/05/03(木) 20:32:29.04
>>804
すみません。そうですね。pを右から0に近づけたときのf(p)の極限を
調べたいです。そうした場合、f(p)は無限大に発散してしまいますか?

コンピュータでpの値を小さくして言ったらf(p)の値は0に近づくのですが
806132人目の素数さん:2012/05/03(木) 20:41:54.13
x^a/exp(x)→0 x→∞
807132人目の素数さん:2012/05/03(木) 20:44:54.35
そうですね。わかりました。ありがとうございました。
808132人目の素数さん:2012/05/03(木) 21:07:35.96
aを実数として、a≦x≦a+2で定義される関数f(x)=x^2-2x+3の最大値、最小値を
それぞれM(a)、m(a)とする。このとき、関数M(a)、m(a)を求めよ。

という問題で解答に

軸は直線x=1である
a≦x≦a+2の中央は x=a+1
M(a)について
[1]1<a+1すなわちa>0のとき
x=a+2で最大となり・・・・・

と続くのですが
自分の解答には
a≦1<a+1 すなわち0<a≦1のとき・・・

って感じで書いたのですが 「a≦」 って言う表記はなぜいらないのでしょうか。
809132人目の素数さん:2012/05/03(木) 21:15:33.42
分数関数の極値を求める時にf(x)=g(x)/h(x)がx=αで極値をとり
h'(α)=0でないならば

f(α)=g'(α)/h'(α)であるっていうのは

問題でとくに明記せずf(α)=g'(α)/h'(α)なのでf(α)=・・・・


みたいな感じで使って大丈夫でしょうか。
810132人目の素数さん:2012/05/03(木) 21:18:49.20
>>808
そこで分ける必要がないから。
811132人目の素数さん:2012/05/03(木) 21:19:12.53
>>808
最大値を考えるに当たり
評価するのは
定義域の真ん中a+1が 1より大きいか小さいかだけじゃないの?

自信ないなら自分で考えられる全ての場合を考慮して答案を作ればいい。
それを踏まえて自分の解答と問題集(?)の解答と比べればいい
812132人目の素数さん:2012/05/03(木) 21:43:15.53
          __ノ)-'´ ̄ ̄`ー- 、_
        , '´  _. -‐'''"二ニニ=-`ヽ、
      /   /:::::; -‐''"        `ーノ
     /   /:::::/           \
     /    /::::::/          | | |  |
     |   |:::::/ /     |  | | | |  |
      |   |::/ / / |  | ||  | | ,ハ .| ,ハ|
      |   |/ / / /| ,ハノ| /|ノレ,ニ|ル' 
     |   |  | / / レ',二、レ′ ,ィイ|゙/   私は只の数ヲタなんかとは付き合わないわ。
.     |   \ ∠イ  ,イイ|    ,`-' |      頭が良くて数学が出来てかっこいい人。それが必要条件よ。
     |     l^,人|  ` `-'     ゝ  |        さらに Ann.of Math に論文書けば十分条件にもなるわよ。
      |      ` -'\       ー'  人          一番嫌いなのは論文数を増やすためにくだらない論文を書いて
    |        /(l     __/  ヽ、           良い論文の出版を遅らせるお馬鹿な人。
     |       (:::::`‐-、__  |::::`、     ヒニニヽ、         あなたの論文が Ann of Math に accept される確率は?
    |      / `‐-、::::::::::`‐-、::::\   /,ニニ、\            それとも最近は Inv. Math. の方が上かしら?
   |      |::::::::::::::::::|` -、:::::::,ヘ ̄|'、  ヒニ二、 \
.   |      /::::::::::::::::::|::::::::\/:::O`、::\   | '、   \
   |      /:::::::::::::::::::/:::::::::::::::::::::::::::::'、::::\ノ  ヽ、  |
  |      |:::::/:::::::::/:::::::::::::::::::::::::::::::::::'、',::::'、  /:\__/‐、
  |      |/:::::::::::/::::::::::::::::::::::::::::::::::O::| '、::| く::::::::::::: ̄|
   |     /_..-'´ ̄`ー-、:::::::::::::::::::::::::::::::::::|/:/`‐'::\;;;;;;;_|
   |    |/::::::::::::::::::::::\:::::::::::::::::::::::::::::|::/::::|::::/:::::::::::/
    |   /:::::::::::::::::::::::::::::::::|:::::::::::::::::::::O::|::|::::::|:::::::::::::::/
813132人目の素数さん:2012/05/03(木) 21:50:24.88
>>809
単なる計算なので結果あってればokと思う
814132人目の素数さん:2012/05/03(木) 22:57:56.85
長さ2の線分ABを直径とする半円上に点P,Qをとり、四辺形ABQCをつくる。
ABの中点をOとし、角AOP=2α、角POQ=2β、角QOB=γ、l=AP+PQ+QBとする
lが最大になるのはα=β=γ=π/6の時であることを示せ

解答でγをα、βであらわしてニ文字にして、βで固定してαを動かしている(α、βが独立に動く)のですが、何故独立に動くのでかすか?
αを決めればβも決まりませんか?
あと、l=2(sinα+sinβ+sinγ)として計算していくと、この最大値が2sin(π/4-β/2)+sinβまで変形できて、ことあとどう変形すればπ/6のときが最大になることを示せるのかも分かりません

お願いします
815132人目の素数さん:2012/05/03(木) 23:03:49.32
>>814
いろいろ考えての質問だと思いますが
もう一度、読み返して問題文を正確に書いてください。

四辺形ABQC → ABQP ?
角AOP=2α、角POQ=2β、角QOB=γ → 角AOP=2α、角POQ=2β、角QOB=2γ ?
816132人目の素数さん:2012/05/03(木) 23:10:53.71
相加相乗平均の不等式を証明せよという問題で
(a+b)/2≧√ab
をどんどん同値変形していったら(a-b)^2≧0という当たり前の式が出てきたので
これをもって証明終了にしたんですがアウトですか?
結論式を使ってはいけない などと習ったんですが、結論式を完全に同値変形させて
正しい式が出てきちゃったんで、もともとの証明すべき不等式も当然成り立っている
という結論にしてはだめなのすか
817132人目の素数さん:2012/05/03(木) 23:13:02.20
>>816
もちろんいいんだけど、各段階で同値であると自覚していることが、採点者に伝わるようにね
818132人目の素数さん:2012/05/03(木) 23:14:34.55
>>814
訂正しました
指摘されたところを間違えていました
すみません

>長さ2の線分ABを直径とする半円周上に点P,Qをとり、四辺形APQBをつくる。
>ABの中点をOとし、角AOP=2α、角POQ=2β、角QOB=2γ、l=AP+PQ+QBとする
>lが最大になるのはα=β=γ=π/6の時であることを示せ

>解答でγをα、βであらわしてニ文字にして、βで固定してαを動かしている(α、βが独立に動く)のですが、何故独立に動くのでかすか?
>αを決めればβも決まりませんか?
>あと、l=2(sinα+sinβ+sinγ)として計算していくと、この最大値が2sin(π/4-β/2)+sinβまで変形できて、ことあとどう変形すればπ/6のときが最大になることを示せるのかも分かりません

>お願いします
819132人目の素数さん:2012/05/03(木) 23:15:48.59
>>817
ありがとうございます。
「証明では結論式を使ってはいけない」という教師の説明にとても違和感を感じてたとこです。
820132人目の素数さん:2012/05/03(木) 23:16:30.25
>>816
(a+b)/2≧√ab は a≧0 かつ b≧0 でないと成り立たないが
(a-b)^2≧0 は a<0 または b<0 でも成り立つから
多分同値変形してない。
821132人目の素数さん:2012/05/03(木) 23:22:14.00
>>820
aho
822132人目の素数さん:2012/05/03(木) 23:28:12.77
>>821
めっ!(*´д`*)
823132人目の素数さん:2012/05/03(木) 23:30:27.39
xy座標平面上において2点(5、0)および(3、6)から等しい距離にある、原点を通る直線lについて、
l:y=mxとするとx=0が含まれないことを注意しなければいけない

なぜ、x=0(y軸)のときは含まれないのでしょうか?
詳しい説明をお願いします
824132人目の素数さん:2012/05/03(木) 23:33:35.82
そうではなくて
原点を通る直線のうちx=0だけはl:y=mxでは扱えないから
x=0については別に検討しなくてはいけないということ
825132人目の素数さん:2012/05/03(木) 23:37:11.74
x=0のときだけy=mxでは扱えない

この部分の詳しい説明をお願いします
826132人目の素数さん:2012/05/03(木) 23:38:04.33
y=mxという方程式はmをいくら変えてもx軸に垂直な直線を表さない。((0,0)と(1,m)を通る直線になる。)
x=0という方程式は原点を通るx軸に垂直な直線を表している。
827132人目の素数さん:2012/05/03(木) 23:39:46.85
>>816
>相加相乗平均の不等式を証明せよという問題で
>(a+b)/2≧√ab
>をどんどん同値変形していったら

だめだろ…
作法の問題だから。
828132人目の素数さん:2012/05/03(木) 23:39:59.07
          __ノ)-'´ ̄ ̄`ー- 、_
        , '´  _. -‐'''"二ニニ=-`ヽ、
      /   /:::::; -‐''"        `ーノ
     /   /:::::/           \
     /    /::::::/          | | |  |
     |   |:::::/ /     |  | | | |  |
      |   |::/ / / |  | ||  | | ,ハ .| ,ハ|
      |   |/ / / /| ,ハノ| /|ノレ,ニ|ル' 
     |   |  | / / レ',二、レ′ ,ィイ|゙/   私は只の数ヲタなんかとは付き合わないわ。
.     |   \ ∠イ  ,イイ|    ,`-' |      頭が良くて数学が出来てかっこいい人。それが必要条件よ。
     |     l^,人|  ` `-'     ゝ  |        さらに Ann.of Math に論文書けば十分条件にもなるわよ。
      |      ` -'\       ー'  人          一番嫌いなのは論文数を増やすためにくだらない論文を書いて
    |        /(l     __/  ヽ、           良い論文の出版を遅らせるお馬鹿な人。
     |       (:::::`‐-、__  |::::`、     ヒニニヽ、         あなたの論文が Ann of Math に accept される確率は?
    |      / `‐-、::::::::::`‐-、::::\   /,ニニ、\            それとも最近は Inv. Math. の方が上かしら?
   |      |::::::::::::::::::|` -、:::::::,ヘ ̄|'、  ヒニ二、 \
.   |      /::::::::::::::::::|::::::::\/:::O`、::\   | '、   \
   |      /:::::::::::::::::::/:::::::::::::::::::::::::::::'、::::\ノ  ヽ、  |
  |      |:::::/:::::::::/:::::::::::::::::::::::::::::::::::'、',::::'、  /:\__/‐、
  |      |/:::::::::::/::::::::::::::::::::::::::::::::::O::| '、::| く::::::::::::: ̄|
   |     /_..-'´ ̄`ー-、:::::::::::::::::::::::::::::::::::|/:/`‐'::\;;;;;;;_|
   |    |/::::::::::::::::::::::\:::::::::::::::::::::::::::::|::/::::|::::/:::::::::::/
    |   /:::::::::::::::::::::::::::::::::|:::::::::::::::::::::O::|::|::::::|:::::::::::::::/
829132人目の素数さん:2012/05/03(木) 23:47:08.74
a>0,b>0のもとで、
(a+b)/2≧√abと(a-b)^2≧0が同値であることを示し、後者が成り立つから前者も成り立つ。


この答案の数学的に間違いである理由があるのか?
830132人目の素数さん:2012/05/03(木) 23:50:11.20
>>826
方程式y=mxの右辺にx=0を代入するとy=0が得られる
すなわち、mがいかなる値をとろうともx=0のときにyが0以外の実数をとることはあり得ないため(原点を表してしまう)、
直線y=mxはx=0とはならない

て解釈でよいでしょうか?
831132人目の素数さん:2012/05/03(木) 23:55:59.85
>>819
証明する途中過程では結論式を使わない方が無難。
証明しようとする結論式を途中で使っていたら循環論法的な表現になり要らぬ誤解をもたらす。
832132人目の素数さん:2012/05/04(金) 00:06:17.30
>>829
証明する途中過程で↓のような式も現れるから要らぬ誤解をもたらす。
(a+b)/2-√ab ≧ 0

(a+b)/2-√ab=[(a-b)^2]/2 と変形してから 
したがって[(a-b)^2]/2 ≧ 0 であるとした方が素直な表現に思うけど。
833132人目の素数さん:2012/05/04(金) 00:10:28.73
>>832
イミフ
834132人目の素数さん:2012/05/04(金) 00:19:28.64
まぁ好きにすればいいじゃん。
つまらん意地はって茨の道に足を突っ込む事は推奨はしないけど、本人が絶対の自身を持ってやると言うなら止めないよ。
835132人目の素数さん:2012/05/04(金) 00:19:32.88
>>818
今与えられている式は
α+β+γ=π/2
のみ
αを固定したところでβ,γの2変数が残ります。
実際図を描いて考えた方がわかりやすいと思います。

l/2 = sinα+sinβ+sinγ
= sinα+sinβ+sin(α+β)
= (1+cosβ)sinα+sinβ*cosα + sinβ
≦ √{(1+cosβ)^2+sinβ^2} + sinβ
= √(2+2cosβ) + sinβ
= 2cosβ/2 + sinβ

じゃないか?
後は微分すれば出来そうだけど。
836132人目の素数さん:2012/05/04(金) 00:23:34.14
>>833
(√(a)-√(b))^2≧0 と書きたかったことくらい察してやれよ。
 
837132人目の素数さん:2012/05/04(金) 00:27:31.49
>>836
アリガト、その通り。
838132人目の素数さん:2012/05/04(金) 00:29:01.08
>>818
余談で
α+β+γ=π/2 の時
l/2 = sinα+sinβ+sinγ
≦ 3sin((α+β+γ)/3)
= 3/2

凸関数y=sinxのグラフと
3点(α,sinα)(β,sinβ)(γ,sinγ)でできる三角形の重心と
点((α+β+γ)/3,sin((α+β+γ)/3))
の位置を考慮すればわかる。
839837:2012/05/04(金) 00:30:50.78
っていうか>>829氏の式を信用してコピーしたことが原因だ、もっとも前に指摘すれよ。
840132人目の素数さん:2012/05/04(金) 00:31:30.68
>>836
バカが増えた
841132人目の素数さん:2012/05/04(金) 00:32:57.27
>>839
>>829は間違ってないよ
842132人目の素数さん:2012/05/04(金) 00:48:22.04
>>841
間違っていないかも知れないけど「(a+b)/2-√ab」と「(a-b)^2」とでは明らかに違う式なので
今度は同値であることの証明が必要。問題は↑の式が同値であることの証明でないから。

寄り道とか回り道が好きだったとか? お母さんによくしかられたろうな。
843132人目の素数さん:2012/05/04(金) 00:53:10.93
>>842
なんで話すりかえてるの
>>829は正しくて>>832は間違ってるっていってんだが
844132人目の素数さん:2012/05/04(金) 01:01:41.49
お母さんw
845132人目の素数さん:2012/05/04(金) 01:33:23.72
>>843
書いてある通りのことしか認めないとう意味で、アホ。
846132人目の素数さん:2012/05/04(金) 01:46:33.06

>>832が間違えたのは>>829が間違えてるからだ。>>829の間違いを指摘しろ。」

>>829は間違えてないよ。」

>>829は回り道だから>>829の間違いを指摘しろ。」

847132人目の素数さん:2012/05/04(金) 01:49:28.48
もっとも前に
認めないとう意味
848132人目の素数さん:2012/05/04(金) 02:43:48.11
連続する奇数の和が6通りある最小の数を求めよという問題です。
432が6通りであらわせるのはわかりましたが、それより小さいものはあるのでしょうか?
誘導は、72は何通りであらわせるか(3通り)でした。
849132人目の素数さん:2012/05/04(金) 03:20:52.62
6x+2+4+6+8+10=6x+30
3x+14+3x+16
2x+8+2x+10+2x+12
x+y+x+y+2+x+y+4+x+y+6,4y+12=2x+30
5(x+y)+20,5y+20=x+30
6(x+y)+30,6y+30=30
850132人目の素数さん:2012/05/04(金) 08:12:05.02
n(x+y)+2(n-1)=7x+42
2(x+y)+2,2y+2=5x+42,y=2x+20,x=1,y=22,23,25=48;41,43=84;84+4p
3(x+y)+6,3y+6=4x+42,3y=4x+36,x=3,y=16,19,21=40;27,29,31=87;87+6p
4(x+y)+12,4y+12=2x+42,2y=x+15,x=1,y=16,17,19=36;17,19,21,23=80;80+8p
5(x+y)+20,5y+20=2x+42,5y=2x+22,x=9,y=8,17,19,21,23,25=105;105+10p
6(x+y)+30,6y+30=x+42,6y=x+12,x=6,y=3,9,11,13,15,17,19=84+12p
7(x+y)+42,7y+42=42,y=0,x=1,7,9,11,13,15,17,19=91+14p

2x+2+4p=3x+6+6p=4x+12+8p=5x+20+10p=6x+30+12p=7x+42+14p
2x+4p=3x+4+6p=4x+10+8p=5x+18+10p=6x+28+12p=7x+40+14p
851132人目の素数さん:2012/05/04(金) 09:45:57.12
          __ノ)-'´ ̄ ̄`ー- 、_
        , '´  _. -‐'''"二ニニ=-`ヽ、
      /   /:::::; -‐''"        `ーノ
     /   /:::::/           \
     /    /::::::/          | | |  |
     |   |:::::/ /     |  | | | |  |
      |   |::/ / / |  | ||  | | ,ハ .| ,ハ|
      |   |/ / / /| ,ハノ| /|ノレ,ニ|ル' 
     |   |  | / / レ',二、レ′ ,ィイ|゙/   私は只の数ヲタなんかとは付き合わないわ。
.     |   \ ∠イ  ,イイ|    ,`-' |      頭が良くて数学が出来てかっこいい人。それが必要条件よ。
     |     l^,人|  ` `-'     ゝ  |        さらに Ann.of Math に論文書けば十分条件にもなるわよ。
      |      ` -'\       ー'  人          一番嫌いなのは論文数を増やすためにくだらない論文を書いて
    |        /(l     __/  ヽ、           良い論文の出版を遅らせるお馬鹿な人。
     |       (:::::`‐-、__  |::::`、     ヒニニヽ、         あなたの論文が Ann of Math に accept される確率は?
    |      / `‐-、::::::::::`‐-、::::\   /,ニニ、\            それとも最近は Inv. Math. の方が上かしら?
   |      |::::::::::::::::::|` -、:::::::,ヘ ̄|'、  ヒニ二、 \
.   |      /::::::::::::::::::|::::::::\/:::O`、::\   | '、   \
   |      /:::::::::::::::::::/:::::::::::::::::::::::::::::'、::::\ノ  ヽ、  |
  |      |:::::/:::::::::/:::::::::::::::::::::::::::::::::::'、',::::'、  /:\__/‐、
  |      |/:::::::::::/::::::::::::::::::::::::::::::::::O::| '、::| く::::::::::::: ̄|
   |     /_..-'´ ̄`ー-、:::::::::::::::::::::::::::::::::::|/:/`‐'::\;;;;;;;_|
   |    |/::::::::::::::::::::::\:::::::::::::::::::::::::::::|::/::::|::::/:::::::::::/
    |   /:::::::::::::::::::::::::::::::::|:::::::::::::::::::::O::|::|::::::|:::::::::::::::/
852132人目の素数さん:2012/05/04(金) 10:22:03.32
凄く手間がかかるやり方な上に不完全な解答なんだけど。

奇数を奇数回足したら奇数
奇数を偶数回足したら偶数
この2つの方法で共通の和になる事は無いので別々に考える。

・連続する奇数を偶数回足した場合
連続する2つの奇数の和=4*1(m+1) →8以上の4(=2*2)の倍数
連続する4つの奇数の和=4*2(m+2) →24以上の8(=2*2*2)の倍数
連続する6つの奇数の和=4*3(m+3) →48以上の12(=2*2*3)の倍数
連続する8つの奇数の和=4*4(m+4) →80以上の16(=2*2*2*2)の倍数
連続する10つの奇数の和=4*5(m+5) →120以上の20(=2*2*5)の倍数
連続する12つの奇数の和=4*6(m+6) →168以上の24(=2*2*2*3)の倍数
連続する14つの奇数の和=4*7(m+7) →224以上の28(=2*2*7)の倍数
連続する16つの奇数の和=4*8(m+8) →288以上の32(=2*2*2*2*2)の倍数
連続する18つの奇数の和=4*9(m+9) →360以上の36(=2*2*3*3)の倍数
連続する20つの奇数の和=4*10(m+10) →440以上の40(=2*2*2*5)の倍数 これ以上は最小値が432を超えてしまうので止め
(※連続する2nつの奇数の和=4*n(m+n) →n(n+1)以上の4nの倍数)

この中の6つのパターンに合致する最小の組み合わせは144(=2*2*2*2*3*3)の倍数で360以上
→→連続する奇数の和で表す方法が6通りある最小の偶数は432


奇数の場合はグチャグチャになりすぎてわからんかった
俺も知りたいので、誰かスマートな証明方法教えて下さい。
853132人目の素数さん:2012/05/04(金) 10:23:37.37
>>846
既に>>832での間違いは認めている。

次の話題に移るけど
>「>>829は回り道だから>>829の間違いを指摘しろ。」
その前に↓を証明してくれないか? 結局、説明が増え手間が増えるだけと思うけど。
「a>0,b>0のもとで、(a+b)/2≧√abと(a-b)^2≧0が同値である」

その見通しもできないのかな、ヤレヤレ。
(a+b)/2-√ab=[(√a-√b)^2]/2 と完全平方式に変形して
結論と同値の (√a-√b)^2≧0 を得る。

↑を証明した後でないと↓で同値と言い張るには論理の飛躍が生じる。
「a>0,b>0のもとで、(a+b)/2≧√abと(a-b)^2≧0が同値である」

証明問題での二度手間ってなんだろうな・・・なので寄り道とか回り道が好きだったのか? って聞いた。
それでも世間は広いから相加平均・相乗平均の関係で↓を使った教科書なり参考書があれば紹介を頼む。
「a>0,b>0のもとで、(a+b)/2≧√abと(a-b)^2≧0が同値である」
854132人目の素数さん:2012/05/04(金) 10:25:40.17
アンカー付け忘れた
>>852>>848の問題です
855132人目の素数さん:2012/05/04(金) 10:50:49.44
……ボケているのは俺なのか?
119+121
57+59+61+63
35+37+39+41+43+45
23+25+27+29+31+33+35+37
15+17+19+21+23+25+27+29+31+33
9+11+13+15+17+19+21+23+25+27+29+31
856132人目の素数さん:2012/05/04(金) 10:51:18.79

袋の中にn個(n≧2)の球があり、内2個が白球、それ以外は全て黒球とする
この袋の中から、一つずつ無作為に取り出す、但し取り出した球は元に戻さない

2個目の白球が出るまでの回数をXとするとき、
(1):Xの確率分布を求めよ
(2):Xの平均E(X)を求めよ

こういう問題は、今高校でやりますかね?
857132人目の素数さん:2012/05/04(金) 11:03:15.92
奇数なら
315
103+105+107
59+61+63+65+67
39+41+43+45+47+49+51
27+29+31+33+35+37+39+41+43
7+9+11+13+15+17+19+21+23+25+27+29+31+33+35

あるいは
383+385+387
227+229+231+233+235
159+161+163+165+167+169+171
95+97+99+101+103+105+107+109+111+113+115
35+37+39+41+43+45+47+49+51+53+55+57+59+61+63+65+67+69+71+73+75
3+5+7+9+11+13+15+17+19+21+23+25+27+29+31+33+35+
37+39+41+43+45+47+49+51+53+55+57+59+61+63+65+67
858132人目の素数さん:2012/05/04(金) 11:08:00.90
>>856
選択する人は少ないだろうな。
859132人目の素数さん:2012/05/04(金) 11:09:37.78
>>856
一応高校範囲内じゃない?

ただ大学入試で頻度よく出る問題でもないから
実際の教育の場で教えられてるかは不明だけど。

ttp://www.mext.go.jp/b_menu/shuppan/sonota/990301d/990301e.htm
860132人目の素数さん:2012/05/04(金) 11:24:25.90
>>848
そのとある整数aを、四角数より a =N^2-M^2 (連続する奇数)(N>M>0) と表して、
さらに a = (N+M)(N-M) = p*q とおけば、
題意より p-q は偶数であり、
差を偶数にするために、pとqは、少なくとも一つずつの2を含む。

例えば 72=2*2*2*3*3 はp,qを偶数にするために2個の2を「予約」しておく。
72=(2*2) * (2*3*3)
つまり、(2*3*3)の分解による組み合わせは下の3通りある。
1,  2*3*3
2,  3*3
2*3,  3

ここで題意より、
a = (2*2) * (????)
の (????) の部分で出来上がる整数が6通りになるような素因数分解が入る

例えば432なら
432 = (2*2) * (2*2*3*3*3) になって、
(2*2*3*3*3) の分解の組み合わせが6通り、出来上がるp,qも6通り
861132人目の素数さん:2012/05/04(金) 11:58:25.15
そうか四角数か。スマートだわ
862132人目の素数さん:2012/05/04(金) 12:06:09.29
ミスってた…
313+315+317
185+187+189+191+193
129+131+133+135+137+139+141
97+99+101+103+105+107+109+111+113
49+51+53+55+57+59+61+63+65+67+69+71+73+75+77
25+27+29+31+33+35+37+39+41+43+45+47+49+51+53+55+57+59+61+63+65
863132人目の素数さん:2012/05/04(金) 15:01:58.64
因数分解の問題
x^2-2xy-3y^2+x+5y-2

どうしても解けないのでよろしくお願いします
864132人目の素数さん:2012/05/04(金) 16:00:34.76
時数の低いも時に
ついて生理
865132人目の素数さん:2012/05/04(金) 16:00:54.95
次数の低い文字について整理しろ
866132人目の素数さん:2012/05/04(金) 16:01:16.24
>>863
次数の低い文字について整理しろ
教科書ついてるぞ?
867132人目の素数さん:2012/05/04(金) 16:01:52.59
次数の低い文字で整理が鉄板>>863
868132人目の素数さん:2012/05/04(金) 16:55:15.13
>>866-867
解決しましたありがとうございます
869132人目の素数さん:2012/05/04(金) 22:18:47.86
全くわからないです…http://beebee2see.appspot.com/i/azuYxKGtBgw.jpg
因数分解です
870132人目の素数さん:2012/05/04(金) 22:24:26.51
x^2 を A など置いてみてはいかがでしょう
きっとあなたの望む何かが手に入るかと思います
871132人目の素数さん:2012/05/04(金) 22:27:16.84
>>869
典型問題だ
x^4 - 7x^2 + 9
= x^4 - 6x^2 + 9 - x^2
872132人目の素数さん:2012/05/04(金) 22:30:58.48
x^4 - 7x^2 + 9
= x^4 + 6x^2 + 9 - 13x^2
= ( x^2 + 3 )^2 - ( (√13) x)^2
= ( x^2 + (√13)x + 3 )( x^2 -(√13)x + 3 )

とするのもいいですか?
873132人目の素数さん:2012/05/04(金) 22:32:51.80
f(x)=x^3+ax^2+bx+c (a、b、cは実数)であるとき
f(f(x))-xがf(x)-xで割りきれることを示せ。

という問題で

f(x)-x=g(x)っておいて代入→展開→g(x)でくくると
f(f(x))-x=
g(x){g(x)^2+3g(x)+3x^2+ag(x)+2ax+b}

となったので示せたと思ったのですが、上式の場合a、bが整数でないと駄目だ、と言われました。
なぜa、bが整数でないといけないのですか?
874132人目の素数さん:2012/05/04(金) 22:35:55.16
>>870.871.872

ありがとうございます!!

872さん因数分解でルート使ってもいいんですか?無知で申し訳ない;
875132人目の素数さん:2012/05/04(金) 22:38:56.48
>>874
だめとは言わないけど係数は整数くらいでするのが暗黙の了解
876132人目の素数さん:2012/05/04(金) 22:39:01.69
          __ノ)-'´ ̄ ̄`ー- 、_
        , '´  _. -‐'''"二ニニ=-`ヽ、
      /   /:::::; -‐''"        `ーノ
     /   /:::::/           \
     /    /::::::/          | | |  |
     |   |:::::/ /     |  | | | |  |
      |   |::/ / / |  | ||  | | ,ハ .| ,ハ|
      |   |/ / / /| ,ハノ| /|ノレ,ニ|ル' 
     |   |  | / / レ',二、レ′ ,ィイ|゙/   私は只の数ヲタなんかとは付き合わないわ。
.     |   \ ∠イ  ,イイ|    ,`-' |      頭が良くて数学が出来てかっこいい人。それが必要条件よ。
     |     l^,人|  ` `-'     ゝ  |        さらに Ann.of Math に論文書けば十分条件にもなるわよ。
      |      ` -'\       ー'  人          一番嫌いなのは論文数を増やすためにくだらない論文を書いて
    |        /(l     __/  ヽ、           良い論文の出版を遅らせるお馬鹿な人。
     |       (:::::`‐-、__  |::::`、     ヒニニヽ、         あなたの論文が Ann of Math に accept される確率は?
    |      / `‐-、::::::::::`‐-、::::\   /,ニニ、\            それとも最近は Inv. Math. の方が上かしら?
   |      |::::::::::::::::::|` -、:::::::,ヘ ̄|'、  ヒニ二、 \
.   |      /::::::::::::::::::|::::::::\/:::O`、::\   | '、   \
   |      /:::::::::::::::::::/:::::::::::::::::::::::::::::'、::::\ノ  ヽ、  |
  |      |:::::/:::::::::/:::::::::::::::::::::::::::::::::::'、',::::'、  /:\__/‐、
  |      |/:::::::::::/::::::::::::::::::::::::::::::::::O::| '、::| く::::::::::::: ̄|
   |     /_..-'´ ̄`ー-、:::::::::::::::::::::::::::::::::::|/:/`‐'::\;;;;;;;_|
   |    |/::::::::::::::::::::::\:::::::::::::::::::::::::::::|::/::::|::::/:::::::::::/
    |   /:::::::::::::::::::::::::::::::::|:::::::::::::::::::::O::|::|::::::|:::::::::::::::/
877132人目の素数さん:2012/05/04(金) 22:42:40.84
>>875
ありがとうございます、また一つ賢くなりました!!
878132人目の素数さん:2012/05/04(金) 22:50:10.30
>>873
さあ・・・・
879132人目の素数さん:2012/05/04(金) 23:58:43.25
>>873
計算力スゴすぎ、正直言ってついて行けない。

なので↓は解らない。
>なぜa、bが整数でないといけないのですか?

だけど自分が高校数学で唯一つまずいた剰余(因数)の定理、の応用だと思うので・・・
「f(f(x))-x」と言う言う式を「f(x)-x」と言う式で割った「余り」は
結局「f(f(x))-x」の内側のf(x)にxを代入して「f(x)-x」にならないかな。
この最後の式の余りは当然「f(x)-x」で割り切れると思うけど、どうかな?

まぁ(a、b、c)に適切でかつ具体的な数値を当てはめて検証して下さい。
880132人目の素数さん:2012/05/05(土) 00:03:39.05
>>873
先生がアホだから
青チャートからやり直してくださいって言っとけ
881132人目の素数さん:2012/05/05(土) 02:49:20.20
>>873
標準的な解法は
f(f(x))-x=( f(f(x))-f(x) )+( f(x)-x )
と置いて、f(f(x))-f(x) が f(x)-x で割りきれることを示すんじゃないかな。
つまり、
f(x)^3+a f(x)^2+b f(x)+c-( x^3+a x^2+b x+c )=( f(x)^3-x^3 )+a( f(x)^2-x^2 )+b( f(x)-x )
が f(x)-x で割りきれることを示すわけで、これは自明と。
当然、a,b,c に制限など無い。
882132人目の素数さん:2012/05/05(土) 05:32:01.10
f(x、y)のグラフをy=xに関して対象移動したらf(y、x)になる証明が載ってる
サイトしりませんか
883132人目の素数さん:2012/05/05(土) 05:51:34.68
>>882
探せばあるだろうけど
それぐらいならいわゆる逆手流ですぐ示せる

点 P ( x , y ) が 点 Q ( X , Y ) に移るとすると
   X = y , Y = x
   ∴ x = Y , y = X   ←移動前を移動後で表して
これを f( x , y )= 0 代入して   ←移動前が満たす式に代入
   f( Y , X )= 0
あとは大文字を小文字に改めればよい
884132人目の素数さん:2012/05/05(土) 08:21:42.28
>>883
ばかか?
885132人目の素数さん:2012/05/05(土) 09:00:18.10
(x,y)+a(1,-1)がy=x上にあると表せるので(aは実数(1,-1)は対称軸の法線ベクトル)
よってx+a=y-a
対称の点は
(x,y)+2a(1,-1)であらわせる
(x+2a,y-2a)=(y-a+a,x+a-a)=(y,x)
886132人目の素数さん:2012/05/05(土) 12:39:35.07
すいません、確率の問題なのですが教えてください(>_<)


1から12までの番号のついた赤玉が12個、同様に1から12までの番号のついた白玉が12個、合計24個の玉が入った袋があります。この袋から順番に5個の玉を取り出すとき、次の確率を求めなさい。 ただし、一度取り出した玉は袋に戻さないこととする。

(1) 5個目の玉を取り出したときに初めて同じ番号の赤玉と白玉が出る確率

(2) 取り出した5個の玉のうち、同じ番号の赤玉と白玉が1組だけ含まれる確率。

(3) 3個目に取り出した玉の番号よりも5個目に取り出した玉の番号の方が大きくなる確率。


よろしくおねがいしますm(_ _)m
887132人目の素数さん:2012/05/05(土) 12:54:55.24
(1)(2)はゴリ押しで

(1)
(1個目はどれでも良い)×(2個目がダブらない確率)×(3個目がダブらない確率)×(4個目がダブらない確率)×(5個目でダブる確率)

=1*(22/23)*(20/22)*(18/21)*(4/20)

(2)
(5個目でダブる確率)
+(4個目でダブって5個目はダブらない確率)
+(3個目でダブって4,5個目はダブらない確率)
+(2個目でダブって3,4,5個目はダブらない確率)

=1*(22/23)*(20/22)*(18/21)*(4/20)
+1*(22/23)*(20/22)*(3/21)*(18/20)
+1*(22/23)*(2/22)*(20/21)*(18/20)
+1*(1/23)*(22/22)*(20/21)*(18/20)

(3)はシラネ
888132人目の素数さん:2012/05/05(土) 13:00:31.29
>>886

どこまで考えたのか書こうね

>(1) 5個目の玉を取り出したときに初めて同じ番号の赤玉と白玉が出る確率

逆に言えば1から4個目の数字は全部違ってそれぞれ赤白考えられるって事で、五個目は四個のうちのどれかがでたって事だよね

>(2) 取り出した5個の玉のうち、同じ番号の赤玉と白玉が1組だけ含まれる確率。

数字が四種を選ぶ。そのうちどれか一つが二つある。それを並びかえたらいいね

>(3) 3個目に取り出した玉の番号よりも5個目に取り出した玉の番号の方が大きくなる確率。

これは難しいね。上手いやり方うかばないから説明が面倒。ただ、同じ数字がない場合、一組ある場合二組ある場合って考えると(1)(2)の考えが生きるかな。
889132人目の素数さん:2012/05/05(土) 13:37:36.24
どんだけ空行入れんねん
890132人目の素数さん:2012/05/05(土) 13:38:01.30
ってか、13.650が必要な状況でなんで難易度落としたん?
891132人目の素数さん:2012/05/05(土) 13:58:36.04
>>887
>>888
ご丁寧にどうもありがとうございました!!
(3)に関しては、がんばってみます!(>_<)
892132人目の素数さん:2012/05/05(土) 19:25:49.62
問題じゃないですけど誰か詳しく説明お願いします

場合の数の約数の個数と和についてです
約数・倍数の問題は素因数分解からスタート
12=2^2*3
(1+2+2^2)(1+3)=1*1+1*3+2*1+2*3+2^2*1+2^2*3
右辺の各項はどれも2^3の正の約数であり・・・・

って感じで続くのですが、2^2という部分を(1+2+2^2) と足してるのは
なんでなのでしょうか。
893132人目の素数さん:2012/05/05(土) 19:52:14.92
>>892
直線 x = 0 , x = 2^0 , x = 2^1 , x = 2^2 ,
    y = 0 , y = 3^0 , y = 3^1 を描く
各小長方形の面積が12の約数になる
これらの個数,総和を考えればよい
894132人目の素数さん:2012/05/05(土) 19:52:34.20
>>892
ちゃんと左辺を展開したら12の約数が出てくるだろ?
12の約数の和を求める問題だろ?
895132人目の素数さん:2012/05/05(土) 19:55:53.87
>>892
12の約数は
2^a*3^b
a;0,1,2 b;0,1
と書けるのはわかるのか?
896132人目の素数さん:2012/05/05(土) 20:02:43.92
          __ノ)-'´ ̄ ̄`ー- 、_
        , '´  _. -‐'''"二ニニ=-`ヽ、
      /   /:::::; -‐''"        `ーノ
     /   /:::::/           \
     /    /::::::/          | | |  |
     |   |:::::/ /     |  | | | |  |
      |   |::/ / / |  | ||  | | ,ハ .| ,ハ|
      |   |/ / / /| ,ハノ| /|ノレ,ニ|ル' 
     |   |  | / / レ',二、レ′ ,ィイ|゙/   私は只の数ヲタなんかとは付き合わないわ。
.     |   \ ∠イ  ,イイ|    ,`-' |      頭が良くて数学が出来てかっこいい人。それが必要条件よ。
     |     l^,人|  ` `-'     ゝ  |        さらに Ann.of Math に論文書けば十分条件にもなるわよ。
      |      ` -'\       ー'  人          一番嫌いなのは論文数を増やすためにくだらない論文を書いて
    |        /(l     __/  ヽ、           良い論文の出版を遅らせるお馬鹿な人。
     |       (:::::`‐-、__  |::::`、     ヒニニヽ、         あなたの論文が Ann of Math に accept される確率は?
    |      / `‐-、::::::::::`‐-、::::\   /,ニニ、\            それとも最近は Inv. Math. の方が上かしら?
   |      |::::::::::::::::::|` -、:::::::,ヘ ̄|'、  ヒニ二、 \
.   |      /::::::::::::::::::|::::::::\/:::O`、::\   | '、   \
   |      /:::::::::::::::::::/:::::::::::::::::::::::::::::'、::::\ノ  ヽ、  |
  |      |:::::/:::::::::/:::::::::::::::::::::::::::::::::::'、',::::'、  /:\__/‐、
  |      |/:::::::::::/::::::::::::::::::::::::::::::::::O::| '、::| く::::::::::::: ̄|
   |     /_..-'´ ̄`ー-、:::::::::::::::::::::::::::::::::::|/:/`‐'::\;;;;;;;_|
   |    |/::::::::::::::::::::::\:::::::::::::::::::::::::::::|::/::::|::::/:::::::::::/
    |   /:::::::::::::::::::::::::::::::::|:::::::::::::::::::::O::|::|::::::|:::::::::::::::/
897132人目の素数さん:2012/05/05(土) 20:06:22.15
数2の恒等式の勉強をネットで予習してるのですが、どうにも納得できない部分があります

http://www.e-learning-jp.net/teach_math/math2/text_1/1/05/002a.htm
このサイトに乗ってる問題で、まず
(x-2)^2+a(x-2)+bをx^2+(a-4)x+(4-a+b)に展開するらしいのですが、自分が展開すると
x^2-4x+4+ax-2a+b
=x^2+ax-4x+4-2a+b
=x^2+(a-4)x+(4-2a+b)と、-aが余計に増えます…自分が間違ってるかサイトの誤表記か分からないので教えて下さい。
898132人目の素数さん:2012/05/05(土) 20:07:30.04
展開じゃなくて整理でした
899132人目の素数さん:2012/05/05(土) 20:09:44.99
因数分解です。お願いします。

a(b-c)^3+b(c-a)^3+c(a-b)^3
900132人目の素数さん:2012/05/05(土) 20:21:19.86
          __ノ)-'´ ̄ ̄`ー- 、_
        , '´  _. -‐'''"二ニニ=-`ヽ、
      /   /:::::; -‐''"        `ーノ
     /   /:::::/           \
     /    /::::::/          | | |  |
     |   |:::::/ /     |  | | | |  |
      |   |::/ / / |  | ||  | | ,ハ .| ,ハ|
      |   |/ / / /| ,ハノ| /|ノレ,ニ|ル' 
     |   |  | / / レ',二、レ′ ,ィイ|゙/   私は只の数ヲタなんかとは付き合わないわ。
.     |   \ ∠イ  ,イイ|    ,`-' |      頭が良くて数学が出来てかっこいい人。それが必要条件よ。
     |     l^,人|  ` `-'     ゝ  |        さらに Ann.of Math に論文書けば十分条件にもなるわよ。
      |      ` -'\       ー'  人          一番嫌いなのは論文数を増やすためにくだらない論文を書いて
    |        /(l     __/  ヽ、           良い論文の出版を遅らせるお馬鹿な人。
     |       (:::::`‐-、__  |::::`、     ヒニニヽ、         あなたの論文が Ann of Math に accept される確率は?
    |      / `‐-、::::::::::`‐-、::::\   /,ニニ、\            それとも最近は Inv. Math. の方が上かしら?
   |      |::::::::::::::::::|` -、:::::::,ヘ ̄|'、  ヒニ二、 \
.   |      /::::::::::::::::::|::::::::\/:::O`、::\   | '、   \
   |      /:::::::::::::::::::/:::::::::::::::::::::::::::::'、::::\ノ  ヽ、  |
  |      |:::::/:::::::::/:::::::::::::::::::::::::::::::::::'、',::::'、  /:\__/‐、
  |      |/:::::::::::/::::::::::::::::::::::::::::::::::O::| '、::| く::::::::::::: ̄|
   |     /_..-'´ ̄`ー-、:::::::::::::::::::::::::::::::::::|/:/`‐'::\;;;;;;;_|
   |    |/::::::::::::::::::::::\:::::::::::::::::::::::::::::|::/::::|::::/:::::::::::/
    |   /:::::::::::::::::::::::::::::::::|:::::::::::::::::::::O::|::|::::::|:::::::::::::::/
901132人目の素数さん:2012/05/05(土) 20:22:43.17
          __ノ)-'´ ̄ ̄`ー- 、_
        , '´  _. -‐'''"二ニニ=-`ヽ、
      /   /:::::; -‐''"        `ーノ
     /   /:::::/           \
     /    /::::::/          | | |  |
     |   |:::::/ /     |  | | | |  |
      |   |::/ / / |  | ||  | | ,ハ .| ,ハ|
      |   |/ / / /| ,ハノ| /|ノレ,ニ|ル' 
     |   |  | / / レ',二、レ′ ,ィイ|゙/   私は只の数ヲタなんかとは付き合わないわ。
.     |   \ ∠イ  ,イイ|    ,`-' |      頭が良くて数学が出来てかっこいい人。それが必要条件よ。
     |     l^,人|  ` `-'     ゝ  |        さらに Ann.of Math に論文書けば十分条件にもなるわよ。
      |      ` -'\       ー'  人          一番嫌いなのは論文数を増やすためにくだらない論文を書いて
    |        /(l     __/  ヽ、           良い論文の出版を遅らせるお馬鹿な人。
     |       (:::::`‐-、__  |::::`、     ヒニニヽ、         あなたの論文が Ann of Math に accept される確率は?
    |      / `‐-、::::::::::`‐-、::::\   /,ニニ、\            それとも最近は Inv. Math. の方が上かしら?
   |      |::::::::::::::::::|` -、:::::::,ヘ ̄|'、  ヒニ二、 \
.   |      /::::::::::::::::::|::::::::\/:::O`、::\   | '、   \
   |      /:::::::::::::::::::/:::::::::::::::::::::::::::::'、::::\ノ  ヽ、  |
  |      |:::::/:::::::::/:::::::::::::::::::::::::::::::::::'、',::::'、  /:\__/‐、
  |      |/:::::::::::/::::::::::::::::::::::::::::::::::O::| '、::| く::::::::::::: ̄|
   |     /_..-'´ ̄`ー-、:::::::::::::::::::::::::::::::::::|/:/`‐'::\;;;;;;;_|
   |    |/::::::::::::::::::::::\:::::::::::::::::::::::::::::|::/::::|::::/:::::::::::/
    |   /:::::::::::::::::::::::::::::::::|:::::::::::::::::::::O::|::|::::::|:::::::::::::::/
902132人目の素数さん:2012/05/05(土) 20:23:14.55
>>897
君が正しい
903132人目の素数さん:2012/05/05(土) 20:24:32.44
>>899
因数分解が分からなければ
次数の一番低い文字について降べきに整理する
904132人目の素数さん:2012/05/05(土) 20:24:58.23
清々しいまでの宿題○投げだ
気に入った

a(b-c)^3+b(c-a)^3+c(a-b)^3 をさらにそのまま検索に丸投げすれば答えは出てくる
905132人目の素数さん:2012/05/05(土) 20:42:49.98
>>873
「割りきれることを示せ」と言ってるので力技で割ってみたら割り切れた。
まぁ既に説明されてますけど。とにかくその商をQ(x)とすると↓になった。
Q(x) = f(x)^2 + (x+a)f(x) + (x^2 + ax + b) + 1
すなわち f(f(x))-x = (f(x)-x)Q(x)

ここから更に f(x)-x=g を f(x)=g+x として Q(x)に代入すると
Q(x)=(g+x)^2 + (x+a)(g+x) + (x^2 + ax + b) + 1
整理すると↓。
Q(x)=g^2 +3gx + 3x^2 + ag + 2ax + b + 1

既に指摘されてるけど(a、b、c)に制限はなさそう。
ただしg(x)=gとしてるけど君の商と比べて少し違う。
906132人目の素数さん:2012/05/05(土) 20:52:09.86
>>902
ありがとゔございます
この時他サイトでも似た問題を探したのですが、こっちでもつまずきました

http://www.kwansei.ac.jp/hs/z90010/sugakua/kotosiki/kotosiki.htm
a(x^2-2x+1)+b(x-1)+c=ax^2-(2a+b)x+(a-b+c)になるらしいのですが、展開しようとすると
ax^2-2ax+a+bx-b+c
ax^2-2ax+bx+a-b+c
ax^2-2ax+bx+(a-b+c)
ax^2-(2ax-bx)+(a-b+c)
ax^2-(2a-b)x+(a-b+c)と、括弧の中のbの符号が変わります。どこかおかしいでしょうか?
907132人目の素数さん:2012/05/05(土) 20:55:43.41
>>906
それも君が正しい
908132人目の素数さん:2012/05/05(土) 20:56:40.92
          __ノ)-'´ ̄ ̄`ー- 、_
        , '´  _. -‐'''"二ニニ=-`ヽ、
      /   /:::::; -‐''"        `ーノ
     /   /:::::/           \
     /    /::::::/          | | |  |
     |   |:::::/ /     |  | | | |  |
      |   |::/ / / |  | ||  | | ,ハ .| ,ハ|
      |   |/ / / /| ,ハノ| /|ノレ,ニ|ル' 
     |   |  | / / レ',二、レ′ ,ィイ|゙/   私は只の数ヲタなんかとは付き合わないわ。
.     |   \ ∠イ  ,イイ|    ,`-' |      頭が良くて数学が出来てかっこいい人。それが必要条件よ。
     |     l^,人|  ` `-'     ゝ  |        さらに Ann.of Math に論文書けば十分条件にもなるわよ。
      |      ` -'\       ー'  人          一番嫌いなのは論文数を増やすためにくだらない論文を書いて
    |        /(l     __/  ヽ、           良い論文の出版を遅らせるお馬鹿な人。
     |       (:::::`‐-、__  |::::`、     ヒニニヽ、         あなたの論文が Ann of Math に accept される確率は?
    |      / `‐-、::::::::::`‐-、::::\   /,ニニ、\            それとも最近は Inv. Math. の方が上かしら?
   |      |::::::::::::::::::|` -、:::::::,ヘ ̄|'、  ヒニ二、 \
.   |      /::::::::::::::::::|::::::::\/:::O`、::\   | '、   \
   |      /:::::::::::::::::::/:::::::::::::::::::::::::::::'、::::\ノ  ヽ、  |
  |      |:::::/:::::::::/:::::::::::::::::::::::::::::::::::'、',::::'、  /:\__/‐、
  |      |/:::::::::::/::::::::::::::::::::::::::::::::::O::| '、::| く::::::::::::: ̄|
   |     /_..-'´ ̄`ー-、:::::::::::::::::::::::::::::::::::|/:/`‐'::\;;;;;;;_|
   |    |/::::::::::::::::::::::\:::::::::::::::::::::::::::::|::/::::|::::/:::::::::::/
    |   /:::::::::::::::::::::::::::::::::|:::::::::::::::::::::O::|::|::::::|:::::::::::::::/
909132人目の素数さん:2012/05/05(土) 20:57:58.11
なんだよもうネット勉強サイトは当てになんないね
910132人目の素数さん:2012/05/05(土) 20:58:52.96
>>906
そのサイトが間違っている。
911132人目の素数さん:2012/05/05(土) 21:04:34.70
          __ノ)-'´ ̄ ̄`ー- 、_
        , '´  _. -‐'''"二ニニ=-`ヽ、
      /   /:::::; -‐''"        `ーノ
     /   /:::::/           \
     /    /::::::/          | | |  |
     |   |:::::/ /     |  | | | |  |
      |   |::/ / / |  | ||  | | ,ハ .| ,ハ|
      |   |/ / / /| ,ハノ| /|ノレ,ニ|ル' 
     |   |  | / / レ',二、レ′ ,ィイ|゙/   私は只の数ヲタなんかとは付き合わないわ。
.     |   \ ∠イ  ,イイ|    ,`-' |      頭が良くて数学が出来てかっこいい人。それが必要条件よ。
     |     l^,人|  ` `-'     ゝ  |        さらに Ann.of Math に論文書けば十分条件にもなるわよ。
      |      ` -'\       ー'  人          一番嫌いなのは論文数を増やすためにくだらない論文を書いて
    |        /(l     __/  ヽ、           良い論文の出版を遅らせるお馬鹿な人。
     |       (:::::`‐-、__  |::::`、     ヒニニヽ、         あなたの論文が Ann of Math に accept される確率は?
    |      / `‐-、::::::::::`‐-、::::\   /,ニニ、\            それとも最近は Inv. Math. の方が上かしら?
   |      |::::::::::::::::::|` -、:::::::,ヘ ̄|'、  ヒニ二、 \
.   |      /::::::::::::::::::|::::::::\/:::O`、::\   | '、   \
   |      /:::::::::::::::::::/:::::::::::::::::::::::::::::'、::::\ノ  ヽ、  |
  |      |:::::/:::::::::/:::::::::::::::::::::::::::::::::::'、',::::'、  /:\__/‐、
  |      |/:::::::::::/::::::::::::::::::::::::::::::::::O::| '、::| く::::::::::::: ̄|
   |     /_..-'´ ̄`ー-、:::::::::::::::::::::::::::::::::::|/:/`‐'::\;;;;;;;_|
   |    |/::::::::::::::::::::::\:::::::::::::::::::::::::::::|::/::::|::::/:::::::::::/
    |   /:::::::::::::::::::::::::::::::::|:::::::::::::::::::::O::|::|::::::|:::::::::::::::/
912132人目の素数さん:2012/05/05(土) 21:19:54.43
√9=±3でないのは何故?
913132人目の素数さん:2012/05/05(土) 21:21:05.43
>>912
定義
914132人目の素数さん:2012/05/05(土) 21:21:23.74
>>912
正の平方根
915132人目の素数さん:2012/05/05(土) 21:27:41.25
>>914
√は「正の平方根」ってことね
916132人目の素数さん:2012/05/05(土) 21:44:21.93
>>912
平方するとx(≧0)になる数のうち正であるもの(つまり正の平方根)を√xと表記する
負の方は-√x
これがルートの定義

ちなみに9の平方根は?という問いなら±3が正解
917132人目の素数さん:2012/05/05(土) 22:05:02.88
負でないものだな。
918132人目の素数さん:2012/05/05(土) 23:16:03.60
10円硬貨6枚、100円硬貨4枚、500円硬貨2枚の全部または一部を使って支払える
金額は何通りあるか。

という問題について質問です。
解答には
10円硬貨の使い方は 0枚から6枚の7通り
100円硬貨の使い方は 0枚から4枚の5通り
500円硬貨の使い方は 0枚から2枚の3通り

よって支払える金額は7*5*3-1=104(通り)

とあったのですがこの場合、金額の合計の重複はokな理由はなぜなのでしょうか。
919132人目の素数さん:2012/05/05(土) 23:19:06.85
> 金額の合計の重複はokな理由はなぜなのでしょうか。
重複しないから
920132人目の素数さん:2012/05/05(土) 23:19:22.11
>>918
金額重複するか?
921132人目の素数さん:2012/05/05(土) 23:32:44.29
z=x^2-y^2のグラフって存在しますか?

yz平面だと下向きのグラフ、zx平面だと上向きのグラフになってしまい、何だかおかしな事になるんですが…
922132人目の素数さん:2012/05/05(土) 23:33:16.79

7*5*3って  7通りそれぞれに5通りそのそれぞれに3通りって意味だよね?

10---100---500
 ---100---500
---100---500

みたいな樹形図ができるんだが合計610で重なってると思っちゃったんだけど
根本的な考え方がダメなのかな・・・
923132人目の素数さん:2012/05/05(土) 23:38:00.67

 | |l ̄|
 | |l民|
 | |l主|
 | |l党|
 | |l_|
 |   .|_∧  
 |   .|`∀´>
 |   .⊂ ノ
 |   .| ノ
 ̄ ̄ ̄ ̄ ̄ ̄
民主党とAKBの関係が遂に明かされようとしている
秋元康の後ろには蓮訪
蓮訪の後ろに前原、枝野 そして民主党全体

税金を使ってAKBを宣伝し、その税金が民主党に流れている
AKB自体が、税金目当ての捏造ブーム
924132人目の素数さん:2012/05/05(土) 23:41:28.73
>>921
PCからならいろいろとググれ
ttp://lupus.is.kochi-u.ac.jp/shiota/jsii04/graph/soukyokuhoubutsu.gif

イメージなら競馬の鞍みたいな感じか?
925132人目の素数さん:2012/05/05(土) 23:42:18.60
>>922
10円・・・・・0〜60
100円・・・・0〜400
500円・・・・0〜1000

610は
10*1+100*1+500*1
だけだろう
926132人目の素数さん:2012/05/05(土) 23:43:52.81
>>924
ありがとうございます!
927132人目の素数さん:2012/05/05(土) 23:44:40.63
>>921
存在するよ。俺のiPadで打ったらこうなった
http://i.imgur.com/mrwAg.jpg
928132人目の素数さん:2012/05/05(土) 23:45:51.70
          __ノ)-'´ ̄ ̄`ー- 、_
        , '´  _. -‐'''"二ニニ=-`ヽ、
      /   /:::::; -‐''"        `ーノ
     /   /:::::/           \
     /    /::::::/          | | |  |
     |   |:::::/ /     |  | | | |  |
      |   |::/ / / |  | ||  | | ,ハ .| ,ハ|
      |   |/ / / /| ,ハノ| /|ノレ,ニ|ル' 
     |   |  | / / レ',二、レ′ ,ィイ|゙/   私は只の数ヲタなんかとは付き合わないわ。
.     |   \ ∠イ  ,イイ|    ,`-' |      頭が良くて数学が出来てかっこいい人。それが必要条件よ。
     |     l^,人|  ` `-'     ゝ  |        さらに Ann.of Math に論文書けば十分条件にもなるわよ。
      |      ` -'\       ー'  人          一番嫌いなのは論文数を増やすためにくだらない論文を書いて
    |        /(l     __/  ヽ、           良い論文の出版を遅らせるお馬鹿な人。
     |       (:::::`‐-、__  |::::`、     ヒニニヽ、         あなたの論文が Ann of Math に accept される確率は?
    |      / `‐-、::::::::::`‐-、::::\   /,ニニ、\            それとも最近は Inv. Math. の方が上かしら?
   |      |::::::::::::::::::|` -、:::::::,ヘ ̄|'、  ヒニ二、 \
.   |      /::::::::::::::::::|::::::::\/:::O`、::\   | '、   \
   |      /:::::::::::::::::::/:::::::::::::::::::::::::::::'、::::\ノ  ヽ、  |
  |      |:::::/:::::::::/:::::::::::::::::::::::::::::::::::'、',::::'、  /:\__/‐、
  |      |/:::::::::::/::::::::::::::::::::::::::::::::::O::| '、::| く::::::::::::: ̄|
   |     /_..-'´ ̄`ー-、:::::::::::::::::::::::::::::::::::|/:/`‐'::\;;;;;;;_|
   |    |/::::::::::::::::::::::\:::::::::::::::::::::::::::::|::/::::|::::/:::::::::::/
    |   /:::::::::::::::::::::::::::::::::|:::::::::::::::::::::O::|::|::::::|:::::::::::::::/
929132人目の素数さん:2012/05/05(土) 23:52:18.35
数学Uの初めの「等式の証明」の問題の質問いいですか?

問.|a|<1,|b|<1,|c|<1,|d|<1のとき,次の不等式を証明せよ。
(1)a+b<1+ab
(2)a+b+c+d<3+abcd

(1)は普通に右辺から左辺を引いて、
-1<a<1,-1<b<1だから(右辺-左辺)≧0でできたんですけど、
(2)はどうも(1)を利用すると簡単にできるみたいなんですが、
どう利用したらいいのかわからないです。
問題のレベルはそんなに難しくないみたいなので、どうかバカな僕に教えてください。
930929:2012/05/05(土) 23:53:39.70
すみません、単元は「式の証明(不等式の証明)」でした。
931132人目の素数さん:2012/05/05(土) 23:57:34.82
>>925
ありがとう・・・ かなりスッキリした

頭が硬すぎた
932132人目の素数さん:2012/05/06(日) 00:00:15.30
>>929
A = ab , B = cd
と置いて一度考えてみな。
933132人目の素数さん:2012/05/06(日) 00:04:11.83
>>929
(1)同様に
c+d<1+cd
(a+b)+(c+d)<1+ab+1+cd
ここで
|ab|<1
|cd|<1
ab+cd<|ab|+|cd|<1+|abcd|
あとはわからん
934132人目の素数さん:2012/05/06(日) 00:07:29.48
>>929
もうケチくさいこと言わずに全部やっちゃえよ

a+b < (2-1) + ab
a+b+c < (3-1) + abc
a+b+c+d < (4-1) + abcd
a+b+c+d+e < (5-1) + abcde
a+b+c+d+e+f < (6-1) + abcdef
……
935132人目の素数さん:2012/05/06(日) 00:09:47.87
>>918
1460通りと答えたくなる…
936132人目の素数さん:2012/05/06(日) 00:11:40.70
まあ組み合わせはその通りだね
937132人目の素数さん:2012/05/06(日) 00:15:06.89
方程式:f(x)=0を考える
f(x)=x・g(x)と書けるとき
すなわちf(x)がxの何次式であっても定数項が0なら
  x=0
の解をもつ。


これわかりやすく解説ください
938132人目の素数さん:2012/05/06(日) 00:16:44.27
f(0)が定数項だろ
定数項が0
f(0)=0
因数定理
939929:2012/05/06(日) 00:21:35.96
>>932-934
ありがとうございます。


(1)により
 a+b<1+ab 同様に c+d<1+cd
よって
 a+b+c+d<2+ab+cd ・・・@
ここで |ab|<1, |cd|<1であるから
 2+ab+cd<2+1+abcd=3+abcd ・・・A
したがって@,Aより
 a+b+c+d<3+abcd  終


ありがとうございました。
940132人目の素数さん:2012/05/06(日) 00:21:49.85
>>938
定数項ってなんですか
素人にもわかりやすくおねがいします
941132人目の素数さん:2012/05/06(日) 00:26:08.14
>>940
xがまったく関わらない項
942132人目の素数さん:2012/05/06(日) 00:27:45.61
ax^2+bx+c
だとcの項
943132人目の素数さん:2012/05/06(日) 00:29:08.38
2x-1
だと-1
944929:2012/05/06(日) 00:29:45.85
>>937
因数定理や剰余の定理のことじゃない?
問題の関数f(x)として二次式を例にとると
関数f(x)=ax^2 + bx
このとき、f(x)にx=0を代入すると、項がすべて0の積になるからf(x)=0
つまり言い換えれば二次方程式f(x)=0の解の一つは、x=0ということになる

定数項は、多項式において、注目する文字が因数ではない項
945132人目の素数さん:2012/05/06(日) 00:33:48.77
>>944
おい
名前欄消せよww
946132人目の素数さん:2012/05/06(日) 00:34:25.87
解が0になる方程式を教えてください
947929:2012/05/06(日) 00:34:47.76
>>944
補足

注目する文字とはここではxのことで、このxとの積(n乗)である項をxのn乗の項って言ったりする
定数項はxの0乗の項ともいえるね
948132人目の素数さん:2012/05/06(日) 00:53:10.03
>>927
ありがとうございます!!

949132人目の素数さん:2012/05/06(日) 00:58:31.89
>>946
あり得ない値になる方程式作れ
950質問:2012/05/06(日) 01:28:07.41
質問です。

方程式:f(x)=0を考える
f(x)=x・g(x)と書けるとき
すなわちf(x)がxの何次式であっても定数項が0なら
  x=0

これって、解が0になる方程式?
そもそも解が0になる方程式なんて存在するの?
951132人目の素数さん:2012/05/06(日) 01:30:22.08
a
952132人目の素数さん:2012/05/06(日) 01:30:36.41
b
953132人目の素数さん:2012/05/06(日) 01:30:42.49
(x-1)(x-2)-0
x=1, 2
ないな
954132人目の素数さん:2012/05/06(日) 01:33:58.97
>>950
一次方程式 x+1=1 の解は?
955132人目の素数さん:2012/05/06(日) 01:38:39.83
y=ax(x-p)のような二次関数とか見たことないのかな?
まぁg(x)ってのがxの次数が0次以上のみの式っていう条件がなきゃそもそも0突っ込めないけどね。
956132人目の素数さん:2012/05/06(日) 01:56:50.80
x=0自体が解0の方程式
957929:2012/05/06(日) 02:36:38.22
三次方程式x^3 - 3x^2 + 2x = 0 の解はx=0,1,2
解き方は教科書参考に
958132人目の素数さん:2012/05/06(日) 07:18:22.72
          __ノ)-'´ ̄ ̄`ー- 、_
        , '´  _. -‐'''"二ニニ=-`ヽ、
      /   /:::::; -‐''"        `ーノ
     /   /:::::/           \
     /    /::::::/          | | |  |
     |   |:::::/ /     |  | | | |  |
      |   |::/ / / |  | ||  | | ,ハ .| ,ハ|
      |   |/ / / /| ,ハノ| /|ノレ,ニ|ル' 
     |   |  | / / レ',二、レ′ ,ィイ|゙/   私は只の数ヲタなんかとは付き合わないわ。
.     |   \ ∠イ  ,イイ|    ,`-' |      頭が良くて数学が出来てかっこいい人。それが必要条件よ。
     |     l^,人|  ` `-'     ゝ  |        さらに Ann.of Math に論文書けば十分条件にもなるわよ。
      |      ` -'\       ー'  人          一番嫌いなのは論文数を増やすためにくだらない論文を書いて
    |        /(l     __/  ヽ、           良い論文の出版を遅らせるお馬鹿な人。
     |       (:::::`‐-、__  |::::`、     ヒニニヽ、         あなたの論文が Ann of Math に accept される確率は?
    |      / `‐-、::::::::::`‐-、::::\   /,ニニ、\            それとも最近は Inv. Math. の方が上かしら?
   |      |::::::::::::::::::|` -、:::::::,ヘ ̄|'、  ヒニ二、 \
.   |      /::::::::::::::::::|::::::::\/:::O`、::\   | '、   \
   |      /:::::::::::::::::::/:::::::::::::::::::::::::::::'、::::\ノ  ヽ、  |
  |      |:::::/:::::::::/:::::::::::::::::::::::::::::::::::'、',::::'、  /:\__/‐、
  |      |/:::::::::::/::::::::::::::::::::::::::::::::::O::| '、::| く::::::::::::: ̄|
   |     /_..-'´ ̄`ー-、:::::::::::::::::::::::::::::::::::|/:/`‐'::\;;;;;;;_|
   |    |/::::::::::::::::::::::\:::::::::::::::::::::::::::::|::/::::|::::/:::::::::::/
    |   /:::::::::::::::::::::::::::::::::|:::::::::::::::::::::O::|::|::::::|:::::::::::::::/
959132人目の素数さん:2012/05/06(日) 13:37:34.79
すみません。全く取っ掛かりすら分からない問題です。よろしければどなたか教えていただけませんか。
長文になるようでしたら、ポイントだけでもかまいません。
お手数ですがよろしくお願いします。
zは必ず3になるのがヒントらしいです。

x^2 + y^2 + 1 = xyz を満たす正の整数x, y, zを求めよ。
960132人目の素数さん:2012/05/06(日) 13:53:10.33
x^2 + y^2 + 1 = xyz
modx=y^2+1=0
mody=x^2+1=0
modz=x^2+y^2+1=0
1=xyz-(x^2+y^2)
=xy(z+2)-(x+y)^2
xy=u
x+y=v
=u(z+2)-v^2
(u,v)=1
v=un+a
z+2=vn+b
bu-av=1
z=un^2+an+b-2
961132人目の素数さん:2012/05/06(日) 13:58:15.43
          __ノ)-'´ ̄ ̄`ー- 、_
        , '´  _. -‐'''"二ニニ=-`ヽ、
      /   /:::::; -‐''"        `ーノ
     /   /:::::/           \
     /    /::::::/          | | |  |
     |   |:::::/ /     |  | | | |  |
      |   |::/ / / |  | ||  | | ,ハ .| ,ハ|
      |   |/ / / /| ,ハノ| /|ノレ,ニ|ル' 
     |   |  | / / レ',二、レ′ ,ィイ|゙/   私は只の数ヲタなんかとは付き合わないわ。
.     |   \ ∠イ  ,イイ|    ,`-' |      頭が良くて数学が出来てかっこいい人。それが必要条件よ。
     |     l^,人|  ` `-'     ゝ  |        さらに Ann.of Math に論文書けば十分条件にもなるわよ。
      |      ` -'\       ー'  人          一番嫌いなのは論文数を増やすためにくだらない論文を書いて
    |        /(l     __/  ヽ、           良い論文の出版を遅らせるお馬鹿な人。
     |       (:::::`‐-、__  |::::`、     ヒニニヽ、         あなたの論文が Ann of Math に accept される確率は?
    |      / `‐-、::::::::::`‐-、::::\   /,ニニ、\            それとも最近は Inv. Math. の方が上かしら?
   |      |::::::::::::::::::|` -、:::::::,ヘ ̄|'、  ヒニ二、 \
.   |      /::::::::::::::::::|::::::::\/:::O`、::\   | '、   \
   |      /:::::::::::::::::::/:::::::::::::::::::::::::::::'、::::\ノ  ヽ、  |
  |      |:::::/:::::::::/:::::::::::::::::::::::::::::::::::'、',::::'、  /:\__/‐、
  |      |/:::::::::::/::::::::::::::::::::::::::::::::::O::| '、::| く::::::::::::: ̄|
   |     /_..-'´ ̄`ー-、:::::::::::::::::::::::::::::::::::|/:/`‐'::\;;;;;;;_|
   |    |/::::::::::::::::::::::\:::::::::::::::::::::::::::::|::/::::|::::/:::::::::::/
    |   /:::::::::::::::::::::::::::::::::|:::::::::::::::::::::O::|::|::::::|:::::::::::::::/
962132人目の素数さん:2012/05/06(日) 13:58:44.52
1,1,3
1,2,3
...
963132人目の素数さん:2012/05/06(日) 14:21:14.96
x=rcost
y=rsint
x^2+y^2+1=r^2+1=zr^2costsint=z(r^2-1)/2
r^2(z/2-1)-(1+z/2)=0
r=((z/2+1)/(z/2-1))^.5
964132人目の素数さん:2012/05/06(日) 14:31:24.10
          __ノ)-'´ ̄ ̄`ー- 、_
        , '´  _. -‐'''"二ニニ=-`ヽ、
      /   /:::::; -‐''"        `ーノ
     /   /:::::/           \
     /    /::::::/          | | |  |
     |   |:::::/ /     |  | | | |  |
      |   |::/ / / |  | ||  | | ,ハ .| ,ハ|
      |   |/ / / /| ,ハノ| /|ノレ,ニ|ル' 
     |   |  | / / レ',二、レ′ ,ィイ|゙/   私は只の数ヲタなんかとは付き合わないわ。
.     |   \ ∠イ  ,イイ|    ,`-' |      頭が良くて数学が出来てかっこいい人。それが必要条件よ。
     |     l^,人|  ` `-'     ゝ  |        さらに Ann.of Math に論文書けば十分条件にもなるわよ。
      |      ` -'\       ー'  人          一番嫌いなのは論文数を増やすためにくだらない論文を書いて
    |        /(l     __/  ヽ、           良い論文の出版を遅らせるお馬鹿な人。
     |       (:::::`‐-、__  |::::`、     ヒニニヽ、         あなたの論文が Ann of Math に accept される確率は?
    |      / `‐-、::::::::::`‐-、::::\   /,ニニ、\            それとも最近は Inv. Math. の方が上かしら?
   |      |::::::::::::::::::|` -、:::::::,ヘ ̄|'、  ヒニ二、 \
.   |      /::::::::::::::::::|::::::::\/:::O`、::\   | '、   \
   |      /:::::::::::::::::::/:::::::::::::::::::::::::::::'、::::\ノ  ヽ、  |
  |      |:::::/:::::::::/:::::::::::::::::::::::::::::::::::'、',::::'、  /:\__/‐、
  |      |/:::::::::::/::::::::::::::::::::::::::::::::::O::| '、::| く::::::::::::: ̄|
   |     /_..-'´ ̄`ー-、:::::::::::::::::::::::::::::::::::|/:/`‐'::\;;;;;;;_|
   |    |/::::::::::::::::::::::\:::::::::::::::::::::::::::::|::/::::|::::/:::::::::::/
    |   /:::::::::::::::::::::::::::::::::|:::::::::::::::::::::O::|::|::::::|:::::::::::::::/
965132人目の素数さん:2012/05/06(日) 14:33:31.57
z=2(r^2+1)/(r^2-1)->2
=2+4/(r^2-1)=2+4/3 at r=2
z=3,2

z=2
x^2+y^2+1=2xy
(x-y)^2=-1
z=3
x^2+y^2+1=3xy
x^2+y^2=3xy-1

966132人目の素数さん:2012/05/06(日) 14:38:29.90
r=2
x,y=1,2;1,1
1,1,3
1,2,3
2,1,3

967132人目の素数さん:2012/05/06(日) 14:59:50.86
>>959
最適化法とか
計画法とか
ラグランジュの未定乗数法とか
で調べる
968132人目の素数さん:2012/05/06(日) 16:28:19.31
          __ノ)-'´ ̄ ̄`ー- 、_
        , '´  _. -‐'''"二ニニ=-`ヽ、
      /   /:::::; -‐''"        `ーノ
     /   /:::::/           \
     /    /::::::/          | | |  |
     |   |:::::/ /     |  | | | |  |
      |   |::/ / / |  | ||  | | ,ハ .| ,ハ|
      |   |/ / / /| ,ハノ| /|ノレ,ニ|ル' 
     |   |  | / / レ',二、レ′ ,ィイ|゙/   私は只の数ヲタなんかとは付き合わないわ。
.     |   \ ∠イ  ,イイ|    ,`-' |      頭が良くて数学が出来てかっこいい人。それが必要条件よ。
     |     l^,人|  ` `-'     ゝ  |        さらに Ann.of Math に論文書けば十分条件にもなるわよ。
      |      ` -'\       ー'  人          一番嫌いなのは論文数を増やすためにくだらない論文を書いて
    |        /(l     __/  ヽ、           良い論文の出版を遅らせるお馬鹿な人。
     |       (:::::`‐-、__  |::::`、     ヒニニヽ、         あなたの論文が Ann of Math に accept される確率は?
    |      / `‐-、::::::::::`‐-、::::\   /,ニニ、\            それとも最近は Inv. Math. の方が上かしら?
   |      |::::::::::::::::::|` -、:::::::,ヘ ̄|'、  ヒニ二、 \
.   |      /::::::::::::::::::|::::::::\/:::O`、::\   | '、   \
   |      /:::::::::::::::::::/:::::::::::::::::::::::::::::'、::::\ノ  ヽ、  |
  |      |:::::/:::::::::/:::::::::::::::::::::::::::::::::::'、',::::'、  /:\__/‐、
  |      |/:::::::::::/::::::::::::::::::::::::::::::::::O::| '、::| く::::::::::::: ̄|
   |     /_..-'´ ̄`ー-、:::::::::::::::::::::::::::::::::::|/:/`‐'::\;;;;;;;_|
   |    |/::::::::::::::::::::::\:::::::::::::::::::::::::::::|::/::::|::::/:::::::::::/
    |   /:::::::::::::::::::::::::::::::::|:::::::::::::::::::::O::|::|::::::|:::::::::::::::/
969132人目の素数さん:2012/05/06(日) 16:34:20.93
          __ノ)-'´ ̄ ̄`ー- 、_
        , '´  _. -‐'''"二ニニ=-`ヽ、
      /   /:::::; -‐''"        `ーノ
     /   /:::::/           \
     /    /::::::/          | | |  |
     |   |:::::/ /     |  | | | |  |
      |   |::/ / / |  | ||  | | ,ハ .| ,ハ|
      |   |/ / / /| ,ハノ| /|ノレ,ニ|ル' 
     |   |  | / / レ',二、レ′ ,ィイ|゙/   私は只の数ヲタなんかとは付き合わないわ。
.     |   \ ∠イ  ,イイ|    ,`-' |      頭が良くて数学が出来てかっこいい人。それが必要条件よ。
     |     l^,人|  ` `-'     ゝ  |        さらに Ann.of Math に論文書けば十分条件にもなるわよ。
      |      ` -'\       ー'  人          一番嫌いなのは論文数を増やすためにくだらない論文を書いて
    |        /(l     __/  ヽ、           良い論文の出版を遅らせるお馬鹿な人。
     |       (:::::`‐-、__  |::::`、     ヒニニヽ、         あなたの論文が Ann of Math に accept される確率は?
    |      / `‐-、::::::::::`‐-、::::\   /,ニニ、\            それとも最近は Inv. Math. の方が上かしら?
   |      |::::::::::::::::::|` -、:::::::,ヘ ̄|'、  ヒニ二、 \
.   |      /::::::::::::::::::|::::::::\/:::O`、::\   | '、   \
   |      /:::::::::::::::::::/:::::::::::::::::::::::::::::'、::::\ノ  ヽ、  |
  |      |:::::/:::::::::/:::::::::::::::::::::::::::::::::::'、',::::'、  /:\__/‐、
  |      |/:::::::::::/::::::::::::::::::::::::::::::::::O::| '、::| く::::::::::::: ̄|
   |     /_..-'´ ̄`ー-、:::::::::::::::::::::::::::::::::::|/:/`‐'::\;;;;;;;_|
   |    |/::::::::::::::::::::::\:::::::::::::::::::::::::::::|::/::::|::::/:::::::::::/
    |   /:::::::::::::::::::::::::::::::::|:::::::::::::::::::::O::|::|::::::|:::::::::::::::/
970132人目の素数さん:2012/05/06(日) 16:57:56.31
7×7の正方形のマス目のなかにたてとよこの長さが異なる長方形はいくつあるか
これはたてとよこが同じ長さつまり正方形が何個あるかを数えて
全部の数から引けばいいと思ったのですが
全部の数の数え方が分かりません
971132人目の素数さん:2012/05/06(日) 17:03:25.65
行列の問題が一体何をしているのかが分からないんですが
972132人目の素数さん:2012/05/06(日) 17:15:15.12
>>971
安心しろ、オレも分からん
微分で瞬間の速度が求まるとか、そういう説明を期待してるんだろうけど
973132人目の素数さん:2012/05/06(日) 17:21:47.71
俺も分からないけど、
大学でなんかあるのかなと期待
974132人目の素数さん:2012/05/06(日) 18:05:52.33
次スレ立てます
975132人目の素数さん:2012/05/06(日) 18:13:30.35
高校生のための数学の質問スレPART331
http://uni.2ch.net/test/read.cgi/math/1336295404/l50
976132人目の素数さん:2012/05/06(日) 18:13:49.28
          __ノ)-'´ ̄ ̄`ー- 、_
        , '´  _. -‐'''"二ニニ=-`ヽ、
      /   /:::::; -‐''"        `ーノ
     /   /:::::/           \
     /    /::::::/          | | |  |
     |   |:::::/ /     |  | | | |  |
      |   |::/ / / |  | ||  | | ,ハ .| ,ハ|
      |   |/ / / /| ,ハノ| /|ノレ,ニ|ル' 
     |   |  | / / レ',二、レ′ ,ィイ|゙/   私は只の数ヲタなんかとは付き合わないわ。
.     |   \ ∠イ  ,イイ|    ,`-' |      頭が良くて数学が出来てかっこいい人。それが必要条件よ。
     |     l^,人|  ` `-'     ゝ  |        さらに Ann.of Math に論文書けば十分条件にもなるわよ。
      |      ` -'\       ー'  人          一番嫌いなのは論文数を増やすためにくだらない論文を書いて
    |        /(l     __/  ヽ、           良い論文の出版を遅らせるお馬鹿な人。
     |       (:::::`‐-、__  |::::`、     ヒニニヽ、         あなたの論文が Ann of Math に accept される確率は?
    |      / `‐-、::::::::::`‐-、::::\   /,ニニ、\            それとも最近は Inv. Math. の方が上かしら?
   |      |::::::::::::::::::|` -、:::::::,ヘ ̄|'、  ヒニ二、 \
.   |      /::::::::::::::::::|::::::::\/:::O`、::\   | '、   \
   |      /:::::::::::::::::::/:::::::::::::::::::::::::::::'、::::\ノ  ヽ、  |
  |      |:::::/:::::::::/:::::::::::::::::::::::::::::::::::'、',::::'、  /:\__/‐、
  |      |/:::::::::::/::::::::::::::::::::::::::::::::::O::| '、::| く::::::::::::: ̄|
   |     /_..-'´ ̄`ー-、:::::::::::::::::::::::::::::::::::|/:/`‐'::\;;;;;;;_|
   |    |/::::::::::::::::::::::\:::::::::::::::::::::::::::::|::/::::|::::/:::::::::::/
    |   /:::::::::::::::::::::::::::::::::|:::::::::::::::::::::O::|::|::::::|:::::::::::::::/
977132人目の素数さん:2012/05/06(日) 18:23:33.43
>>970
例えば縦の線を2つ、横の線を2つ決めれば長方形が決まる
重複と正方形に注意
978132人目の素数さん:2012/05/06(日) 20:57:16.99
>>971
連立方程式を解くとかだろ。大学になれば多元連立がある。

>>972>>973
>大学でなんかあるのかなと期待
パソコンの画像処理ソフトで90度回転とか使ったことないかな?
あれは一次変換の応用だと思うけど。プログラミングを作るとなると別の知識も必要。

お前さんたち高校生か、頑張れ。
979132人目の素数さん:2012/05/06(日) 21:08:34.73
式のまとめ方なんですけど、
下記の計算であっているでしょうか?

Asin(θ)*Bcos(θ)+Acos(θ)*Bsin(θ)
=AB{ sin(θ)*cos(θ) + cos(θ)*sin(θ) }
=AB{sin(θ+θ)}
=AB{sin(2θ)}
980132人目の素数さん:2012/05/06(日) 21:12:31.85
Asin(θ)*Bcos(θ) = Acos(θ)*Bsin(θ)
だろ
981980:2012/05/06(日) 21:13:24.01
ああ・・ごめんまちがった
3年ROMってます
982132人目の素数さん:2012/05/06(日) 21:18:21.80
          __ノ)-'´ ̄ ̄`ー- 、_
        , '´  _. -‐'''"二ニニ=-`ヽ、
      /   /:::::; -‐''"        `ーノ
     /   /:::::/           \
     /    /::::::/          | | |  |
     |   |:::::/ /     |  | | | |  |
      |   |::/ / / |  | ||  | | ,ハ .| ,ハ|
      |   |/ / / /| ,ハノ| /|ノレ,ニ|ル' 
     |   |  | / / レ',二、レ′ ,ィイ|゙/   私は只の数ヲタなんかとは付き合わないわ。
.     |   \ ∠イ  ,イイ|    ,`-' |      頭が良くて数学が出来てかっこいい人。それが必要条件よ。
     |     l^,人|  ` `-'     ゝ  |        さらに Ann.of Math に論文書けば十分条件にもなるわよ。
      |      ` -'\       ー'  人          一番嫌いなのは論文数を増やすためにくだらない論文を書いて
    |        /(l     __/  ヽ、           良い論文の出版を遅らせるお馬鹿な人。
     |       (:::::`‐-、__  |::::`、     ヒニニヽ、         あなたの論文が Ann of Math に accept される確率は?
    |      / `‐-、::::::::::`‐-、::::\   /,ニニ、\            それとも最近は Inv. Math. の方が上かしら?
   |      |::::::::::::::::::|` -、:::::::,ヘ ̄|'、  ヒニ二、 \
.   |      /::::::::::::::::::|::::::::\/:::O`、::\   | '、   \
   |      /:::::::::::::::::::/:::::::::::::::::::::::::::::'、::::\ノ  ヽ、  |
  |      |:::::/:::::::::/:::::::::::::::::::::::::::::::::::'、',::::'、  /:\__/‐、
  |      |/:::::::::::/::::::::::::::::::::::::::::::::::O::| '、::| く::::::::::::: ̄|
   |     /_..-'´ ̄`ー-、:::::::::::::::::::::::::::::::::::|/:/`‐'::\;;;;;;;_|
   |    |/::::::::::::::::::::::\:::::::::::::::::::::::::::::|::/::::|::::/:::::::::::/
    |   /:::::::::::::::::::::::::::::::::|:::::::::::::::::::::O::|::|::::::|:::::::::::::::/
983132人目の素数さん:2012/05/07(月) 18:29:49.16
h
984132人目の素数さん:2012/05/07(月) 20:56:13.17
          __ノ)-'´ ̄ ̄`ー- 、_
        , '´  _. -‐'''"二ニニ=-`ヽ、
      /   /:::::; -‐''"        `ーノ
     /   /:::::/           \
     /    /::::::/          | | |  |
     |   |:::::/ /     |  | | | |  |
      |   |::/ / / |  | ||  | | ,ハ .| ,ハ|
      |   |/ / / /| ,ハノ| /|ノレ,ニ|ル' 
     |   |  | / / レ',二、レ′ ,ィイ|゙/   私は只の数ヲタなんかとは付き合わないわ。
.     |   \ ∠イ  ,イイ|    ,`-' |      頭が良くて数学が出来てかっこいい人。それが必要条件よ。
     |     l^,人|  ` `-'     ゝ  |        さらに Ann.of Math に論文書けば十分条件にもなるわよ。
      |      ` -'\       ー'  人          一番嫌いなのは論文数を増やすためにくだらない論文を書いて
    |        /(l     __/  ヽ、           良い論文の出版を遅らせるお馬鹿な人。
     |       (:::::`‐-、__  |::::`、     ヒニニヽ、         あなたの論文が Ann of Math に accept される確率は?
    |      / `‐-、::::::::::`‐-、::::\   /,ニニ、\            それとも最近は Inv. Math. の方が上かしら?
   |      |::::::::::::::::::|` -、:::::::,ヘ ̄|'、  ヒニ二、 \
.   |      /::::::::::::::::::|::::::::\/:::O`、::\   | '、   \
   |      /:::::::::::::::::::/:::::::::::::::::::::::::::::'、::::\ノ  ヽ、  |
  |      |:::::/:::::::::/:::::::::::::::::::::::::::::::::::'、',::::'、  /:\__/‐、
  |      |/:::::::::::/::::::::::::::::::::::::::::::::::O::| '、::| く::::::::::::: ̄|
   |     /_..-'´ ̄`ー-、:::::::::::::::::::::::::::::::::::|/:/`‐'::\;;;;;;;_|
   |    |/::::::::::::::::::::::\:::::::::::::::::::::::::::::|::/::::|::::/:::::::::::/
    |   /:::::::::::::::::::::::::::::::::|:::::::::::::::::::::O::|::|::::::|:::::::::::::::/
985132人目の素数さん:2012/05/07(月) 21:13:58.82
ume
986132人目の素数さん:2012/05/07(月) 21:20:12.82
うめ
987132人目の素数さん:2012/05/07(月) 21:23:10.62
産め
988132人目の素数さん:2012/05/07(月) 21:30:45.99
989132人目の素数さん:2012/05/07(月) 21:39:26.85
膿め
990132人目の素数さん:2012/05/07(月) 21:53:59.74
宇目
991132人目の素数さん:2012/05/07(月) 22:01:21.07
倦め
992132人目の素数さん:2012/05/07(月) 22:16:48.61
埋め
993132人目の素数さん:2012/05/07(月) 22:19:43.11
994132人目の素数さん:2012/05/07(月) 22:21:59.47
ウメ
995132人目の素数さん:2012/05/07(月) 22:24:44.98
熟め
996132人目の素数さん:2012/05/07(月) 22:30:31.68
ウメェ
997132人目の素数さん:2012/05/07(月) 22:30:46.20
績め
998132人目の素数さん:2012/05/07(月) 22:36:39.89
999132人目の素数さん:2012/05/07(月) 22:38:08.74
1000132人目の素数さん:2012/05/07(月) 22:39:37.52
10011001
このスレッドは1000を超えました。
もう書けないので、新しいスレッドを立ててくださいです。。。